You are on page 1of 46

JAVIER V. VERIDIANO 237 SCRA 565 FACTS: Javier filed a miscellaneous sales application over a parcel of land.

Thereafter, she instituted an action for forcible entry against Ben Babol, alleging that she was forcibly dispossessed of the parcel of land. Her complaint was dismissing, followed the finding of the Bureau of Lands that the disputed portion of land is outside the lot owned by Javier. Later, the sales application has been granted. Babol had already sold the land to somebody else. Petitioner demanded the return of the land to her and after 4 years since the dismissal of her earlier complaint, she files an action for quieting of title and recovery of possession against Babol and Rosete. The latter moved for the dismissal of the case based on res judicata. HELD: A judgment in a forcible entry or detainer case disposes of no other issue than possession and declares only who has the right of possession, but by no means constitutes a bar to an action for determination of who has right or title of ownership. HELD: Article 429 is inapplicable to the case at bar. The complainant didnt usurp or invade said lot. She had merely remained in possession thereof, even though the hacienda owner may have become its co-possessor. Appellants didnt repel or prevent an actual or threatened unlawful physical invasion or usurpation of the property. They expelled the complainant from a property on which she and her husband were in possession even before the action for forcible entry was filed against them.

SPOUSES

CRISTINO and BRIGIDA CUSTODIO and SPOUSES LITO and MARIA CRISTINA SANTOS, petitioners, vs. COURT OF APPEALS, HEIRS OF PACIFICO C. MABASA and REGIONAL TRIAL COURT OF PASIG, METRO MANILA, BRANCH 181, respondents.

DECISION REGALADO, J.:

GERMAN MANAGEMENT AND SERVICES V. COURT OF APPEALS 177 SCRA 495 This petition for review on certiorari assails the decision of respondent Court of Appeals in CA-G.R. CV No. 29115, promulgated on November 10, 1993, which affirmed with modification the decision of the trial court, as well as its resolution dated July 8, 1994 denying petitioners motion for reconsideration. [1] On August 26, 1982, Civil Case No. 47466 for the grant of an easement of right of way was filed by Pacifico Mabasa against Cristino Custodio, Brigida R. Custodio, Rosalina R. Morato, Lito Santos and Maria Cristina C. Santos before the Regional Trial Court of Pasig and assigned to Branch 22 thereof. [2] The generative facts of the case, as synthesized by the trial court and adopted by the Court of Appeals, are as follows:

FACTS: Spouses Jose issued a power of attorney in favor of petitioner for the development of their parcel of land into a subdivision. Private respondents were occupying the land and petitioner advised them to vacate but they refused. Thereafter, petitioner continued their development and construction. Respondents then filed a case for forcible entry. The trial court dismissed the complaint and this was reversed by the CA.

HELD: Notwithstanding petitioners claim that it was duly authorized by the owners to develop the subject property, private respondents as actual possessors, can commence a forcible entry case against petitioner because ownership is not in issue. Forcible entry is merely a quieting process, and never determines the actual title to an estate. Title is not involved. Although admittedly petitioner may validly claim ownership based on the muniment of title it presented, such evidence doesnt responsively address the issue of prior actual possession raised in a forcible entry case. It must be stated that regardless of the actual condition of title to the property, the party in a peaceable quiet possession shall not be turned out by a strong hand, violence or terror. Thus, a party who can prove prior possession can recover such possession even against the owner himself. Whatver may be the character of his prior possession, if he has in favor priority in time, he has security that entitles him to remain on the property until he has been lawfully ejected by a person having a better right by accion publiciana or accion reivindicatoria.

Perusing the record, this Court finds that the original plaintiff Pacifico Mabasa died during the pendency of this case and was substituted by Ofelia Mabasa, his surviving spouse [and children].

The plaintiff owns a parcel of land with a two-door apartment erected thereon situated at Interior P. Burgos St., Palingon, Tipas, Taguig, Metro Manila. The plaintiff was able to acquire said property through a contract of sale with spouses Mamerto Rayos and Teodora Quintero as vendors last September 1981. Said property may be described to be surrounded by other immovables pertaining to defendants herein. Taking P. Burgos Street as the point of reference, on the left side, going to plaintiffs property, the row of houses will be as follows: That of defendants Cristino and Brigido Custodio, then that of Lito and Maria Cristina Santos and then that of Ofelia Mabasa. On the right side (is) that of defendant Rosalina Morato and then a Septic Tank (Exhibit D). As an access to P. Burgos Street from plaintiffs property, there are two possible passageways. The first passageway is approximately one meter wide and is about 20 meters distan(t) from Mabasas residence to P. Burgos Street. Such path is passing in between the previously mentioned row of houses. The second passageway is about 3 meters in width and length from plaintiff Mabasas residence to P. Burgos Street; it is about 26 meters. In passing thru said passageway, a less than a meter wide path through the septic tank and with 5-6 meters in length has to be traversed.

CAISIP V. PEOPLE 36 SCRA 17 When said property was purchased by Mabasa, there were tenants occupying the premises and who were acknowledged by plaintiff Mabasa as tenants. However, sometime in February, 1982. one of said tenants vacated the apartment and when plaintiff Mabasa went to see the premises, he saw that there had been built an adobe fence in the first passageway making it narrower in width. Said adobe fence was first constructed by defendants Santoses along their property which is also along the first passageway. Defendant Morato constructed her adobe fence and even extended said fence in such a way that the entire passageway was enclosed (Exhibit 1-Santoses and Custodios, Exh. D for plaintiff, Exhs. 1-C, 1-D and I -E) And it was then that the remaining tenants of said apartment vacated the area. Defendant Ma. Cristina Santos testified that she constructed said fence because there was an incident when her daughter was dragged by a bicycle pedalled by a son of one of the tenants in said apartment along the first passageway. She also mentioned some other inconveniences of having (at) the front of her house a pathway such as when some of the tenants were drunk and would bang their doors and windows. Some of their footwear were even lost. x x x[3] (Italics in original text; corrections in parentheses supplied)

FACTS: Cabalag and her husband cultivated a parcel of land in Hacienda Palico, which was owned by Roxas y Cia. Caisip is the overseer of the land. The land cultivated was previously tenanted by the father of Cabalag. On a relevant date, Guevarra sought recognition as a lawful tenant from the CAR but such was dismissed as it was held that he wasnt a tenant thereof. Thereafter, Roxas filed a case for forcible entry against Guevarra, on which he won and the court ordered Guevarra to vacate the lot. Meanwhile, there was an altercation between Cabalag and Caisip over the cutting of sugar cane. She was being asked to leave by Caisip but she refused. Thereafter, she was charged with grave coercion. When she was again seen in the Hacienda and weeding, she was forcibly dragged by policemen and Caisip. This prompted for the filing a case for grave coercion.

On February 27, 1990, a decision was rendered by the trial court, with this dispositive part:

Page 1 of 46

Accordingly, judgment is hereby rendered as follows:

duty and the imposition of liability for that breach before damages may be awarded; it is not sufficient to state that there should be tort liability merely because the plaintiff suffered some pain and suffering) [11] Many accidents occur and many injuries are inflicted by acts or omissions which cause damage or loss to another but which violate no legal duty to such other person, and consequently create no cause of action in his favor. In such cases, the consequences must be borne by the injured person alone. The law affords no remedy for damages resulting from an act which does not amount to a legal injury or wrong. [12] In other words, in order that the law will give redress for an act causing damage, that act must be not only hurtful, but wrongful. There must be damnum et injuria.[13] If, as may happen in many cases, a person sustains actual damage, that is, harm or loss to his person or property, without sustaining any legal injury, that is, an act or omission which the law does not deem an injury, the damage is regarded as damnum absque injuria.[14] In the case at bar, although there was damage, there was no legal injury. Contrary to the claim of private respondents, petitioners could not be said to have violated the principle of abuse of right. In order that the principle of abuse of right provided in Article 21 of the Civil Code can be applied, it is essential that the following requisites concur: (1) The defendant should have acted in a manner that is contrary to morals, good customs or public policy; (2) The acts should be willful; and (3) There was damage or injury to the plaintiff. [15] The act of petitioners in constructing a fence within their lot is a valid exercise of their right as owners, hence not contrary to morals, good customs or public policy. The law recognizes in the owner the right to enjoy and dispose of a thing, without other limitations than those established by law.[16] It is within the right of petitioners, as owners, to enclose and fence their property. Article 430 of the Civil Code provides that (e)very owner may enclose or fence his land or tenements by means of walls, ditches, live or dead hedges, or by any other means without detriment to servitudes constituted thereon. At the time of the construction of the fence, the lot was not subject to any servitudes. There was no easement of way existing in favor of private respondents, either by law or by contract. The fact that private respondents had no existing right over the said passageway is confirmed by the very decision of the trial court granting a compulsory right of way in their favor after payment of just compensation. It was only that decision which gave private respondents the right to use the said passageway after payment of the compensation and imposed a corresponding duty on petitioners not to interfere in the exercise of said right. Hence, prior to said decision, petitioners had an absolute right over their property and their act of fencing and enclosing the same was an act which they may lawfully perform in the employment and exercise of said right. To repeat, whatever injury or damage may have been sustained by private respondents by reason of the rightful use of the said land by petitioners is damnum absque injuria.[17] A person has a right to the natural use and enjoyment of his own property, according to his pleasure, for all the purposes to which such property is usually applied. As a general rule, therefore, there is no cause of action for acts done by one person upon his own property in a lawful and proper manner, although such acts incidentally cause damage or an unavoidable loss to another, as such damage or loss is damnum absque injuria.[18] When the owner of property makes use thereof in the general and ordinary manner in which the property is used, such as fencing or enclosing the same as in this case, nobody can complain of having been injured, because the inconvenience arising from said use can be considered as a mere consequence of community life. [19] The proper exercise of a lawful right cannot constitute a legal wrong for which an action will lie, [20] although the act may result in damage to another, for no legal right has been invaded[21] One may use any lawful means to accomplish a lawful purpose and though the means adopted may cause damage to another, no cause of action arises in the latters favor. Any injury or damage occasioned thereby is damnum absque injuria. The courts can give no redress for hardship to an individual resulting from action reasonably calculated to achieve a lawful end by lawful means.[22] WHEREFORE, under the compulsion of the foregoing premises, the appealed decision of respondent Court of Appeals is hereby REVERSED and SET ASIDE and the judgment of the trial court is correspondingly REINSTATED. SO ORDERED. /---!e-library! 6.0 Philippines Copyright 2000 by Sony Valdez---\

1) Ordering defendants Custodios and Santoses to give plaintiff permanent access - ingress and egress, to the public street;

2) Ordering the plaintiff to pay defendants Custodios and Santoses the sum of Eight Thousand Pesos (P8,000) as indemnity for the permanent use of the passageway. The parties to shoulder their respective litigation expenses. [4]

Not satisfied therewith, therein plaintiff represented by his heirs, herein private respondents, went to the Court of Appeals raising the sole issue of whether or not the lower court erred in not awarding damages in their favor. On November 10, 1993, as earlier stated, the Court of Appeals rendered its decision affirming the judgment of the trial court with modification, the decretal portion of which disposes as follows:

WHEREFORE, the appealed decision of the lower court is hereby AFFIRMED WITH MODIFICATION only insofar as the herein grant of damages to plaintiffs-appellants. The Court hereby orders defendants-appellees to pay plaintiffs-appellants the sum of Sixty Five Thousand (P65,000) Pesos as Actual Damages, Thirty Thousand (P30,000) Pesos as Moral Damages, and Ten Thousand (P10,000) Pesos as Exemplary Damages. The rest of the appealed decision is affirmed to all respects. [5] On July 8, 1994, the Court of Appeals denied petitioners motion for reconsideration. [6] Petitioners then took the present recourse to us, raising two issues, namely, whether or not the grant of right of way to herein private respondents is proper, and whether or not the award of damages is in order. With respect to the first issue, herein petitioners are already barred from raising the same. Petitioners did not appeal from the decision of the court a quo granting private respondents the right of way, hence they are presumed to be satisfied with the adjudication therein. With the finality of the judgment of the trial court as to petitioners, the issue of propriety of the grant of right of way has already been laid to rest. For failure to appeal the decision of the trial court to the Court of Appeals, petitioners cannot obtain any affirmative relief other than those granted in the decision of the trial court. That decision of the court below has become final as against them and can no longer be reviewed, much less reversed, by this Court. The rule in this jurisdiction is that whenever an appeal is taken in a civil case, an appellee who has not himself appealed may not obtain from the appellate court any affirmative relief other than what was granted in the decision of the lower court. The appellee can only advance any argument that he may deem necessary to defeat the appellants claim or to uphold the decision that is being disputed, and he can assign errors in his brief if such is required to strengthen the views expressed by the court a quo. These assigned errors, in turn, may be considered by the appellate court solely to maintain the appealed decision on other grounds, but not for the purpose of reversing or modifying the judgment in the appellees favor and giving him other affirmative reliefs. [7] However, with respect to the second issue, we agree with petitioners that the Court of Appeals erred in awarding damages in favor of private respondents. The award of damages has no substantial legal basis. A reading of the decision of the Court of Appeals will show that the award of damages was based solely on the fact that the original plaintiff, Pacifico Mabasa, incurred losses in the form of unrealized rentals when the tenants vacated the leased premises by reason of the closure of the passageway. However, the mere fact that the plaintiff suffered losses does not give rise to a right to recover damages. To warrant the recovery of damages, there must be both a right of action for a legal wrong inflicted by the defendant, and damage resulting to the plaintiff therefrom. Wrong without damage, or damage without wrong, does not constitute a cause of action, since damages are merely part of the remedy allowed for the injury caused by a breach or wrong.[8] There is a material distinction between damages and injury. Injury is the illegal invasion of a legal right; damage is the loss, hurt, or harm which results from the injury; and damages are the recompense or compensation awarded for the damage suffered. Thus, there can be damage without injury in those instances in which the loss or harm was not the result of a violation of a legal duty. These situations are often called damnum absque injuria.[9] in order that a plaintiff may maintain an action for the injuries of which he complains, he must establish that such injuries resulted from a breach of duty which the defendant owed to the plaintiff - a concurrence of injury to the plaintiff and legal responsibility by the person causing it. [10] The underlying basis for the award of tort damages is the premise that an individual was injured in contemplation of law. Thus, there must first be the breach of some

Page 2 of 46

NATIVIDAD V. ANDAMO and EMMANUEL R. ANDAMO, petitioners, vs. INTERMEDIATE APPELLATE COURT (First Civil Cases Division) and MISSIONARIES OF OUR LADY OF LA SALETTE, INC., respondents.1990 November 063rd DivisionG.R. No. 74761D E C I S I O N FERNAN, J.: The pivotal issue in this petition for certiorari, prohibition and mandamus is whether a corporation, which has built through its agents, waterpaths, water conductors and contrivances within its land, thereby causing inundation and damage to an adjacent land, can be held civilly liable for damages under Articles 2176 and 2177 of the Civil Code on quasi-delicts such that the resulting civil case can proceed independently of the criminal case. The antecedent facts are as follows:

Directly at issue is the propriety of the dismissal of Civil Case: No. TG-748 in accordance with Section 3 (a) of Rule 111 of the Pules of Court. Petitioners contend that the trial court and the Appellate Court erred in dismissing Civil Case No. TG-748 since it is predicated on a quasi-delict. Petitioners have raised a valid point. It is axiomatic that the nature of an action filed in court is determined by the facts alleged in the complaint as constituting the cause of action. 7 The purpose of an action or suit and the law to govern it, including the period of prescription, is to be determined not by the claim of the party filing the action, made in his argument or brief, but rather by the complaint itself, its allegations and prayer for relief. 8 The nature of an action is not necessarily determined or controlled by its title or heading but by the body of the pleading or complaint itself. To avoid possible denial of substantial justice due to legal technicalities, pleadings as well as remedial laws should be liberally construed so that the litigants may have ample opportunity to prove their respective claims. 9 Quoted hereunder are the pertinent portions of petitioners' complaint in Civil Case No. TG-748:

Petitioner spouses Emmanuel and Natividad Andamo are the owners of a parcel of land situated in Biga (Biluso) Silang, Cavite which is adjacent to that of private respondent, Missionaries of Our Lady of La Salette, Inc., a religious corporation. Within the land of respondent corporation, waterpaths and contrivances, including an artificial lake, were constructed, which allegedly inundated and eroded petitioners' land, caused a young man to drown, damaged petitioners' crops and plants, washed away costly fences, endangered the lives of petitioners and their laborers during rainy and stormy seasons, and exposed plants and other improvements to destruction. In July 1982, petitioners instituted a criminal action, docketed as Criminal Case No. TG 907-82, before the Regional Trial Court of Cavite, Branch 4 (Tagaytay City), against Efren Musngi, Orlando Sapuay and Rutillo Mallillin, officers and directors of herein respondent corporation, for destruction by means of injunction under Article 324 of the Revised Penal Code. Subsequently, on February 22, 1983, petitioners filed another action against respondent corporation, this time a civil case, docketed as Civil Case No. TG-748, for damages with prayer for the issuance of a writ of preliminary injunction before the same court. 1 On March 11, 1983, respondent corporation filed its answer to the complaint and opposition to the issuance of a writ of preliminary injunction. Hearings were conducted including ocular inspections on the land. However, on April 26, 1984, the trial court, acting on respondent corporation's motion to dismiss or suspend the civil action, issued an order suspending further hearings in Civil Case No. TG-748 until after judgment in the related Criminal Case No. TG-907-82.

4) That within defendant's land, likewise located at Biga (Biluso), Silang, Cavite, adjacent on the right side of the aforesaid land of plaintiffs, defendant constructed waterpaths starting from the middle-right portion thereof leading to a big hole or opening, also constructed by defendant, thru the lower portion of its concrete hollow-blocks fence situated on the right side of its cemented gate fronting the provincial highway, and connected by defendant to a man-height inter-connected cement culverts which were also constructed and lain by defendant cross-wise beneath the tip of the said cemented gate, the left-end of the said inter-connected culverts again connected by defendant to a big hole or opening thru the lower portion of the same concrete hollow-blocks fence on the left side of the said cemented gate, which hole or opening is likewise connected by defendant to the cemented mouth of a big canal, also constructed by defendant, which runs northward towards a big hole or opening which was also built by defendant thru the lower portion of its concrete hollow-blocks fence which separates the land of plaintiffs from that of defendant (and which serves as the exit-point of the floodwater coming from the land of defendant, and at the same time, the entrance-point of the same floodwater to the land of plaintiffs, year after year, during rainy or stormy seasons. "5) That moreover, on the middle-left portion of its land just beside the land of plaintiffs, defendant also constructed an artificial lake, the base of which is soil, which utilizes the water being channeled thereto from its water system thru inter-connected galvanized iron pipes (No. 2) and complimented by rain water during rainy or stormy seasons, so much so that the water below it seeps into, and the excess water above it inundates, portions of the adjoining land of plaintiffs. "6) That as a result of the inundation brought about by defendant's aforementioned water conductors, contrivances and manipulators, a young man was drowned to death, while herein plaintiffs suffered and will continue to suffer, as follows: "a) Portions of the land of plaintiffs were eroded and converted to deep, wide and long canals, such that the same can no longer be planted to any crop or plant.

Resolving respondent corporation's motion to dismiss filed on June 22, 1984, the trial court issued on August 27,1984 the disputed order dismissing Civil Case No. TG-748 for lack of jurisdiction, as the criminal case which was instituted ahead of the civil case was still unresolved. Said order was anchored on the provision of Section 3 (a), Rule III of the Rules of Court which provides that "criminal and civil actions arising from the same offense may be instituted separately, but after the criminal action has been commenced the civil action cannot be instituted until final judgment has been rendered in the criminal action." 2 Petitioners appealed from that order to the Intermediate Appellate Court. 3 On February 17, 1986, respondent Appellate Court, First Civil Cases Division, promulgated a decision, 4 affirming the questioned order of the trial court. 5 A motion for reconsideration filed by petitioners was denied by the Appellate Court in its resolution dated May 19, 1986. 6

"b) Costly fences constructed by plaintiffs were, on several occasions, washed away. "c) during rainy and stormy seasons the lives of plaintiffs and their laborers are always in danger. "d) Plants and other improvements on other portions of the land of plaintiffs are exposed to destruction. . . ." 10 A careful examination of the aforequoted complaint shows that the civil action is one under Articles 2176 and 2177 of the Civil Code on quasi-delicts. All the elements of a quasi-delict are present, to wit: (a) damages suffered by the plaintiff; (b) fault or negligence of the defendant, or some other person for whose acts he must respond; and (c) the connection of cause and effect between the fault or negligence of the defendant and the damages incurred by the plaintiff. 11

Page 3 of 46

Clearly, from petitioners' complaint, the waterpaths and contrivances built by respondent corporation are alleged to have inundated the land of petitioners. There is therefore, an assertion of a causal connection between the act of building these waterpaths and the damage sustained by petitioners. Such action if proven constitutes fault or negligence which may be the basis for the recovery of damages. In the case of Samson vs. Dionisio, 12 the Court applied Article 1902, now Article 2176 of the Civil Code and held that "any person who without due authority constructs a bank or dike, stopping the flow or communication between a creek or a lake and a river, thereby causing loss and damages to a third party who, like the rest of the residents, is entitled to the use and enjoyment of the stream or lake, shall be liable to the payment of an indemnity for loss and damages to the injured party." While the property involved in the cited case belonged to the public domain and the property subject of the instant case is privately owned, the fact remains that petitioners' complaint sufficiently alleges that petitioners have sustained and will continue to sustain damage due to the waterpaths and contrivances built by respondent corporation. Indeed, the recitals of the complaint, the alleged presence of damage to the petitioners, the act or omission of respondent corporation supposedly constituting fault or negligence, and the causal connection between the act and the damage, with no pre-existing contractual obligation between the parties make a clear case of a quasi-delict or culpa aquiliana. It must be stressed that the use of one's property is not without limitations. Article 431 of the Civil Code provides that "the owner of a thing cannot make use thereof in such a manner as to injure the rights of a third person." SIC UTERE TUO UT ALIENUM NON LAEDAS. Moreover, adjoining landowners have mutual and reciprocal duties which require that each must use his own land in a reasonable manner so as not to infringe upon the rights and interests of others. Although we recognize the right of an owner to build structures on his land, such structures must be so constructed and maintained using all reasonable care so that they cannot be dangerous to adjoining landowners and can withstand the usual and expected forces of nature. If the structures cause injury or damage to an adjoining landowner or a third person, the latter can claim indemnification for the injury or damage suffered. Article 2176 of the Civil Code imposes a civil liability on a person for damage caused by his act or omission constituting fault or negligence, thus: "Article 2176. Whoever by act or omission causes damage to another, there being fault or negligence, is obliged to pay for the damage done. Such fault or negligence, if there is no pre-existing contractual relation between the parties, is called a quasi-delict is governed by the provisions of this chapter."

of the criminal law, while the latter is a distinct and independent negligence, which is a "culpa aquiliana" or quasidelict, of ancient origin, having always had its own foundation and individuality, separate from criminal negligence. Such distinction between criminal negligence and "culpa extra-contractual" or "cuasi-delito" has been sustained by decisions of the Supreme Court of Spain . . ." 14 In the case of Castillo vs. Court of Appeals, 15 this Court held that a quasi-delict or culpa aquiliana is a separate legal institution under the Civil Code with a substantivity all its own, and individuality that is entirely apart and independent from a delict or crime a distinction exists between the civil liability arising from a crime and the responsibility for quasi-delicts or culpa extra-contractual. The same negligence causing damages may produce civil liability arising from a crime under the Penal Code, or create an action for quasi-delicts or culpa extra-contractual under the Civil Code. Therefore, the acquittal or conviction in the criminal case is entirely irrelevant in the civil case, unless, of course, in the event of an acquittal where the court has declared that the fact from which the civil action arose did not exist, in which case the extinction of the criminal liability would carry with it the extinction of the civil liability.

In Azucena vs. Potenciano, 16 the Court declared that in quasi-delicts, "(t)he civil action is entirely independent of the criminal case according to Articles 33 and 2177 of the Civil Code. There can be no logical conclusion than this, for to subordinate the civil action contemplated in the said articles to the result of the criminal prosecution whether it be conviction or acquittal would render meaningless the independent character of the civil action and the clear injunction in Article 31, that his action may proceed independently of the criminal proceedings and regardless of the result of the latter." WHEREFORE, the assailed decision dated February 17, 1986 of the then Intermediate Appellate Court affirming the order of dismissal of the Regional Trial Court of Cavite, Branch 18 (Tagaytay City) dated August 17, 1984 is hereby REVERSED and SET ASIDE. The trial court is ordered to reinstate Civil Case No. TG-748 entitled "Natividad V. Andamo and Emmanuel R. Andamo vs. Missionaries of Our Lady of La Salette, Inc." and to proceed with the hearing of the case with dispatch. This decision is immediately executory. Costs against respondent corporation. SO ORDERED. FIRST DIVISION

Article 2176, whenever it refers to "fault or negligence", covers not only acts "not punishable by law" but also acts criminal in character, whether intentional and voluntary or negligent. Consequently, a separate civil action lies against the offender in a criminal act, whether or not he is criminally prosecuted and found guilty or acquitted, provided that the offended party is not allowed, (if the tortfeasor is actually charged also criminally), to recover damages on both scores, and would be entitled in such eventuality only to the bigger award of the two, assuming the awards made in the two cases vary. 13 The distinctness of quasi-delicts is shown in Article 2177 of the Civil Code, which states:

NATIONAL POWER CORPORATION, Petitioner,

G.R. No. 168732

Present: -versus"Article 2177. Responsibility for fault or negligence under the preceding article is entirely separate and distinct from the civil liability arising from negligence under the Penal Code. But the plaintiff cannot recover damages twice for the same act or omission of the defendant." LUCMAN G. IBRAHIM, OMAR According to the Report of the Code Commission "the foregoing provision though at first sight startling, is not so novel or extraordinary when we consider the exact nature of criminal and civil negligence. The former is a violation PUNO, C.J., Chairperson,

Page 4 of 46

G. MARUHOM, ELIAS G. MARUHOM, BUCAY G. MARUHOM, FAROUK G. MARUHOM, HIDJARA G. MARUHOM, ROCANIA G. MARUHOM, POTRISAM G. MARUHOM, LUMBA G. MARUHOM, SINAB G. MARUHOM, ACMAD G. MARUHOM, SOLAYMAN G. MARUHOM, MOHAMAD M. IBRAHIM, and CAIRONESA M. IBRAHIM,

SANDOVAL-GUTIERREZ,* CORONA, AZCUNA, and GARCIA, JJ.

2, and 3 consisting of 31,894, 14,915, and 23,191 square meters each respectively. Sometime in 1978, NAPOCOR, through alleged stealth and without respondents knowledge and prior consent, took possession of the sub-terrain area of their lands and constructed therein underground tunnels. The existence of the tunnels was only discovered sometime in July 1992 by respondents and then later confirmed on November 13, 1992 by NAPOCOR itself through a memorandum issued by the latters Acting Assistant Project Manager. The tunnels were apparently being used by NAPOCOR in siphoning the water of Lake Lanao and in the operation of NAPOCORs Agus II, III, IV, V, VI, VII projects located in Saguiran, Lanao del Sur; Nangca and Balo-i in Lanao del Norte; and Ditucalan and Fuentes in Iligan City. On September 19, 1992, respondent Omar G. Maruhom requested the Marawi City Water District for

Promulgated:

a permit to construct and/or install a motorized deep well in Lot 3 located in Saduc, Marawi City but his request was turned down because the construction of the deep well would cause danger to lives and property. On October 7, 1992, respondents demanded that NAPOCOR pay damages and vacate the sub-terrain portion of their lands but the latter refused to vacate much less pay damages. Respondents further averred that the construction of the underground tunnels has endangered their lives and properties as Marawi City lies in an area of local volcanic and tectonic activity. Further, these illegally constructed tunnels caused them sleepless nights, serious anxiety and June 29, 2007 shock thereby entitling them to recover moral damages and that by way of example for the public good, NAPOCOR must be held liable for exemplary damages. Disputing respondents claim, NAPOCOR filed an answer with counterclaim denying the material allegations of the complaint and interposing affirmative and special defenses, namely that (1) there is a failure to state a cause of action since respondents seek possession of the sub-terrain portion when they were never in possession of the same, (2) respondents have no cause of action because they failed to show proof that they were

Respondents. DECISION AZCUNA, J.: This is a petition for review on certiorari under Rule 45 of the Rules of Court seeking to annul the Decision1[1] dated June 8, 2005 rendered by the Court of Appeals (CA) in C.A.-G.R. CV No. 57792. The facts are as follows: On November 23, 1994, respondent Lucman G. Ibrahim, in his personal capacity and in behalf of his co-heirs Omar G. Maruhom, Elias G. Maruhom, Bucay G. Maruhom, Mamod G. Maruhom, Farouk G. Maruhom, Hidjara G. Maruhom, Rocania G. Maruhom, Potrisam G. Maruhom, Lumba G. Maruhom, Sinab G. Maruhom, Acmad G. Maruhom, Solayman G. Maruhom, Mohamad M. Ibrahim and Caironesa M. Ibrahim, instituted an action against petitioner National Power Corporation (NAPOCOR) for recovery of possession of land and damages before the Regional Trial Court (RTC) of Lanao del Sur. In their complaint, Ibrahim and his co-heirs claimed that they were owners of several parcels of land described in Survey Plan FP (VII-5) 2278 consisting of 70,000 square meters, divided into three (3) lots, i.e. Lots 1,

the owners of the property, and (3) the tunnels are a government project for the benefit of all and all private lands are subject to such easement as may be necessary for the same. 2[2] On August 7, 1996, the RTC rendered a Decision, the decretal portion of which reads as follows: WHEREFORE, judgment is hereby rendered: 1. Denying plaintiffs [private respondents] prayer for defendant [petitioner] National Power Corporation to dismantle the underground tunnels constructed between the lands of plaintiffs in Lots 1, 2, and 3 of Survey Plan FP (VII-5) 2278;

2. Ordering defendant to pay to plaintiffs the fair market value of said 70,000 square meters of land covering Lots 1, 2, and 3 as described in Survey Plan FP (VII-5) 2278 less the area of 21,995 square meters at P1,000.00 per square meter or a total of P48,005,000.00 for the remaining unpaid portion of 48,005 square meters; with 6% interest per annum from the filing of this case until paid;

3. Ordering defendant to pay plaintiffs a reasonable monthly rental of P0.68 per square meter of the total area of 48,005 square meters effective from its occupancy of the foregoing area in 1978 or a total of P7,050,974.40.

* 1[1]

On Leave. Rollo, pp. 114-133.

2[2]

Id. at 117-118.

Page 5 of 46

4. Ordering defendant to pay plaintiffs the sum of P200,000.00 as moral damages; and

4)

they were prevented by fraud, mistake, accident, or excusable negligence from taking legal steps to protect and preserve their rights over their parcels of land in so far as the part of the decision decreeing just compensation for petitioners properties;

5. Ordering defendant to pay the further sum of P200,000.00 as attorneys fees and the costs. 5) SO ORDERED.3[3] they would never have agreed to the alienation of their property in favor of anybody, considering the fact that the parcels of land involved in this case were among the valuable properties they inherited from their dear father and they would rather see their land crumble to dust than sell it to anybody.4[4]

The RTC granted the petition and rendered a modified judgment dated September 8, 1997, thus: On August 15, 1996, Ibrahim, joined by his co-heirs, filed an Urgent Motion for Execution of Judgment Pending Appeal. On the other hand, NAPOCOR filed a Notice of Appeal by registered mail on August 19, 1996. Thereafter, NAPOCOR filed a vigorous opposition to the motion for execution of judgment pending appeal with a motion for reconsideration of the Decision which it had received on August 9, 1996. 1) On August 26, 1996, NAPOCOR filed a Manifestation and Motion withdrawing its Notice of Appeal purposely to give way to the hearing of its motion for reconsideration. On August 28, 1996, the RTC issued an Order granting execution pending appeal and denying NAPOCORs motion for reconsideration, which Order was received by NAPOCOR on September 6, 1996. On September 9, 1996, NAPOCOR filed its Notice of Appeal by registered mail which was denied by the RTC on the ground of having been filed out of time. Meanwhile, the Decision of the RTC was executed pending appeal and funds of NAPOCOR were garnished by respondents Ibrahim and his co-heirs. On October 4, 1996, a Petition for Relief from Judgment was filed by respondents Omar G. Maruhom, Elias G. Maruhom, Bucay G. Maruhom, Mamod G. Maruhom, Farouk G. Maruhom, Hidjara G. Maruhom, Potrisam G. Maruhom and Lumba G. Maruhom asserting as follows: 1) they did not file a motion to reconsider or appeal the decision within the reglementary period of fifteen (15) days from receipt of judgment because they believed in good faith that the decision was for damages and rentals and attorneys fees only as prayed for in the complaint: 3) Ordering defendant embodied in the August 7, 1996 decision to pay plaintiffs the sum of P200,000.00 as moral damages; and further sum of P200,000.00 as attorneys fees and costs. 2) Reducing the judgment award of plaintiffs for the fair market value of P48,005,000.00 by 9,526,000.00 or for a difference by P38,479,000.00 and by the further sum of P33,603,500.00 subject of the execution pending appeal leaving a difference of 4,878,500.00 which may be the subject of execution upon the finality of this modified judgment with 6% interest per annum from the filing of the case until paid. WHEREFORE, a modified judgment is hereby rendered:

Awarding the sum of P1,476,911.00 to herein petitioners Omar G. Maruhom, Elias G. Maruhom, Bucay G. Maruhom, Mahmod G. Maruhom, Farouk G. Maruhom, Hidjara G. Maruhom, Portrisam G. Maruhom and Lumba G. Maruhom as reasonable rental deductible from the awarded sum of P7,050,974.40 pertaining to plaintiffs.

SO ORDERED.5[5]

2)

it was only on August 26, 1996 that they learned that the amounts awarded to the plaintiffs represented not only rentals, damages and attorneys fees but the greatest portion of which was payment of just compensation which in effect would make the defendant NPC the owner of the parcels of land involved in the case;

Subsequently, both respondent Ibrahim and NAPOCOR appealed to the CA. In the Decision dated June 8, 2005, the CA set aside the modified judgment and reinstated the original Decision dated August 7, 1996, amending it further by deleting the award of moral damages and reducing

3)

when they learned of the nature of the judgment, the period of appeal has already expired;

the amount of rentals and attorneys fees, thus:

4[4] 3[3]

Id. at 118-119.

5[5]

Id. at 121-122. Id. at 122-123.

Page 6 of 46

WHEREFORE, premises considered, herein Appeals are hereby partially GRANTED, the Modified Judgment is ordered SET ASIDE and rendered of no force and effect and the original Decision of the court a quo dated 7 August 1996 is hereby RESTORED with the MODIFICATION that the award of moral damages is DELETED and the amounts of rentals and attorneys fees are REDUCED to P6,888,757.40 and P50,000.00, respectively.

the underground tunnels 115 meters below respondents property could not have caused damage or prejudice to respondents and their claim to this effect was, therefore, purely conjectural and speculative.7[7] The contention lacks merit. Generally, in an appeal by certiorari under Rule 45 of the Rules of Court, the Court does not pass upon questions of fact. Absent any showing that the trial and appellate courts gravely abused their discretion, the

In this connection, the Clerk of Court of RTC Lanao del Sur is hereby directed to reassess and determine the additional filing fee that should be paid by Plaintiff-Appellant IBRAHIM taking into consideration the total amount of damages sought in the complaint vis--vis the actual amount of damages awarded by this Court. Such additional filing fee shall constitute a lien on the judgment. SO ORDERED.6[6]

Court will not examine the evidence introduced by the parties below to determine if they correctly assessed and evaluated the evidence on record.8[8] The jurisdiction of the Court in cases brought to it from the CA is limited to reviewing and revising the errors of law imputed to it, its findings of fact being as a rule conclusive and binding on the Court. In the present case, petitioner failed to point to any evidence demonstrating grave abuse of discretion on the part of the CA or to any other circumstances which would call for the application of the exceptions to the above rule. Consequently, the CAs findings which upheld those of the trial court that respondents owned and possessed the property and that its substrata was possessed by petitioner since 1978 for the underground tunnels,

Hence, this petition ascribing the following errors to the CA:

cannot be disturbed. Moreover, the Court sustains the finding of the lower courts that the sub-terrain portion of the property similarly belongs to respondents. This conclusion is drawn from Article 437 of the Civil Code which provides:

(a)

RESPONDENTS WERE NOT DENIED THE BENEFICIAL USE OF THEIR SUBJECT PROPERTIES TO ENTITLE THEM TO JUST COMPENSATION BY WAY OF DAMAGES; ART. 437. The owner of a parcel of land is the owner of its surface and of everything under it, and he can construct thereon any works or make any plantations and excavations which he may deem proper, without detriment to servitudes and subject to special laws and ordinances. He cannot complain of the reasonable requirements of aerial navigation. Thus, the ownership of land extends to the surface as well as to the subsoil under it. In Republic of the Philippines v. Court of Appeals,9[9] this principle was applied to show that rights over lands are indivisible and, consequently, require a definitive and categorical classification, thus:

(b)

ASSUMING THAT RESPONDENTS ARE ENTITLED TO JUST COMPENSATION BY WAY OF DAMAGES, NO EVIDENCE WAS PRESENTED ANENT THE VALUATION OF RESPONDENTS PROPERTY AT THE TIME OF ITS TAKING IN THE YEAR 1978 TO JUSTIFY THE AWARD OF ONE THOUSAND SQUARE METERS (P1000.00/SQ. M.) EVEN AS PAYMENT OF BACK RENTALS IS ITSELF IMPROPER.

This case revolves around the propriety of paying just compensation to respondents, and, by extension, the basis for computing the same. The threshold issue of whether respondents are entitled to just compensation hinges upon who owns the sub-terrain area occupied by petitioner. Petitioner maintains that the sub-terrain portion where the underground tunnels were constructed does not belong to respondents because, even conceding the fact that respondents owned the property, their right to the subsoil of the same does not extend beyond what is necessary to enable them to obtain all the utility and convenience that such property can normally give. In any case, petitioner asserts that respondents were still able to use the subject property even with the existence of the tunnels, citing as an example the fact that one of the respondents, Omar G. Maruhom, had established his residence on a part of the property. Petitioner concludes that The Court of Appeals justified this by saying there is no conflict of interest between the owners of the surface rights and the owners of the sub-surface rights. This is rather strange doctrine, for it is a well-known principle that the owner of a piece of land has rights not only to its surface but also to everything underneath and the airspace above it up to a reasonable height. Under the aforesaid ruling, the land is classified as mineral underneath and agricultural on the surface, subject to separate claims of title. This is also difficult to understand, especially in its practical application. Under the theory of the respondent court, the surface owner will be planting on the land while the mining locator will be boring tunnels underneath. The farmer cannot dig a well because he may interfere with the mining operations below

7[7] 8[8] 6[6]

Id. at 95-98. Concepcion v. CA, G.R. No. 120707, January 31, 2000, 324 SCRA G.R. No. L-43938, April 15, 1988, 160 SCRA 228.

85. Id. at 132-133.

9[9]

Page 7 of 46

and the miner cannot blast a tunnel lest he destroy the crops above. How deep can the farmer, and how high can the miner go without encroaching on each others rights? Where is the dividing line between the surface and the sub-surface rights? The Court feels that the rights over the land are indivisible and that the land itself cannot be half agricultural and half mineral. The classification must be categorical; the land must be either completely mineral or completely agricultural.

extensive earth movements and is considered to be a drowned basin of volcano/tectonic origin. In Marawi City, there are a number of former volcanoes and an extensive amount of faulting. Some of these faults are still moving. (Feasibility Report on Marawi City Water District by Kampsa-Kruger, Consulting Engineers, Architects and Economists, Exh. R). Moreover, it has been shown that the underground tunnels [have] deprived the plaintiffs of the lawful use of the land and considerably reduced its value. On March 6, 1995, plaintiffs applied for a two-million peso loan with the Amanah Islamic Bank for the expansion of the operation of the Ameer Construction and Integrated Services to be secured by said land (Exh. N), but the application was disapproved by the bank in its letter of April 25, 1995 (Exh. O) stating that: Apropos to this, we regret to inform you that we cannot consider your loan application due to the following reasons, to wit: That per my actual ocular inspection and verification, subject property offered as collateral has an existing underground tunnel by the NPC for the Agus I Project, which tunnel is traversing underneath your property, hence, an encumbrance. As a matter of bank policy, property with an existing encumbrance cannot be considered neither accepted as collateral for a loan.

Registered landowners may even be ousted of ownership and possession of their properties in the event the latter are reclassified as mineral lands because real properties are characteristically indivisible. For the loss sustained by such owners, they are entitled to just compensation under the Mining Laws or in appropriate expropriation proceedings.10[10] Moreover, petitioners argument that the landowners right extends to the sub-soil insofar as necessary for their practical interests serves only to further weaken its case. The theory would limit the right to the sub-soil upon the economic utility which such area offers to the surface owners. Presumably, the landowners right extends to such height or depth where it is possible for them to obtain some benefit or enjoyment, and it is extinguished beyond such limit as there would be no more interest protected by law. 11[11] In this regard, the trial court found that respondents could have dug upon their property motorized deep wells but were prevented from doing so by the authorities precisely because of the construction and existence of the tunnels underneath the surface of their property. Respondents, therefore, still had a legal interest in the subterrain portion insofar as they could have excavated the same for the construction of the deep well. The fact that they could not was appreciated by the RTC as proof that the tunnels interfered with respondents enjoyment of their property and deprived them of its full use and enjoyment, thus: Has it deprived the plaintiffs of the use of their lands when from the evidence they have already existing residential houses over said tunnels and it was not shown that the tunnels either destroyed said houses or disturb[ed] the possession thereof by plaintiffs? From the evidence, an affirmative answer seems to be in order. The plaintiffs and [their] co-heirs discovered [these] big underground tunnels in 1992. This was confirmed by the defendant on November 13, 1992 by the Acting Assistant Project Manager, Agus 1 Hydro Electric Project (Exh. K). On September 16, 1992, Atty. Omar Maruhom (co-heir) requested the Marawi City Water District for permit to construct a motorized deep well over Lot 3 for his residential house (Exh. Q). He was refused the permit because the construction of the deep well as (sic) the parcels of land will cause danger to lives and property. He was informed that beneath your lands are constructed the Napocor underground tunnel in connection with Agua Hydroelectric plant (Exh. Q-2). There in fact exists ample evidence that this construction of the tunnel without the prior consent of plaintiffs beneath the latters property endangered the lives and properties of said plaintiffs. It has been proved indubitably that Marawi City lies in an area of local volcanic and tectonic activity. Lake Lanao has been formed by

All the foregoing evidence and findings convince this Court that preponderantly plaintiffs have established the condemnation of their land covering an area of 48,005 sq. meters located at Saduc, Marawi City by the defendant National Power Corporation without even the benefit of expropriation proceedings or the payment of any just compensation and/or reasonable monthly rental since 1978.12[12] In the past, the Court has held that if the government takes property without expropriation and devotes the property to public use, after many years, the property owner may demand payment of just compensation in the event restoration of possession is neither convenient nor feasible.13[13] This is in accordance with the principle that persons shall not be deprived of their property except by competent authority and for public use and always upon payment of just compensation.14[14] Petitioner contends that the underground tunnels in this case constitute an easement upon the property of respondents which does not involve any loss of title or possession. The manner in which the easement was created by petitioner, however, violates the due process rights of respondents as it was without notice and indemnity to them and did not go through proper expropriation proceedings. Petitioner could have, at any time, validly exercised the power of eminent domain to acquire the easement over respondents property as this power encompasses not only the taking or appropriation of title to and possession of the expropriated property but likewise covers even the imposition of a mere burden upon the owner of the condemned property. 15[15] Significantly,

12[12] 13[13]

10[10] 11[11]

Id. TOLENTINO, COMMENTARIES AND JURISPRUDENCE ON THE CIVIL CODE, Vol. II, p. 90.

14[14] 15[15]

Rollo, pp. 152-154. Militante v. Court of Appeals, 386 Phil. 522 (2000). CONSTITUTION, Art. III, Sec. 9. See also CIVIL CODE, Art. 435. Republic v. PLDT, 136 Phil. 20 (1969).

Page 8 of 46

though, landowners cannot be deprived of their right over their land until expropriation proceedings are instituted in court. The court must then see to it that the taking is for public use, that there is payment of just compensation and that there is due process of law.16[16] In disregarding this procedure and failing to recognize respondents ownership of the sub-terrain portion, petitioner took a risk and exposed itself to greater liability with the passage of time. It must be emphasized that the acquisition of the easement is not without expense. The underground tunnels impose limitations on respondents use of the property for an indefinite period and deprive them of its ordinary use. Based upon the foregoing, respondents are clearly entitled to the payment of just compensation.17[17] Notwithstanding the fact that petitioner only occupies the sub-terrain portion, it is liable to pay not merely an easement fee but rather the full compensation for land. This is so because in this case, the nature of the easement practically deprives the owners of its normal beneficial use. Respondents, as the owners of the property thus expropriated, are entitled to a just compensation which should be neither more nor less, whenever it is possible to make the assessment, than the money equivalent of said property.18[18]

purpose of devoting the property to a public use in such a manner as to oust the owner and deprive him of all beneficial enjoyment thereof.23[23] Moreover, taking of the property for purposes of eminent domain entails that the entry into the property must be under warrant or color of legal authority.24[24] Under the factual backdrop of this case, the last element of taking mentioned, i.e., that the entry into the property is under warrant or color of legal authority, is patently lacking. Petitioner justified its nonpayment of the indemnity due respondents upon its mistaken belief that the property formed part of the public dominion. This situation is on all fours with that in the Mangondato case. NAPOCOR in that case took the property of therein respondents in 1979, using it to build its Aqua I Hydroelectric Plant Project, without paying any compensation, allegedly under the mistaken belief that it was public land. It was only in 1990, after more than a decade of beneficial use, that NAPOCOR recognized therein respondents ownership and negotiated for the voluntary purchase of the property. In Mangondato, this Court held:

The entitlement of respondents to just compensation having been settled, the issue now is on the manner of computing the same. In this regard, petitioner claims that the basis for the computation of the just compensation should be the value of the property at the time it was taken in 1978. Petitioner also impugns the reliance made by the CA upon National Power Corporation v. Court of Appeals and Macapanton Mangondato 19[19] as the basis for computing the amount of just compensation in this action. The CA found that the award of damages is not excessive because the P1000 per square meter as the fair market value was sustained in a case involving a lot adjoining the property in question which case involved an expropriation by [petitioner] of portion of Lot 1 of the subdivision plan (LRC) PSD 116159 which is adjacent to Lots 2 and 3 of the same subdivision plan which is the subject of the instant controversy. 20[20] Sec. 4. Order of Condemnation. When such a motion is overruled or when any party fails to defend as required by this rule, the court may enter an order of condemnation declaring that the plaintiff has a lawful right to take the property sought to be condemned, for the public use or purpose described in the complaint, upon the payment of just compensation to be determined as of the date of the filing of the complaint. x x x (Italics supplied). Normally, the time of the taking coincides with the filing of the complaint for expropriation. Hence, many ruling of this Court have equated just compensation with the value of the property as of the time of filing of the complaint consistent with the above provision of the Rules. So too, where the institution of the action precedes entry to the property, the just compensation is to be ascertained as of the time of filing of the complaint. The general rule, however, admits of an exception: where this Court fixed the value of the property as of the date it was taken and not the date of the commencement of the expropriation proceedings. In the old case of Provincial Government of Rizal vs. Caro de Araullo, the Court ruled that x x x the owners of the land have no right to recover damages for this unearned increment resulting from the construction of the public improvement (lengthening of Taft Avenue from Manila to Pasay) from which the land was taken. To The First Issue: Date of Taking or Date of Suit? The general rule in determining just compensation in eminent domain is the value of the property as of the date of the filing of the complaint, as follows:

Just compensation has been understood to be the just and complete equivalent of the loss 21[21] and is ordinarily determined by referring to the value of the land and its character at the time it was taken by the expropriating authority.22[22] There is a taking in this sense when the owners are actually deprived or dispossessed of their property, where there is a practical destruction or a material impairment of the value of their property, or when they are deprived of the ordinary use thereof. There is a taking in this context when the expropriator enters private property not only for a momentary period but for more permanent duration, for the

16[16]

NAPOCOR v. CA, G.R. No. 106804, August 12, 2004, 436 SCRA

195.

17[17]

NAPOCOR v. Gutierrez, G.R. No. 60077, January 18, 1991, 193 SCRA 1. 18[18] Id. 19[19] G.R. No. 113194, March 11, 1996, 254 SCRA 577. 20[20] Rollo, p. 130. 21[21] Supra note 16. 22[22] Supra note 17.

23[23]

Republic of the Philippines v. Sarabia, G.R. No. 157847, August 25, 2005, 468 SCRA 142. 24[24] Supra note 19.

Page 9 of 46

permit them to do so would be to allow them to recover more than the value of the land at the time it was taken, which is the true measure of the damages, or just compensation, and would discourage the construction of important public improvements. In subsequent cases, the Court, following the above doctrine, invariably held that the time of taking is the critical date in determining lawful or just compensation. Justifying this stance, Mr. Justice (later Chief Justice) Enrique Fernando, speaking for the Court in Municipality of La Carlota vs. The Spouses Felicidad Baltazar and Vicente Gan, said, x x x the owner as is the constitutional intent, is paid what he is entitled to according to the value of the property so devoted to public use as of the date of taking. From that time, he had been deprived thereof. He had no choice but to submit. He is not, however, to be despoiled of such a right. No less than the fundamental law guarantees just compensation. It would be injustice to him certainly if from such a period, he could not recover the value of what was lost. There could be on the other hand, injustice to the expropriator if by a delay in the collection, the increment in price would accrue to the owner. The doctrine to which this Court has been committed is intended precisely to avoid either contingency fraught with unfairness. Simply stated, the exception finds the application where the owner would be given undue incremental advantages arising from the use to which the government devotes the property expropriated -- as for instance, the extension of a main thoroughfare as was in the case in Caro de Araullo. In the instant case, however, it is difficult to conceive of how there could have been an extra-ordinary increase in the value of the owners land arising from the expropriation, as indeed the records do not show any evidence that the valuation of P1,000.00 reached in 1992 was due to increments directly caused by petitioners use of the land. Since the petitioner is claiming an exception to Rule 67, Section 4, it has the burden in proving its claim that its occupancy and use -- not ordinary inflation and increase in land values -- was the direct cause of the increase in valuation from 1978 to 1992. Side Issue: When is there Taking of Property? But there is yet another cogent reason why this petition should be denied and why the respondent Court should be sustained. An examination of the undisputed factual environment would show that the taking was not really made in 1978. This Court has defined the elements of taking as the main ingredient in the exercise of power of eminent domain, in the following words:

the claim for compensation, nakedly insisted that the property was public land and wrongly justified its possession by alleging it had already paid financial assistance to Marawi City in exchange for the rights over the property. Only in 1990, after more than a decade of beneficial use, did the petitioner recognize private respondents ownership and negotiate for the voluntary purchase of the property. A Deed of Sale with provisional payment and subject to negotiations for the correct price was then executed. Clearly, this is not the intent nor the expropriation contemplated by law. This is a simple attempt at a voluntary purchase and sale. Obviously, the petitioner neglected and/or refused to exercise the power of eminent domain. Only in 1992, after the private respondent sued to recover possession and petitioner filed its Complaint to expropriate, did petitioner manifest its intention to exercise the power of eminent domain. Thus the respondent Court correctly held: If We decree that the fair market value of the land be determined as of 1978, then We would be sanctioning a deceptive scheme whereby NAPOCOR, for any reason other than for eminent domain would occupy anothers property and when later pressed for payment, first negotiate for a low price and then conveniently expropriate the property when the land owner refuses to accept its offer claiming that the taking of the property for the purpose of the eminent domain should be reckoned as of the date when it started to occupy the property and that the value of the property should be computed as of the date of the taking despite the increase in the meantime in the value of the property. In Noble vs. City of Manila, the City entered into a lease-purchase agreement of a building constructed by the petitioners predecessor-in-interest in accordance with the specifications of the former. The Court held that being bound by the said contract, the City could not expropriate the building. Expropriation could be resorted to only when it is made necessary by the opposition of the owner to the sale or by the lack of any agreement as to the price. Said the Court: The contract, therefore, in so far as it refers to the purchase of the building, as we have interpreted it, is in force, not having been revoked by the parties or by judicial decision. This being the case, the city being bound to buy the building at an agreed price, under a valid and subsisting contract, and the plaintiff being agreeable to its sale, the expropriation thereof, as sought by the defendant, is baseless. Expropriation lies only when it is made necessary by the opposition of the owner to the sale or by the lack of any agreement as to the price. There being in the present case a valid and subsisting contract, between the owner of the building and the city, for the purchase thereof at an agreed price, there is no reason for the expropriation. (Italics supplied) In the instant case, petitioner effectively repudiated the deed of sale it entered into with the private respondent when it passed Resolution No. 92-121 on May 25, 1992 authorizing its president to negotiate, inter alia, that payment shall be effective only after Agus I HE project has been placed in operation. It was only then that petitioners intent to expropriate became manifest as private respondent disagreed and, barely a month, filed suit.25[25]

A number of circumstances must be present in taking of property for purposes of eminent domain: (1) the expropriator must enter a private property; (2) the entrance into private property must be for more than a momentary period; (3) the entry into the property should be under warrant or color of legal authority; (4) the property must be devoted to a public use or otherwise informally appropriated or injuriously affected; and (5) the utilization of the property for public use must be in such a way to oust the owner and deprive him of all beneficial enjoyment of the property.(Italics supplied) In this case, the petitioners entrance in 1978 was without intent to expropriate or was not made under warrant or color of legal authority, for it believed the property was public land covered by Proclamation No. 1354. When the private respondent raised his claim of ownership sometime in 1979, the petitioner flatly refused

25[25]

Supra, note 19 at 588-592; Emphasis supplied, italics in the

original.

Page 10 of 46

In the present case, to allow petitioner to use the date it constructed the tunnels as the date of valuation would be grossly unfair. First, it did not enter the land under warrant or color of legal authority or with intent to expropriate the same. In fact, it did not bother to notify the owners and wrongly assumed it had the right to dig those tunnels under their property. Secondly, the improvements introduced by petitioner, namely, the tunnels, in no way contributed to an increase in the value of the land. The trial court, therefore, as affirmed by the CA, rightly computed the valuation of the property as of 1992, when respondents discovered the construction of the huge underground tunnels beneath their lands and petitioner confirmed the same and started negotiations for their purchase but no agreement could be reached.
26[26]

has convinced us that, indeed, such general rule should in fact be observed in this case.27[27] Petitioner has not shown any error on the part of the CA in reaching such a valuation. Furthermore, these are factual matters that are not within the ambit of the present review. WHEREFORE, the petition is DENIED and the Decision of the Court of Appeals in C.A.-G.R. CV No. 57792 dated June 8, 2005 is AFFIRMED. No costs. SO ORDERED.

As to the amount of the valuation, the RTC and the CA both used as basis the value of the adjacent property, Lot 1 (the property involved herein being Lots 2 and 3 of the same subdivision plan), which was valued at P1,000 per sq. meter as of 1990, as sustained by this Court in Mangondato, thus: The Second Issue: Valuation We now come to the issue of valuation. The fair market value as held by the respondent Court, is the amount of P1,000.00 per square meter. In an expropriation case where the principal issue is the determination of just compensation, as is the case here, a trial before Commissioners is indispensable to allow the parties to present evidence on the issue of just compensation. Inasmuch as the determination of just compensation in eminent domain cases is a judicial function and factual findings of the Court of Appeals are conclusive on the parties and reviewable only when the case falls within the recognized exceptions, which is not the situation obtaining in this petition, we see no reason to disturb the factual findings as to valuation of the subject property. As can be gleaned from the records, the court-and-the-parties-appointed commissioners did not abuse their authority in evaluating the evidence submitted to them nor misappreciate the clear preponderance of evidence. The amount fixed and agreed to by the respondent appellate Court is not grossly exorbitant. To quote: Commissioner Ali comes from the Office of the Register of Deeds who may well be considered an expert, with a general knowledge of the appraisal of real estate and the prevailing prices of land in the vicinity of the land in question so that his opinion on the valuation of the property cannot be lightly brushed aside. The prevailing market value of the land is only one of the determinants used by the commissioners report the other being as herein shown: xxxxxx Commissioner Doromals report, recommending P300.00 per square meter, differs from the 2 commissioners only because his report was based on the valuation as of 1978 by the City Appraisal Committee as clarified by the latters chairman in response to NAPOCORs general counsels query. In sum, we agree with the Court of Appeals that petitioner has failed to show why it should be granted an exemption from the general rule in determining just compensation provided under Section 4 of Rule 67. On the contrary, private respondent

THE BACHRACH MOTOR CO., INC., plaintiff-appellee, vs. TALISAY-SILAY MILLING CO., ET AL., defendants-appellees. THE PHILIPPINE NATIONAL BANK, intervenor-appellant.

Roman J. Lacson for intervenor-appellant. Mariano Ezpeleta for plaintiff-appellee. Nolan and Hernaez for defendants-appellees Talisay-Silay Milling Co. and Cesar Ledesma.

ROMUALDEZ, J.:

This proceeding originated in a complaint filed by the Bachrach Motor Co., Inc., against the Talisay-Silay Milling Co., Inc., for the delivery of the amount P13,850 or promissory notes or other instruments or credit for that sum payable on June 30, 1930, as bonus in favor of Mariano Lacson Ledesma; the complaint further prays that the sugar central be ordered to render an accounting of the amounts it owes Mariano Lacson Ledesma by way of bonus, dividends, or otherwise, and to pay the plaintiff a sum sufficient to satisfy the judgment mentioned in the complaint, and that the sale made by said Mariano Lacson Ledesma be declared null and void.

The Philippine National Bank filed a third party claim alleging a preferential right to receive any amount which Mariano Lacson Ledesma might be entitled to from the Talisay-Silay Milling Co. as bonus, because that would be civil fruits of the land mortgaged to said bank by said debtor for the benefit of the central referred to, and by virtue of a deed of assignment, and praying that said central be ordered to delivered directly to the intervening bank said sum on account of the latter's credit against the aforesaid Mariano Lacson Ledesma.

The corporation Talisay-Silay Milling Co., Inc., answered the complaint stating that of Mariano Lacson Ledesma's credit, P7,500 belonged to Cesar Ledesma because he had purchased it, and praying that it be absolved from the complaint and that the proper party be named so that the remainder might be delivered.

Cesar Ledesma, in turn, claiming to be the owner by purchase in good faith an for a reconsideration of the P7,500 which is a part of the credit referred to above, answered praying that he be absolved from the complaint.

The plaintiff Bachrach Motor Co., Inc., answered the third party claim alleging that its credit against Mariano Lacson Ledesma was prior and preferential to that of the intervening bank, and praying that the latter's complaint be dismissed.

26[26]

See RTC decision of August 7, 1996, Rollo, p. 158.

27[27]

See, supra note 19 at 592-593.

Page 11 of 46

At the trial all the parties agreed to recognize and respect the sale made in favor of Cesar Ledesma of the P7,500 part of the credit in question, for which reason the trial court dismissed the complaint and cross-complaint against Cesar Ledesma authorizing the defendant central to deliver to him the aforementioned sum of P7,500. And upon conclusion of the hearing, the court held that the Bachrach Motor Co., Inc., had a preferred right to receive the amount of P11,076.02 which was Mariano Lacson Ledesma's bonus, and it ordered the defendant central to deliver said sum to the plaintiff.

The Philippine National Bank appeals, assigning the following alleged errors as committed by the trial court:

This is how the bonus came to be granted: On December 22, 1923, the Talisay-Silay Milling Co., Inc., was indebted to the Philippine National Bank. To secure the payment of its debt, it succeeded in inducing its planters, among whom was Mariano Lacson Ledesma, to mortgage their land to the creditor bank. And in order to compensate those planters for the risk they were running with their property under the mortgage, the aforesaid central, by a resolution passed on that same date, i.e., December 22, 1923, undertook to credit the owners of the plantation thus mortgaged every year with a sum equal to two per centum of the debt secured according to yearly balance, the payment of the bonus being made at once, or in part from time to time, as soon as the central became free of its obligations to the aforesaid bank, and of those contracted by virtue of the contract of supervision, and had funds which might be so used, or as soon as it obtained from said bank authority to make such payment. (Exhibits 5, 6; P.N.B.)

1. In holding that the bonus which the Talisay-Silay Milling Co., Inc., bound itself to pay the planters who had mortgaged their land to the Philippine National Bank to secure the payment of the debt of said central to said bank is not civil fruits of said land.

2. In not holding that said bonus became subject to the mortgage executed by the defendant Mariano Lacson Ledesma to the Philippine National Bank to secure the payment of his personal debt to said bank when it fell due.

Article 355 of the Civil Code considers three things as civil fruits: First, the rents of buildings; second, the proceeds from leases of lands; and, third, the income from perpetual or life annuities, or other similar sources of revenue. It may be noted that according to the context of the law, the phrase "u otras analogas" refers only to rent or income, for the adjectives "otras" and "analogas" agree with the noun "rentas," as do also the other adjectives"perpetuas" and "vitalicias." That is why we say that by "civil fruits" the Civil Code understands one of three and only three things, to wit: the rent of a building, the rent of land, and certain kinds of income.

3. In holding that the assignment (Exhibit 9, P.N.B.) of said bonus made on March 7, 1930, by Mariano Lacson Ledesma to the Philippine National Bank to be applied to the payment of his debt to said Philippine National Bank is fraudulent.

As the bonus in question is not rent of a building or of land, the only meaning of "civil fruits" left to be examined is that of "income."

4. In holding that the Bachrach Motor Co. Inc., in civil case No. 31597 of the Court of First Instance of Manila levied a valid attachment upon the bonus in question.

Assuming that in broad juridical sense of the word "income" it might be said that the bonus in question is "income" under article 355 of the Civil Code, it is obvious to inquire whether it is derived from the land mortgaged by Mariano Lacson Ledesma to the appellant bank for the benefit of the central; for it is not obtained from that land but from something else, it is not civil fruits of that land, and the bank's contention is untenable.

5. In admitting and considering the supplementary complaint filed by the Bachrach Motor Co., Inc., alleging as a cause of action the attachment of the bonus in question which said Bachrach Motor Co., Inc., in civil case No. 31821 of the Court of First Instance of Manila levied after the filing of the original complaint in this case, and after Mariano Lacson Ledesma in this case had been declared in default.

6. In holding that the Bachrach Motor Co., Inc., has a preferential right to receive from the TalisaySilay Milling Co., Inc., the amount of P11,076.02 which is in the possession of said corporation as the bonus to be paid to Mariano Lacson Ledesma, and in ordering the Talisay-Silay Milling Co., Inc., to deliver said amount to the Bachrach Motor Co., Inc.

It is to be noted that the said bonus bears no immediate, but only a remote accidental relation to the land mentioned, having been granted as compensation for the risk of having subjected one's land to a lien in favor of the bank, for the benefit of the entity granting said bonus. If this bonus be income or civil fruits of anything, it is income arising from said risk, or, if one chooses, from Mariano Lacson Ledesma's generosity in facing the danger for the protection of the central, but certainly it is not civil fruits or income from the mortgaged property, which, as far as this case is concerned, has nothing to do with it. Hence, the amount of the bonus, according to the resolution of the central granting it, is not based upon the value, importance or any other circumstance of the mortgaged property, but upon the total value of the debt thereby secured, according to the annual balance, which is something quite distinct from and independent of the property referred to.

7. In not holding that the Philippine National Bank has a preferential right to receive from the TalisaySilay Milling Co., Inc., the amount of P11,076.02 held by said corporation as Mariano Lacson Ledesma's bonus, and in not ordering said Talisay-Silay Milling Co., Inc., to deliver said amount to the Philippine National Bank.

Finding no merit in this appeal, the judgment appealed from is affirmed, without express finding as to costs. So ordered.

8. In not holding that the amended complaint and the supplementary complaint of the Bachrach Motor Co., Inc., do not state facts sufficient to constitute a cause of action in favor of the Bachrach Motor Co., Inc., and against the Talisay-Silay Milling Co., Inc., or against the Philippine National Bank.

PACIFIC FARMS, INC., plaintiff-appellee, vs. SIMPLICIO G. ESGUERRA, ET AL., defendants, CARRIED LUMBER COMPANY, defendant-appellant.

Primicias, Del Castillo, Macaraeg and T. P. Regino for defendant-appellant. Araneta and Araneta for plaintiff-appellee. The appellant bank bases its preferential right upon the contention that the bonus in question is civil fruits of the lands which the owners had mortgaged for the benefit of the central giving the bonus, and that, as civil fruits of said land, said bonus was assigned by Mariano Lacson Ledesma on March 7, 1930, by virtue of the document Exhibit 9 of said intervening institution, which admitted in its brief that "if the bonus in question is not civil fruits or rent which became subject to the mortgage in favor of the Philippine National Bank when Mariano Lacson Ledesma's personal obligation fell due, the assignment of March 7, 1930 (Exhibit 9, P.N.B.), is null and void, not because it is fraudulent, for there was no intent of fraud in executing the deed, but that the cause or consideration of the assignment was erroneous, for it was based upon the proposition that the bonus was civil fruits of the land mortgaged to the Philippine National Bank." (P. 31.)

CASTRO, J.:

Before us for review, on appeal by the defendant Carried Lumber Company (hereinafter referred to as the Company), is the decision, dated May 30, 1962, of the Court of First Instance of Pangasinan in civil case D-1317, annulling the levy and certificate of sale covering six buildings owned by the plaintiff Pacific Farms, Inc., executed by the defendant deputy provincial sheriff Simplicio G. Esguerra in favor of the Company to satisfy a money judgment against the Insular Farms, Inc., the plaintiff's predecessor-in-interest over the said buildings.

The fundamental question, then, submitted to our consideration is whether or not the bonus in question is civil fruits.

The environmental setting is uncontroverted.

Page 12 of 46

On several occasions from October 1, 1956 to March 2, 1957 the Company sold and delivered lumber and construction materials to the Insular Farms, Inc. which the latter used in the construction of the aforementioned six buildings at its compound in Bolinao, Pangasinan, of the total procurement price of P15,000, the sum of P4,710.18 has not been paid by Insular Farms, Inc. Consequently, on October 17, 1958 the Company instituted civil case D775 with the Court of First Instance of Pangasinan to recover the said unpaid balance from the Insular Farms, Inc. On August 23, 1961 the trial court rendered judgment sustaining the Company's claim. The judgment debtor did not appeal; so on December 19, 1961 the corresponding writ of execution was issued. On January 16, 1962 the defendant sheriff levied upon the six buildings. On January 30, 1962 the Pacific Farms, Inc. filed a third-party claim, subscribed by its corporate president, asserting ownership over the levied buildings which it had acquired from the Insular Farms, Inc. by virtue of a deed of absolute sale executed on March 21, 1958, about seven months before the Company filed the above-mentioned action (civil case D-775). Shielded by an indemnity bond of P7,120 put up by the Company and the Cosmopolitan Insurance Company, Inc., the sheriff proceeded with the announced public auction on February 12, 1962 and sold the levied buildings to the Company for P6,110.78.

Likewise unchallenged is the lower court's factual finding that out of the total procurement price of P15,000, the amount of P4,710.18 remains outstanding and unpaid by the Insular Farms, Inc. The appellant is therefore an unpaid furnisher of materials.

Whether there exists a materialman's lien over the six buildings in favor of the appellant, is a question we do not here decide. To our mind the application by analogy of the rules of accession would suffice for a just adjudication. Article 447 of the Civil Code1 provides:

Asserting absolute and exclusive ownership of the buildings in question, the Pacific Farms, Inc. filed a complaint on May 14, 1962 against the Company and the sheriff with the court a quo, praying that judgment be rendered, (a) declaring null and void the levy and judicial sale of the six buildings, and (b) adjudging the defendants jointly and severally liable to the plaintiff in the sum of P2,000 by way of actual damages and for such amount as the court may deem proper and just to impose by way of exemplary damages and for costs of the suit.

The owner of the land who makes thereon personally or through another, plantings, constructions or works with the materials of another, shall pay their value; and, if he acted in bad faith, he shall also be obliged to the reparation of damages. The owner of the materials shall have the right to remove them only in case he can do so without injury to the work constructed, or without the plantings, constructions or works being destroyed. However, if the landowner acted in bad faith, the owner of the materials may remove them in any event with a right to be indemnified for damages.

After due trial, the court a quo on May 30, 1963 rendered judgment annulling the levy of January 16, 1962 and the certificate of sale of February 12, 1962. The court, however, denied the plaintiff's claim for actual and exemplary damages on the ground that it was not "prepared to find that there was gross negligence or bad faith on the part of any of the defendants."

The abovequoted legal provision contemplates a principal and an accessory, the land being considered the principal, and the plantings, constructions or works, the accessory. The owner of the land who in good faith whether personally or through another makes constructions or works thereon, using materials belonging to somebody else, becomes the owner of the said materials with the obligation however of praying for their value. 2The owner of the materials, on the other hand, is entitled to remove them, provided no substantial injury is caused to the landowner. Otherwise, he has the right to reimbursement for the value of his materials.

Hence this appeal, imputing errors which, according to the appellant's formulation, are the following:

1. The lower court erred in holding that the credit of the defendant-appellant, Carried Lumber Company, against the Insular Farms, Inc., consisting of the value of lumber and construction materials used in the buildings which were later acquired by the Pacific Farms, Inc., the appellee, was not a statutory lien on those buildings; .

Although it does not appear from the records of this case that the land upon which the six buildings were built is owned by the appellee, nevertheless, that the appellee claims that it owns the six buildings constructed out of the lumber and construction materials furnished by the appellant, is indubitable. Therefore, applying article 447 by analogy, we perforce consider the buildings as the principal and the lumber and construction materials that went into their construction as the accessory. Thus the appellee, if it does own the six buildings, must bear the obligation to pay for the value of the said materials; the appellant which apparently has no desire to remove the materials, and, even if it were minded to do so, cannot remove them without necessarily damaging the buildings has the corresponding right to recover the value of the unpaid lumber and construction materials.

2. The lower court, likewise, erred in holding that the doctrine laid down in De Barretto, et al. vs. Villanueva, et al. (G.R. No. L-14938, December 29, 1962) is applicable to the facts of this case as found by said court; and .

Well-established in jurisprudence is the rule that compensation should be borne by the person who has been benefited by the accession.3 No doubt, the appellee benefited from the accession, i.e., from the lumber and materials that went into the construction of the six buildings. It should therefore shoulder the compensation due to the appellant as unpaid furnisher of materials.

3. The lower court erred, finally, in declaring that the sale at public auction conducted by the defendant deputy provincial sheriff of Pangasinan, covering the six buildings described in the certificate of sale dated February 12, 1962, was null and void.

Of course, the character of a buyer in good faith and for value, if really possessed by the appellee, could possibly exonerate it from making compensation.

1. In ruling against the appellant below, the trial court relied mainly on the resolution (on the motion for reconsideration) promulgated on December 29, 1962 by this Court in De Barretto, et al. vs. Villanueva, et al., L14938 (6 SCRA 928). The said case, however, is inapplicable because it concerned not one but two or more preferred creditors who, pursuant to articles 2242 and 2249 of the Civil Code, must necessarily be convened and the nature and extent of their respective claims ascertained. Thus, we held that before there can be a pro rata payment of credits entitled to preference as to the same specific real property, there must first be some proceeding where the claims of all the preferred creditors may be bindingly adjudicated, such as insolvency, the settlement of a decedent's estate under Rule 87 of the Rules of Court, or liquidation proceedings of similar import.

But the appellee's stance that it is an innocent purchaser for value and in good faith is open to grave doubt because of certain facts of substantial import (evident from the records) that cannot escape notice.

In the deed of absolute sale, exhibit 1, the Insular Farms, Inc. (vendor) was represented in the contract by its president, J. Antonio Araneta. The latter was a director of the appellee (Pacific Farms, Inc.) and was the counsel who signed the complaint filed by the appellee in the court below. J. Antonio Araneta was, therefore, not only the president of the Insular Farms, Inc. but also a director and counsel of the appellee.

But the case before us does not involve a question of preference of credits, and is not one where two or more creditors have separate and distinct claims against the same debtor who has insufficient property. Indeed, it is a matter of necessity and logic that the question of preference should arise only where the debtor cannot pay his debts in full. For, if debtor A is able in full to pay all his three creditors, B, C, and D, how can the need arise for determining which of the three creditors shall be paid first or whether they shall be paid out of the proceeds of a specific property?

2. It is undenied and undeniable that the appellant furnished lumber and construction materials to the Insular Farms, Inc. (the appellee's predecessor-in-interest) which the latter used in the construction of the six buildings.

During the trial of civil case D-775 the Insular Farms, Inc. was represented by Attorney Amado Santiago, Jr. of the law firm of J. Antonio Araneta. The latter was one of the counsels of the Pacific Farms, Inc. The appellee cannot claim ignorance of the pendency of civil case D-775 because the Insular Farms, Inc. was defended by the same lawyer from the same law firm that commenced the present action. J. Antonio Araneta, as counsel for the Pacific Farms, Inc., cannot close his eyes to facts of which he as president of the Insular Farms, Inc. had actual knowledge. Significantly, exhibit 1 (supra) itself shows that the Insular Farms, Inc. and the Pacific Farms, Inc. were housed in adjacent rooms (nos. 304 and 303, respectively), of the same building, the Insular Life Building, as early as March 21, 1958.

Page 13 of 46

It is reasonable therefore to conclude that the appellee, through its director and counsel, J. Antonio Araneta, knew about the unpaid balance of the purchase price of the lumber and construction materials supplied or furnished by the appellant to the Insular Farms, Inc.

Parenthetically, it is likewise worth our attention that despite the appellee's knowledge of the suit instituted by the appellant against the Insular Farms, Inc. (the appellee's predecessor-in-interest) for the recovery of the unpaid balance of the purchase price of the lumber and materials used in the construction of its six buildings, it merely folded its arms in disinterest and waited, so to speak. Not until a decision was rendered therein in favor of the appellant, a writ of execution issued, and the six buildings levied upon by the sheriff, did it file a third-party claim over the levied buildings. In the face of the knowledge that its predecessor-in-interest had not fully paid for the lumber and construction materials used in the six buildings it had purchased, its natural and expected reaction should have been to intervene in the suit filed by the appellant against the Insular Farms, Inc. and hold the latter to account for breach of the warranties deemed included in the deed of absolute sale conveying said building to it.

Balayunan, Silang Cavite. To secure possession of the land from the vendors the said plaintiff, on July 20, 1929, instituted Civil Case No. 1935 in the Court of First Instance of Cavite. The trial court found for the plaintiff in a decision which was affirmed by this Supreme Court on appeal (G. R. No. 33017). 1 When plaintiff entered upon the premises, however, he found the defendant herein, Catalino Bataclan, who appears to have been authorized by former owners, as far back as 1922, to clear the land and make improvements thereon, As Bataclan, who appears to have been authorized by former owners, as far bank as 1922, to clear the land and make improvements thereon. As Bataclan was not a party in Case No. 1935, plaintiff, on June 11, 1931, instituted against him, in the Court of First Instance of Cavite, Civil Case No. 2428. In this case, plaintiff was declared owner but the defendant was held to be possessor in good faith, entitled to reimbursement in the total sum of P1,642, for work done and improvements made. The dispositive part of the decision reads: "Por las consideraciones expuestas, se declara al demandante Vicente Santo Domingo Bernardo dueo con derecho a la posesion del terreno que se describe en la demanda, y al demandado Catalino Bataclan con derecho a que el demandante le pague la suma de P1,642 por gastos utiles hechos de buena fe en el terreno, y por el cerco y ponos de coco y abaca existentes en el mismo, y con derecho, ademas a retener la posesion del terreno hasta que se le pague dicha cantidad. Al demandante puede optar, en el plazo de treinta dias, a partir de la fecha en que fuere notificado de la presente, por pagar esa suma al demandado, haciendo asi suyos el cerco y todas las plantaciones existentes en el terreno, a razon de trescientos pesos la hectarea. En el caso de que el demandante optara por que el demandado le pagara el precio del terreno, el demandado efectuara el pago en el plazo conveniente por las partes o que sera fijado por el Juzgado. Sin costas." Both parties appealed to this court (G. R. No. 37319). 1 The decision appealed from was modified by allowing the defendant to recover compensation amount to P2,212 and by reducing the price at which the plaintiff could require the defendant to purchase the land in question from P300 to P200 per hectare. Plaintiff was given by this court 30 days from the date when the decision became final within which to exercise his option, either to sell the land to the defendant or to buy the improvements from him. On January 9, 1934, the plaintiff manifested to the lower court his desire "to require the defendant to pay him the value of the land at the rate of P200 per hectare or a total price of P18,000 for the whole tract of land." The defendant informed the lower court that he was unable to pay for the land and, on January 24, 1934, an order was issued giving the plaintiff 30 days within which to pay the defendant the sum of P2,212 stating that, in the event of failure to make such payment, the land would be ordered sold at public auction "Para hacer pago al demandante de la suma de P2,212 y el remanente despus de deducidos los gastos legales de la venta en publica subasta sera entregado al demandante." On February 21, 1934, plaintiff moved to reconsider the foregoing order so that he would have preference over the defendant in the order of payment. The motion was denied on March 1, 1934 but on March 16 following the court below, muto proprio, modified its order of January 24, "en el sentido de que el demandante tiene derecho preferente al importe del terreno no se vendiere en publica subasta, a razon de P200 por hecatarea y el remanente, si acaso o hubiere se entregara el demandado en pago de la cantidad de P2,212 por la limpieza del terreno y las mejoras introducidas en el mismo por el citado demandado." On April 24, 1934, the court below, at the instance of the plaintiff and without objection on the part of the defendant, ordered the sale of the land in question at public auction. The land was sold on April 5, 1935 to Toribio Teodoro, the highest bidder, for P8,000. In the certificate of sale issued to said purchaser on the very day of sale, it was stated that the period of redemption of the land sold was to expire on April 5, 1936. Upon petition of Toribio Teodoro the court below ordered the provincial sheriff to issue another certificate not qualified by any equity of redemption. This was complied with by the sheriff on July 30, 1935. On September 18, 1935, Teodoro moved that he be placed in possession of the land purchased by him. The motion was granted by order of September 26, 1935, the dispositive part of which is as follows:

Curiously enough, although the six buildings in question were supposedly sold by the Insular Farms to the appellee on March 21, 1958, as evidenced by the deed of absolute sale (exhibit 1), about seven months before the appellant filed civil case D-775, the Insular Farms, Inc. never moved to implead the appellee therein as a necessary partydefendant, and remained completely and strangely silent about the sale. It is not amiss to surmise that it is entirely possible that the Insular Farms, Inc. and the appellee chose to remain silent in the hope that the appellant's claim against the Insular Farms, Inc. in civil case D-775 would be dismissed or non-suited.

Moreover, the appellee was in a better position to protect its interest. It knew that the Insular Farms, Inc., its predecessor-in-interest, was a mere lessee of the premises on which the buildings were located. This should have placed it on guard and compelled it to ascertain the circumstances surrounding the construction of the said buildings on the premises.

On the other hand, the appellant was not as advantageously situated as the appellee. There being no separate registry of property for buildings and no procedure provided by law for registering or annotating the claim of an unpaid furnisher of materials, it was helpless to prevent the sale of the property built from lumber and construction materials it furnished. But certainly, because it has a right, pursuant to article 447, supra, to reimbursement for the value of its unpaid materials, the appellant could pursue any remedy available to it under the law in order to enforce the said right. Thus, the appellant acted correctly in bringing an action (D-775) against the Insular Farms, Inc. and enforcing its right of reimbursement through the execution of the final judgment it obtained in the said case against the six buildings in the possession of the appellee who now stands to benefit therefrom. It follows, as a necessary corollary, that the sale at public auction conducted by the defendant sheriff of the six buildings described in the certificate of sale dated February 12, 1962, exhibit 7, was valid and effective.

ACCORDINGLY, the judgment a quo is reversed, and the complaint is hereby dismissed.

In view, however, of the equities clearly attendant in this case, it is the sense of this Court that the plaintiff-appellee Pacific Farms, Inc. should be, as it is hereby, granted a period of thirty (30) days from the date this judgment becomes final, within which it may exercise the option of redeeming the six buildings, by paying to the defendantappellant Carried Lumber Company the sum of P4,710.18, with legal interest from September 23, 1961 (the date the judgment in civil case D-775 became final), until the said amount shall have been fully paid.

No pronouncement as to costs.

[1938V174E] VICENTE STO. DOMINGO BERNARDO, plaintiff-appellant, vs. CATALINO BATACLAN, defendantappellant. TORIBIO TEODORO, purchaser-appellee.1938 Nov 28En BancG.R. No. 44606D E C I S I O N LAUREL, J: This is an appeal taken by both the plaintiff and the defendant from the order of September 26, 1935, hereinbelow referred to, of the Court of First Instance of Cavite in Civil Case No. 2428. There is no controversy as to the facts. By a contract of sale executed on July 17, 1920, the plaintiff herein acquired from Pastor Samonte and others ownership of the parcel of land of about 90 hectares situated in sitio

"Por tanto, se ordena al Shriff Provincial de Cavite ponga a Toribio Teodoro en posesion del terreno comprado por el en subasta publica y por el cual se le expidio certificado de vente definitiva, reservando al demandado su derecho de ejercitar una accion ordinaria para reclamar del demandante la cantidad de P2,212 a que tiene derecho por la limpieza y mejoras del terreno y cuya suma, en justicia y equidad, debe ser descontada y deducida de la suma de P8,000 que ya ha recibido el demandante." The Civil Code confirms certain time-honored principles of the law of property. One of these is the principle of accession whereby the owner of property acquires not only that which it produces but that which is united to it either naturally or artificially. (Art. 353.) Whatever is built, planted or sown on the land of another, and the

Page 14 of 46

improvements or repairs made thereon, belong to the owner of the land (art. 358). Where, however, the planter, builder, or sower has acted in good faith, a conflict of rights arises between the owners and it becomes necessary to protect the owner of the improvements without causing injustice to the owner of the land. In view of the impracticability of creating what Manresa calls a state of "forced coownership" (vol. 3, 4th ed., p. 213), the law has provided a just and equitable solution by giving the owner of the land the option to acquire the improvements after payment of the proper indemnity or to oblige the builder or planter to pay for the land and the sower to pay the proper rent (art. 361). It is the owner of the land who is allowed to exercise the option because his right is older and because, by the principle of accession, he is entitled to the ownership of the accessory thing (3 Manresa, 4th ed., p. 213). In the case before us, the plaintiff, as owner of the land, chose to require the defendant, as owner of the improvements, to pay for the land. The defendant states that he is a possessor in good faith and that the amount of P2,212 to which he is entitled has not yet been paid to him. Therefore, he says, he has a right to retain the land in accordance with the provisions of article 453 of the Civil Code. We do not doubt the validity of the premises stated. "Considera la ley tan sagrada y legitima la deuda, que, hasta que sea pagada, no consiente que la cosa se restituya al vencedor." (4 Manresa, 4th ed., p., 304.) We find, however, that the defendant has lost his right to retention. In obedience to the decision of his right to retention. In obedience to the decision of this court in G. R. No. 37319, the plaintiff expressed his desire to require the defendant to pay for the value of the land. The said defendant could have become owner of both land and improvements and continued in possession thereof. But he said he could not pay and the land was sold at public auction to Toribio Teodoro. The law, as we have already said, requires no more than that the owner of the land should choose between indemnifying the owner of the improvements or requiring the latter to pay for the land. When he failed to pay for the land, the defendant herein lost his right of retention.

(1) Filipinas Colleges, Inc. was declared to have acquired the rights of the spouses Timbang in and to lot No. 2-a mentioned above and in consideration thereof, Filipinas Colleges, Inc. was ordered to pay the spouses Timbang the amount of P15,807.90 plus such other amounts which said spouses might have paid or had to pay after February, 1953, to Hoskins & Co., Inc., agent of the Urban Estates, Inc., original vendor of the lot. Filipinas Colleges, Inc. was required to deposit the total amount with the court within 90 days after the decision shall have become final. (2) Maria Gervacio Blas was declared to be a builder in good faith of the school building constructed on the lot in question and entitled to be paid the amount of P19,000.00 for the same. Filipinas Colleges, Inc., purchaser of the said building was ordered to deliver to Blas stock certificate (Exh. C) for 108 shares of Filipinas Colleges, Inc. with a par value of P10,800.00 and to pay Blas the sum of P8,200.00 representing the unpaid balance of the purchase price of the house. (3) In case Filipinas Colleges, Inc. failed to deposit the value of the land, which after liquidation was fixed at P32,859.34, within the 90-day period set by the court, Filipinas Colleges would lose all its rights to the land and the spouses Timbang would then become the owners thereof. In that eventuality, the Timbangs would make known to the court their option under Art. 448 of the Civil Code whether they would appropriate the building in question, in which even they would have to pay Filipinas Colleges, Inc. the sum of P19,000.00, or would compel the latter to acquire the land and pay the price thereof. Filipinas Colleges, Inc. having failed to pay or deposit the sum of P32,859.34 within the time prescribed, the spouses Timbang, in compliance with the judgment of the Court of Appeals, on September 28, 1956, made known to the court their decision that they had chosen not to appropriate the building but to compel Filipinas Colleges, Inc., to acquire the land and pay them to value thereof. Consequently, on December 29, 1956, the Timbang spouses asked for an order of execution against Filipinas Colleges, Inc. for the payment of the sum of P32,859.34. The motion having been granted, a writ of execution was issued on January 8, 1957. On January 16, 1957, appellee Blas in turn filed a motion for execution of her judgment of P8,200.00 representing the unpaid portion of the price of the house sold to Filipinas Colleges, Inc. Over the objection of the Timbangs, the court granted the motion and the corresponding writ of execution was issued on January 30, 1957. Even before the actual issuance of this writ, or on January 19, 1957, date of the granting of the motion for execution, Blas through counsel, sent a letter to the Sheriff of Manila advising him of her preferential claim or lien on the house to satisfy the unpaid balance of the purchase price thereof under Article 2242 of the Civil Code, and to withhold from the proceed of the auction sale the sum of P8,200.00. Levy having been made on the house in virtue of the writs of execution, the Sheriff of Manila on March 5, 1957, sold the building in public auction in favor of the spouses Timbang, as the highest bidders, in the amount of P5,750.00. Personal properties of Filipinas Colleges, Inc. were also auctioned for P245.00 in favor of the spouses Timbang. As a result of these actuations, three motions were subsequently filed before the lower court:

The sale at public auction having been asked by the plaintiff himself (p. 22, bill of exceptions) and the purchase price of P8,000 received by him from Toribio Teodoro, we find no reason to justify a rupture of the situation has created between them, the defendant- appellant not being entitled, after all, to recover from the plaintiff the sum of P2,212. The judgment of the lower court is accordingly modified by eliminating therefrom the reservation made in favor of the defendant- appellant to recover from the plaintiff the sum of P2,212. In all other respects, the same is affirmed, without pronouncement regarding costs. So ordered. [1959V272E] FILIPINAS COLLEGES, INC., plaintiff-appellee, vs. MARIA GARCIA TIMBANG, ET AL., defendants.1959 Sep 29En BancG.R. No. L-12812D E C I S I O N BARRERA, J.: This is an appeal taken from an order of the Court of First Instance of Manila dated May 10, 1957 (a) declaring the Sheriff's certificate of sale covering a school building sold at public auction null and void unless within 15 days from notice of said order the successful bidders, defendants-appellants spouses Mara Garca Timbang and Marcelino Timbang, shall pay to appellee Mara Gervacio Blas directly or through the Sheriff of Manila the sum of P5,750.00 that the spouses Timbang had bid for the building at the Sheriff's sale; (b) declaring the other appellee Filipinas Colleges, Inc. owner of 24,500/3,285,934 undivided interest in Lot No. 2-a covered by certificate of title No. 45970, on which the building sold in the auction sale is situated; and (c) ordering the sale in public auction of the said undivided interest of the Filipinas Colleges, Inc. in lot No. 2-a aforementioned to satisfy the unpaid portion of the judgment in favor of appellee Blas and against Filipinas Colleges, Inc. in the amount of P8,200.00 minus the sum of P5,750.00 mentioned in (a) above. The order appealed from is the result of three motions filed in the court a quo in the course of the execution of a final judgment of the Court of Appeals rendered in 2 cases appealed to it in which the spouses Timbang, the Filipinas Colleges, Inc. and Maria Gervacio Blas were the parties. In that judgment of the Court of Appeals, the respective rights of the litigants have been adjudicated as follows:

(1) By appellee Blas, praying that the Sheriff of Manila and/or the Timbang spouses be ordered to pay and deliver to her the sum of P5,750.00 representing the proceeds of the auction sale of the building of Filipinas Colleges, Inc. over which she has a lien of P8,200.00 for the unpaid balance of the purchase price thereof; (2) Also by the appellee Blas, praying that there being still two unsatisfied executions, one for the sum of P32,859.34 in favor of the Timbang spouses, and another, for the sum of P8,200.00 in her favor, the land involved, Lot No. 2-a, be sold at public auction; and (3) By Filipinas Colleges, Inc., praying that because its properties, the house and some personal properties, have been auctioned for P5,750.00 and P245.00 respectively in favor of the Timbang spouses who applied the proceeds to the partial payment of the sum of P32,859.34, value of the land, Lot No. 2-a, it (Filipinas Colleges, Inc.) be declared part owner of said lot to the extent of the total amount realized from the execution sales of its properties.

Page 15 of 46

The Timbang spouses presented their opposition to each and all of these motions. After due hearing the lower court rendered its resolution in the manner indicated at the beginning of this decision, from which the Timbangs alone have appealed. In assailing the order of the Court a quo directing the appellants to pay appellee Blas the amount of their bid (P5,750.00) made at the public auction, appellants' counsel has presented a novel, albeit ingenious, argument. It is contended that because the builder in good faith has failed to pay the price of the land after the owners thereof exercised their option under Article 448 of the Civil Code, the builder lost his right of retention provided in Article 546 and by operation of Article 445, the appellants as owners of the land automatically became the owners of the building. And since they are the owners ipso facto, the execution sale of the house in their favor was superflous. Consequently, they are not bound to make good their bid of P5,750.00 as that would be to compel them to pay for their own property. By the same token, Blas' claim for preference on account of the unpaid balance of the purchase price of the house does not apply because preference applies only with respect to the property of the debtor, and the Timbangs, owners of the house, are not the debtors of Blas. This Court cannot accept this oversimplification of appellants' position. Articles 448 and 546 of the Civil Code, defining the rights of the parties in case a person in good faith builds, sows or plants on the land of another, respectively provides: ART. 448. The owner of the land on which anything has been built, sown or planted in good faith, shall have the right to appropriate as his own the works, sowing or planting, after payment of the indemnity provided for in articles 546 and 548, or to oblige the one who built or planted to pay the price of the land, and the one who sowed, the proper rent. However, the builder or planted cannot be obliged to buy the land if its value id considerably more than that of the building or trees. In such case, he shall pay reasonable rent, if the owner of the land does not choose to appropriate the building or trees after proper indemnity. The parties shall agree upon the terms of the lease and in case of disagreement, the court shall fix the terms thereof. ART. 546. Necessary expenses shall be refunded to every possessor; but only the possessor in good faith may retain the thing until he has been reimbursed therefor. Useful expenses shall be refunded only to the possessor in good faith with the same right of retention the person who has defeated him in the possession having the option of refunding the amount of the expenses or of payi ng the increase in value which the thing may have acquired by reason thereof. Under the terms of these articles, it is that the owner of the land has the right to choose between appropriating the building by reimbursing the builder of the value thereof or compelling the builder in good faith to pay for his land. Even this second right cannot be exercised if the value of the land is considerably more than that of the building. In addition to the right of the builder to be paid the value of his improvement, Article 546 gives him the corollary right of retention of the property until he is indemnified by the owner of the land. There is nothing in the language of these two articles, 448 and 546, which would justify the conclusion of appellants that, upon the failure of the builder to pay the value of the land, when such is demanded by the land-owner, the latter becomes automatically the owner of the improvement under Article 445. The case of Bernardo vs. Bataclan, 66 Phil., 590 cited by appellants is no authority for this conclusion. Although it is true it was declared therein that in the event of the failure of the builder to pay the land, after the owner thereof has chosen this alternative, the builder's right of retention provided in Article 546 is lost, nevertheless there was nothing said that as a consequence thereof, the builder loses entirely all rights over his own building. The question is: what is the recourse or remedy left to the parties in such eventuality where the builder fails to pay the value of the land? While the Code is silent on this point, guidance may be derived from the decisions of this Court in the cases of Miranda vs. Fadullon, et al., 97 Phil., 801; 51 Off. Gaz., [12] 6226; Ignacio vs. Hilario, 76 Phil., 605 and the cited case of Bernardo vs. Bataclan, supra. In the first case, this Court has said: "A builder in good faith may not be required to pay rentals. He has a right to retain the land on which he has built in good faith until he is reimbursed the expenses incurred by him. Possibly he might be made to pay rental only when the owner of the land chooses not to appropriate the improvement and requires the builder in good faith to pay for the land but that the builder is unwilling or unable to pay the land, and then they decide to leave things as they are

and assume the relation of lessor and lessee, and should they disagree as to the amount of rental then they can go to the court to fix that amount". mphasis supplied). Should the parties not agree to leave things as they are and to assume the relation of lessor and lessee, another remedy is suggested in the case of Ignacio vs. Hilario, supra, wherein the court has ruled that the owner of the land is entitled to have the improvement removed when after having chosen to sell his land to the other party, i.e., the builder in good faith fails to pay for the same. A further remedy is indicated in the case of Bernardo vs. Bataclan, supra, where this Court approved the sale of the land and the improvement in a public auction applying the proceeds thereof first to the payment of the value of the land and the excess, if any, to be delivered to the owner of the house in payment thereof. The appellants herein, owners of the land, instead of electing any of the alternatives above indicated chose to seek recovery of the value of their land by asking for a writ of execution; levying on the house of the builder; and selling the same in public auction. And because they are the highest bidder in their own auction sale, they now claim they acquired title to the building without necessity of paying in cash on account of their bid. In other words, they in effect pretend to retain their land and acquire the house without paying a cent therefor. This contention is without merit. This Court has already held in Matias vs. The Provincial Sheriff of Nueva Ecija (74 Phil., 326) that while it is the invariable practice, dictated by common sense, that where the successful bidder is the execution creditor himself, he need not pay down the amount of the bid if it does not exceed the amount of his judgment, nevertheless, when there is a claim by a third-party, to the proceeds of the sale superior to his judgment credit, the execution creditor, as successful bidder, must pay in cash the amount of his bid as a condition precedent to the issuance to him of the certificate of sale. In the instant case, the Court of Appeals has already adjudged that appellee Blas is entitled to the payment of the unpaid balance of the purchase price of the school building. Blas' claim is therefore not a mere preferred credit, but is actually a lien on the school building as specifically provided in Article 2242 of the new Civil Code. As such, it is superior to the claim of the Timbangs insofar as the proceeds of the sale of said school building are concerned. The order of the lower court directing the Timbang spouses, as successful bidders, to pay in cash the amount of their bid in the sum of P5,750.00 is therefore correct. With respect to the order of the court declaring appellee Filipinas Colleges, Inc. part owner of the land to the extent of the value of its personal properties sold at public auction in favor of the Timbangs, this Court likewise finds the same as justified, for such amount represents, in effect, a partial payment of the value of the land. If this resulted in the continuation of the so-called involuntary partnership questioned by the appellants, it was due to their own action. As appellee Blas still has an unsatisfied judgment representing the difference between P8,200.00 - the unpaid balance of the purchase price of the building and the sum of P5,750.00 - amount to be paid by the Timbangs, the order of the court directing the sale of such undivided interest of the Filipinas Colleges, Inc. is likewise justified to satisfy the claim of the appellee Blas. Considering that the appellant spouses Marcelino Timbang and Mara Garca Timbang may not voluntarily pay the sum of P5,750.00 as ordered, thereby further delaying the final termination of this case, the first part of the dispositive portion of the order appealed from is modified in the sense that upon failure of the Timbang spouses to pay to the sheriff or to Mara Gervacio Blas said sum of P5,750.00 within fifteen (15) days from notice of the final judgment, an order of execution shall issue in favor of Mara Gervacio Blas to be levied upon all properties of the Timbang spouses not exempt from execution for the satisfaction of the said amount. In all other respects, the appealed order of the court a quo is hereby affirmed, with costs against the appellants. It is so ordered.

RODOLFO V. ROSALES, (represented by his heirs, Rodolfo, Jr., Romeo Allan, Lillian Rhodora, Roy Victor, Roger Lyle and Alexander Nicolai, all surnamed Rosales) and LILY ROSQUETA-ROSALES,Petitioners vs. MIGUEL CASTELLTORT, JUDITH CASTELLTORT, and LINA LOPEZ-VILLEGAS, assisted by her Attorney-inFact, Rene Villegas, Respondents.

Page 16 of 46

DECISION

the spouses named as defendants (tsn, p. 8, January 12, 1998) and which declaration is an utter falsehood as the Contract to Sell itself indicates the civil status of said Elizabeth Yson Cruz to be single.

CARPIO MORALES, J.: The present petition for review on certiorari assails the October 2, 2002 Decision 1 and February 6, 2003 Resolution2 of the Court of Appeals (CA) in CA G.R. CV No. 64046 and seeks to reinstate the April 21, 1999 Decision3 of the Regional Trial Court (RTC) of Calamba, Laguna, Branch 34 in Civil Case No. 2229-95-C. Even if we are to concede that defendants built their house in good faith on account of the representation of attorney-in-fact Rene Villegas, their failure to comply with the requirements of the National Building Code, particularly the procurement of a building permit, stained such good faith and belief.

xxx Spouses-petitioners Rodolfo V. Rosales and Lily Rosqueta-Rosales (petitioners) are the registered owners of a parcel of land with an area of approximately 315 square meters, covered by Transfer Certificate of Title (TCT) No. 368564 and designated as Lot 17, Block 1 of Subdivision Plan LRC Psd-55244 situated in Los Baos, Laguna.

From any and all indications, this deliberate breach is an unmitigated manifestation of bad faith. And from the evidence thus adduced, we hold that defendants and the intervenor were equally guilty of negligence which led to the construction of the defendants house on plaintiffs property and therefore jointly and severally liable for all the damages suffered by the plaintiffs.16 (Underscoring supplied) The dispositive portion of the trial courts Decision reads, quoted verbatim:

On August 16, 1995, petitioners discovered that a house was being constructed on their lot, without their knowledge and consent, by respondent Miguel Castelltort (Castelltort). 5

It turned out that respondents Castelltort and his wife Judith had purchased a lot, Lot 16 of the same Subdivision Plan, from respondent Lina Lopez-Villegas (Lina) through her son-attorney-in-fact Rene Villegas (Villegas) but that after a survey thereof by geodetic engineer Augusto Rivera, he pointed to Lot 17 as the Lot 16 the Castelltorts purchased. Negotiations for the settlement of the case thus began, with Villegas offering a larger lot near petitioners lot in the same subdivision as a replacement thereof.6 In the alternative, Villegas proposed to pay the purchase price of petitioners lot with legal interest.7 Both proposals were, however, rejected by petitioners8 whose counsel, by letter9 of August 24, 1995, directed Castelltort to stop the construction of and demolish his house and any other structure he may have built thereon, and desist from entering the lot. Petitioners subsequently filed on September 1, 1995 a complaint 10 for recovery of possession and damages with prayer for the issuance of a restraining order and preliminary injunction against spouses-respondents Miguel and Judith Castelltort before the RTC of Calamba, Laguna, docketed as Civil Case No. 2229-95-C.
11

ACCORDINGLY, in view of all the foregoing, judgment is hereby rendered in favor of plaintiffs and against the defendants, ordering the latter to surrender the possession of the property covered by TCT No. 36856 of the Register of Deeds of Laguna including any and all improvements built thereon to the plaintiffs.

Defendants and intervenors are likewise jointly and severally directed to pay to plaintiffs the following damages:

a) TWO THOUSAND (P2,000.00) PESOS per month from February 1995 by way of reasonable compensation for the use of plaintiffs property until the surrender of the same;

b) FIFTY THOUSAND (P50,000.00) PESOS by way of moral damages;

c) THIRTY THOUSAND (P30,000.00) PESOS as exemplary damages; To the complaint, the Castelltorts claimed in their Answer with Counterclaim that they were builders in good faith. d) TWENTY THOUSAND (P20,000.00) PESOS as attorneys fees and cost of suit. Lina, represented by her son-attorney-in-fact Villegas, soon filed a Motion for Intervention 12 before the RTC which was granted by Order13 of December 19, 1995. In her Answer to the complaint,14 Lina alleged that the Castelltorts acted in good faith in constructing the house on petitioners lot as they in fact consulted her before commencing any construction thereon, they having relied on the technical description of the lot sold to them, Lot 16, which was verified by her officially designated geodetic engineer.

The counterclaim interposed by the defendants in their responsive pleading is hereby dismissed for lack of merit. SO ORDERED.17

Respondents thereupon filed their respective appeals with the CA.

Nevertheless, Lina proposed to give petitioners a lot containing an area of 536 square meters together with the house and duplex structure built thereon or, if petitioners choose, to encumber the 536 square meter lot as collateral "to get immediate cash" through a financing scheme in order to compensate them for the lot in question. 15

Petitioner Rodolfo Rosales, in the meantime, died on December 7, 2001. His heirs Rodolfo, Jr., Romeo Allan, Lillian Rhodora, Roy Victor, Roger Lyle and Alexander Nicolai, all surnamed Rosales, filed their Appearance 18 as his substitute.

Ruling out good faith, the RTC, by Decision of April 21, 1999, found for petitioners in this wise:

By Decision of October 2, 2002, the CA granted the appeal and set aside the April 21, 1999 RTC Decision. The dispositive portion of the Decision reads, quoted verbatim:

In the instant case, there is no well-founded belief of ownership by the defendants of the land upon which they built their house. The title or mode of acquisition upon which they based their belief of such ownership stemmed from a Contract to Sell (Exhibit "P") of which they were not even parties, the designated buyer being Elizabeth Yson Cruz and the sale even subjected to the judicial reconstitution of the title. And by their own actions, particularly defendant Miguel Castelltort, defendants betrayed this very belief in their ownership when realizing the inutility of anchoring their ownership on the basis of the Contract of Sale, defendant Miguel Castelltort in his testimony declared Elizabeth Yson Cruz as his wife (tsn, pp. 7-8, March 24, 1998) despite an admission in their answer that they are

WHEREFORE, premises considered, the instant appeal is hereby GRANTED and the assailed decision of the court a quo REVERSED AND SET ASIDE. In accordance with the cases of Technogas Philippines Manufacturing Corp. vs. Court of Appeals and Depra vs. Dumlao, applying Article 448 of the Civil Code, this case is REMANDEDto the Regional Trial Court of Calamba, Laguna, Branch 34, for further proceedings, as follows:

Page 17 of 46

1. to determine the present fair price of appellees 315 square meter area of land and the amount of the expenses actually spent by the appellants for building the house as of 21 August 1995, which is the time they were notified of appellees rightful claim over Lot 17.

under the mistaken belief that the same is his property. Otherwise, he should have secured a building permit on Lot 17 instead or should not have bothered to take the necessary measures to obtain a building permit on Lot 16 in the first place.

2. to order the appellees to exercise their option under the law (Article 448, Civil Code), whether to appropriate the house as their own by paying to the appellants the amount of the expenses spent for the house as determined by the court a quo in accordance with the limitations as aforestated or to oblige the appellants to pay the price of the land.

By and large, the records show that, as testified to by Engr. Rebecca T. Lanuang, appellant Miguel had already applied for a building permit as early as February 1994 and was in fact issued a temporary building permit pending the completion of the requirements for said permit. Although the building permit was belatedly issued in January 1996, this does not in any way detract from appellant Miguels good faith.

In case the appellees exercise the option to oblige the appellants to pay the price of the land but the latter reject such purchase because, as found by the court, the value of the land is considerably more than that of the house, the court shall order the parties to agree upon the terms of a forced lease, and give the court a quo a formal written notice of such agreement and its provisos. If no agreement is reached by the parties, the court a quo shall then fix the terms of the forced lease, provided that the monthly rental to be fixed by the Court shall not be less that Two Thousand Pesos (P2,000.00) per month, payable within the first five (5) days of each calendar month and the period thereof shall not be more than two (2) years, counted from the finality of the judgment.

xxx

Upon the expiration of the forced lease, or upon default by the appellants in the payment of rentals for two (2) consecutive months, the appellees shall be entitled to terminate the forced lease, to recover their land, and to have the improvement removed by the appellants at the latters expense. The rentals herein provided shall be tendered by the appellants to the court for payment to the appellees, and such tender shall constitute evidence of whether or not compliance was made within the period fixed by the court.

In holding the appellants as builders in bad faith, the court a quo defied law and settled jurisprudence considering that the factual basis of its findings and the incontrovertible evidence in support thereof prove that the appellant Miguel, in good faith, built the house on appellees land without knowledge of an adverse claim or any other irregularities that might cast a doubt as to the veracity of the assurance given to him by the intervenor. Having been assured by the intervenor that the stone monuments were purposely placed, albeit wrongfully, by the land surveyor in said land to specifically identify the lot and its inclusive boundaries, the appellants cannot be faulted for having relied on the expertise of the land surveyor who is more equipped and experienced in the field of land surveying. Although under the Torrens system of land registration, the appellant is presumed to have knowledge of the metes and bounds of the property with which he is dealing, appellant however, considering that he is a layman not versed in the technical description of his property, cannot be faulted in his reliance on the survey plan that was delivered to him by the intervenor and the stone monuments that were placed in the encroached property.

In any event, the appellants shall pay the appellees the amount of Two Thousand Pesos (P2,000.00) as reasonable compensation for their occupancy of the encroached property from the time said appellants good faith cease (sic) to exist until such time the possession of the property is delivered to the appellees subject to the reimbursement of the aforesaid expenses in favor of the appellants or until such time the payment of the purchase price of the said lot be made by the appellants in favor of the appellees in case the latter opt for the compulsory sale of the same.
19

xxx

SO ORDERED.

(Emphasis in the original)

Peremptorily, contrary to the flawed pronouncements made by the court a quo that appellant Miguel is deemed as a builder in bad faith on the basis of a mere assertion that he built his house without initially satisfying himself that he owns the said property, this Court finds reason to maintain good faith on the part of the appellant. Admittedly, the appellants house erroneously encroached on the property of the appellees due to a mistake in the placement of stone monuments as indicated in the survey plan, which error is directly attributable to the fault of the geodetic engineer who conducted the same. This fact alone negates bad faith on the part of appellant Miguel.

In reversing the trial court, the CA held:

xxx

xxx

Moreover, it is quite illogical for appellant Miguel to knowingly build his house on a property which he knew belongs to another person. x x x

x x x A perusal of the records readily reveals that said court instead relied on flimsy, if not immaterial, allegations of the appellees, which have no direct bearing in the determination of whether the appellants are builders in bad faith.

xxx

For one, the pivotal issue to be resolved in this case, i.e. whether appellant Miguel is a builder in good faith, was ignored by the court a quo. The instant case does not in any way concern the personal and property relations of spouses-appellants and Elizabeth Yson Cruz which is an altogether different matter that can be ventilated by the concerned parties through the institution of a proper action. xxx The court a quo should have focused on the issue of whether appellant Miguel built, in good faith, the subject house without notice of the adverse claim of the appellees and under the honest belief that the lot which he used in the construction belongs to him. xxx

In view of the good faith of both parties in this case, their rights and obligations are to be governed byArticle 448, which has been applied to improvements or portions of improvements built by mistaken belief on land belonging to the adjoining owner. x x x x x x20 (Emphasis and underscoring supplied) Petitioners Motion for Reconsideration21 dated October 22, 2002 having been denied by the CA by Resolution of March 13, 2002, the present petition was filed raising the following issues:

xxx As it is, appellant Miguel relied on the title which the intervenor showed to him which, significantly, has no annotation that would otherwise show a prior adverse claim. Thus, as far as appellant Miguel is concerned, his title over the subject lot, as well as the title of the intervenor thereto, is clean and untainted by an adverse claim or other irregularities. For another, the appellants failure to secure a building permit from the Municipal Engineers Office on their construction on Lot 17 does not impinge on the good faith of the appellants. In fact, it can be told that a building permit was actually filed by appellant Miguel with respect to Lot 16 and it was only due to the confusion and misapprehension by the intervenor of the exact parameters of the property which caused appellants belief that Lot 17 [the questioned lot], is his. This fact bolsters appellant Miguels good faith in building his house on appellees lot

I.

WHETHER OR NOT THE HONORABLE COURT OF APPEALS COMMITTED A GRAVE ABUSE OF DISCRETION IN MAKING A FINDING THAT IS CONTRARY TO THE ADMISSIONS BY THE PARTIES

Page 18 of 46

II.

WHETHER OR NOT THE HONORABLE COURT OF APPEALS COMMITTED A REVERSIBLE ERROR OF LAW IN CONCLUDING THAT THE TRIAL COURT, IN DECIDING THE CASE, RELIED ON FLIMSY, IF NOT IMMATERIAL, ALLEGATIONS OF THE PETITIONERS, WHICH HAVE NO DIRECT BEARING IN THE DETERMINATION OF WHETHER THE RESPONDENTS ARE BUILDERS IN GOOD FAITH

In the case at bar, Lot 16 was sold by Lina, through her attorney-in-fact Villegas, to Castelltort and a certain Elizabeth Cruz29 for a consideration of P500,000.00. While prior to the sale, what Villegas showed Castelltort as evidence of his mother Linas ownership of the property was only a photocopy of her title TCT No. (T-42171) T1855030 he explaining that the owners duplicate of the title was lost and that judicial reconstitution thereof was ongoing, Castelltort acted in the manner of a prudent man and went to the Registry of Deeds of Laguna to procure a certified true copy of the TCT.31 The certified true copy bore no annotation indicating any prior adverse claim on Lot 16. The records indicate that at the time Castelltort began constructing his house on petitioners lot, he believed that it was the Lot 16 he bought and delivered to him by Villegas.

III.

WHETHER OR NOT THE HONORABLE COURT OF APPEALS COMMITTED A REVERSIBLE ERROR OF LAW IN RENDERING A DECISION THAT IS UNENFORCEABLE AGAINST BOTH RESPONDENT JUDITH CASTELLTORT AND THIRD-PARTY ELIZABETH CRUZ22 Petitioners initially hammer against respondents proving that Castelltort and a certain Elizabeth Cruz are the builders of the house on the subject property, they faulting them with estoppel for alleging in their Answer before the trial court that "they (respondents Castelltort and Judith) caused the construction of their house which they bought from a certain Lina Lopez-Villegas." Petitioners rely on the following doctrine established in Elayda v. Court of Appeals:23

In his cross-examination, Villegas testified:

Q: You said the surveyor placed a mujon along boundary of the property?

A: Yes.

Q: When were the mujons placed in the boundary of the property?

A: These mujons were the basis for my locating the property in pointing to Mr. Castelltort. "an admission made in the pleadings cannot be controverted by the party making such admission and are conclusive as to him and that all proofs submitted by him contrary thereto or inconsistent therewith, should be ignored, whether objection is interposed by the party or not x x x" Petitioners contention is hardly relevant to the case at bar. Whether it was Castelltort and Judith or Castelltort and Elizabeth Cruz who purchased the property from Lina is not material to the outcome of the instant controversy. As found by the CA:

xxx

Q: Is it not a fact that before Miguel Castelltort started constructing that house he sought your advice or permission to construct the same over that particular lot?

A: Yes. The fact remains that appellant [Castelltort] is the builder of the house on Lot 17 xxx The court a quo should have focused on the issue of whether appellant Miguel built, in good faith, the subject house without notice of the adverse claim of the appellees and under the honest belief that the lot which he used in the construction belongs to him. xxx it cannot be gainsaid that appellant Miguel has a title over the land that was purchased from the intervenor x x x24
25

Q: And you gave your consent?

A: Yes, because based on my knowledge also that that was the lot as pointed by Engr. Rivera. At all events, as this Court held in the case of Gardner v. Court of Appeals:

xxx In its Resolution reversing the original Decision, respondent Court discredited the testimony of Ariosto SANTOS for being at variance with the allegations in his Answer. The fact, however, that the allegations made by Ariosto SANTOS in his pleadings and in his declarations in open Court differed will not militate against the findings herein made nor support the reversal by respondent Court. As a general rule, facts alleged in a partys pleading are deemed admissions of that party and binding upon it, but this is not an absolute and inflexible rule. An Answer is a mere statement of fact which the party filing it expects to prove, but it is not evidence. As Ariosto SANTOS himself, in open Court, had repudiated the defenses he had raised in his Answer and against his own interest, his testimony is deserving of weight and credence.26 (Underscoring supplied)

Q: Was there any remarkable difference between lot 16 and 17 at the time that this particular lot was sold to Miguel Castelltort and Elizabeth Cruz?

xxx

The issue determinative of the controversy in the case at bar hinges on whether Castelltort is a builder in good faith.

A: Both lots 16 and 17 are practically the same. The (sic) have the same frontage. There is only a difference of 4 square meters, one is 311 square meters and the other 315 square meters. Both sides were fenced, as drawn they were facing the same road. They are practically the same.

A builder in good faith is one who builds with the belief that the land he is building on is his, or that by some title one has the right to build thereon, and is ignorant of any defect or flaw in his title. 27

Q: But at the time or immediately before Mr. Castelltort started the construction of the house, was there any remarkable distinction between these two properties? A: None.32 (Emphasis and underscoring supplied)

Article 527 of the Civil Code provides that good faith is always presumed, and upon him who alleges bad faith on the part of a possessor rests the burden of proof. 28

Page 19 of 46

The confusion in the identification of Lot 16 was eventually traced to the error committed by geodetic engineer Augusto Riveras employees in placing stone monuments on petitioners property, instead of on Lot 16, the lot sold to Castelltort, based on the survey made by the engineer in 1992.

A: I actually reprimanded them verbally and also I dismissed Mario Carpio from my office.

xxx The engineer so testified: Q: And did you investigate how your men committed this mistake of planting these monuments on another lot when corners 4 & 1 were clearly planted on the ground? Q: Now, aside from inspecting personally the site, what else did your men or assistants do? A: I myself rechecked it and found out that they committed an error. A: After computing the subdivision lots, they went back to the field to plant those subdivision corners with concrete monuments. xxx Q: Which is (sic) also called as "mohons"? Q: And now, you are saying that your men committed a mistake by placing thereon monuments by planting these monuments not on Lot 16 but on Lot 17? A: Yes, sir. A: When I investigated how did they commit (sic) a mistake it came to be like this. Before when we surveyed first this in 1992, at that time Dante Villegas contracted my services there was a fence here then when we went back, the road was already removed so they committed an error that this point is Lot 19, they thought that it was Lot 19, the back portion.

Q: Now, can you point to this Honorable Court where exactly did your men place these additional mohons and how many?

A: Later on we discovered that they placed the mohons in the adjoining lot, lot 17. xxx xxx Q: In this particular case, did you find out how your men checked the succeeding lots, how they determine (sic) the exact location of lot 16? Q: x x x when again did you meet Mr. Rene Villegas or after how many months or year? A: They just relied on one side of the subdivision. A: Maybe after a year, sir. Q: By just counting the number of lots? Q: And you met him again because he had a problem regarding the property of one Engr. Rosales? A: Yes, sir. A: Yes, sir. Q: Without making any actual measurement? Q: And when he confided to you this matter, did you go to the site of Lot 16 or 17? A: They made an actual measurement but the reference point is not the one, the correct one because they also checked it with the other corner of the road going back.

A: Yes, sir.

Q: And what did you see there?

xxx

A: A house being constructed then I rechecked the location of the house and it turned out to be in Lot 17.

Q: And how did they commit a mistake when you said they checked the lot at the back of Lot 16?

xxx

A: Because they were quite confident since we had already relocated the property two years ago so they thought that they get (sic) the right lot without checking the other side of the subdivision.

Q: Considering that you found out that a mistake was actually made by your assistants Dennis Orencio, Mario Carpio and Sovejano when you allowed them to proceed on their own to make this computation, did you confront these men of yours afterwards?

xxx

A: Yes, sir.

Q: Now, you said that when you went to the place because you heard from Rene Villegas that there was a mistake you no longer could find the monuments on lines 1 and 4 and according to you the reason is that a fence was already constructed?

Q: In what manner?

Page 20 of 46

A: Yes, sir.

In the case at bar, Castelltorts good faith ceased on August 21, 1995 when petitioners personally apprised him of their title over the questioned lot. As held by the CA, should petitioners then opt to appropriate the house, they should only be made to pay for that part of

Q: For clarification, is this line 1 & 4 on Lot 16 a common line 1 &4 on Lot 17? the improvement built by Castelltort on the questioned property at the time good faith still existed on his part or until August 21, 1995.

A: Yes, sir a common line.

Q: In other words, this line 1 &4 devides (sic) Lot 16 & 17?

The CA, however, failed to qualify that said part of the improvement should be pegged at its current fair market value consistent with this Courts pronouncement in Pecson v. Court of Appeals.40 And, as correctly found by the CA, the commencement of Castelltorts payment of reasonable rent should start on August 21, 1995 as well, to be paid until such time that the possession of the property is delivered to petitioners, subject to the reimbursement of expenses, that is, if such option is for petitioners to appropriate the house. This Court quotes the CAs ratiocination with approval:

A: Yes, sir.

Q: So that when these monuments were placed on lines 1 & 4 somebody could mistake it for Lot 17 also because there were monuments now 1 &4 for lot 16 since these are common lines for

Lot 17 also with Lot 16, it could also be construed that these are monuments for Lot 17? A: Yes, sir possible.33 (Underscoring supplied) x x x Generally, Article 448 of the Civil Code provides that the payment of reasonable rent should be made only up to the date appellees serve notice of their option as provided by law upon the appellants and the court a quo; that is, if such option is for appellees to appropriate the encroaching structure. In such event, appellants would have a right to retain the land on which they have built in good faith until they are reimbursed the expenses incurred by them. This is so because the right to retain the improvements while the corresponding indemnity is not paid implies the tenancy or possession in fact of the land on which it is built, planted or sown.

As correctly found by the CA, both parties having acted in good faith at least until August 21, 1995, the applicable provision in this case is Article 448 of the Civil Code which reads:

Art. 448. The owner of the land on which anything has been built, sown or planted in good faith, shall have the right to appropriate as his own the works, sowing or planting, after payment of the indemnity provided for in Articles 546 and 548, or to oblige the one who built or planted to pay the price of the land, and the one who sowed, the proper rent. However, the builder or planter cannot be obliged to buy the land if its value is considerably more than that of the building or trees. In such case, he shall pay reasonable rent, if the owner of the land does not choose to appropriate the building or trees after proper indemnity. The parties shall agree upon the terms of the lease and in case of disagreement, the court shall fix the terms thereof.

However, considering that appellants had ceased as builders in good faith at the time that appellant Miguel was notified of appellees lawful title over the disputed property, the payment of reasonable rent should accordingly commence at that time since he can no longer avail of the rights provided under the law for builders in good faith. 41

If the option chosen by petitioners is compulsory sale, however, the payment of rent should continue up to the 42 actual transfer of ownership. Respecting petitioners argument that the appellate court erred in rendering a decision that is "unenforceable against Judith who is not the owner of the house and Elizabeth Cruz who was found to be a part owner of the house built on their lot but is not a party to the case," the same does not lie. While one who is not a party to a proceeding shall not be affected or bound 43 by a judgment rendered therein,44like Elizabeth Cruz, this does not detract from the validity and enforceability of the judgment on petitioners and respondents Castelltorts.

Under the foregoing provision, the landowner can choose between appropriating the building by paying the proper indemnity or obliging the builder to pay the price of the land, unless its value is considerably more than that of the structures, in which case the builder in good faith shall pay reasonable rent.34 If the parties cannot come to terms over the conditions of the lease, the court must fix the terms thereof.

The choice belongs to the owner of the land, a rule that accords with the principle of accession, i.e., that the accessory follows the principal and not the other way around. Even as the option lies with the landowner, the grant to him, nevertheless, is preclusive.35 The landowner cannot refuse to exercise either option and compel instead the owner of the building to remove it from the land.36 The raison detre for this provision has been enunciated thus:

WHEREFORE, the petition is DENIED. The Decision dated October 2, 2002 and Resolution dated February 6, 2003 of the Court of Appeals are AFFIRMED with MODIFICATION such that the trial court shall include for determination the increase in value ("plus value") which petitioners 315 square meter lot may have acquired by reason of the existence of that portion of the house built before respondents Miguel and Judith Castelltort were notified of petitioners rightful claim on said lot, and the current fair market value of said portion.

Where the builder, planter or sower has acted in good faith, a conflict of rights arises between the owners, and it becomes necessary to protect the owner of the improvements without causing injustice to the owner of the land. In view of the impracticability of creating a state of forced co-ownership, the law has provided a just solution by giving the owner of the land the option to acquire the improvements after payment of the proper indemnity, or to oblige the builder or planter to pay for the land and the sower the proper rent. He cannot refuse to exercise either option. It is the owner of the land who is authorized to exercise the option, because his right is older, and because, by the principle of accession, he is entitled to the ownership of the accessory thing. 37

SO ORDERED.

Possession acquired in good faith does not lose this character except in the case and from the moment facts exist which show that the possessor is not unaware that he possesses the thing improperly or wrongfully. 38 The good faith ceases or is legally interrupted from the moment defects in the title are made known to the possessor, by extraneous evidence or by suit for recovery of the property by the true owner. 39

[1955V223E] SALVACION MIRANDA, plaintiff-appellants, vs. ESTEBAN FADULLON and spouses DIONISIO SEGARRA and CLEMENCIA N. DE SEGARRA, defendants-appellees.1955 Oct 29En BancG.R. No. L-8220D E C ISION MONTEMAYOR, J.:

Page 21 of 46

The present appeal was first taken to the Court of Appeals. Later by resolution of the said court it was certified to us under section 17, paragraph 6 of the Judiciary Act of 1948, as amended, the said Tribunal being of the opinion that the case involved only questions of law. The facts as may be gathered from the pleadings filed by the parties may be briefly stated as follows. In the year 1939 one Lucio Tio was the owner of a parcel of land, lot 1589-J of the Banilad Estate, Cebu, under Transfer Certificate of Title No. 10548. On December 9, 1939, a power of attorney in favor of one Esteban Fadullon executed by Lucio Tio was registered in the land records of Cebu City and annotated at the back of the transfer certificate of title. On the same date the deed of mortgage in favor of the Cebu Mutual Building and Loan Association was also annotated on the same certificate of title. In the year 1946, on the strength of the said power of attorney Fadullon sold the property to the spouses Dionisio Segarra and Clemencia N. de Segarra with right to repurchase within the short period of 30 days. Upon failure of Fadullon to make the repurchase within this period, the Segarras about ten days after the expiration of the period filed a sworn petition for the consolidation of their ownership and registered said petition in the office of the Register of Deeds on May 15, 1946. Apprised of the sale of his property, Lucio Tio on June 4, 1946, filed a complaint in the Court of First Instance of Cebu, Civil Case No. 181 to annul the sale. Service of summons was made upon the Segarras on June 10, 1946. After hearing the trial court rendered judgment annulling the sale. The Segarras appealed to the Court of Appeals under CA-G. R. No. 6550-R and the said Tribunal affirmed the appealed decision and further required the Segarras to pay plaintiff the reasonable rentals on the property from the filing of the action until said property shall have been returned to plaintiff. Upon the decision becoming final the corresponding writ of execution was issued directing the Sheriff to put plaintiff Tio in possession of the lot. It turned out however that during the possession of the property by the Segarras they had introduced improvements thereon consisting of a building of three rooms and a storage room, and one artesian well, with tower and water tank and a cement flooring covering about onethird of the lot which according to the Segarras cost them P5,300. They then filed a motion with the trial court claiming that they were possessors in good faith of the lot in question, and that they had introduced the improvements aforementioned in good faith and asked the court to order the plaintiff to pay for the said improvements valued at P5,300 or to allow them to buy the land should the plaintiff decide not to pay for the improvements. On August 28, 195, the trial court issued the following order:

"While the evidence did not disclose a collusion or conspiracy between Fadullon and the Segarras, yet, considering the short period of one month within which to redeem and the surrounding circumstances, the possibility of such collusion lingers. "Obviously there was in this transaction a prevailing intention of railroading the property into a new ownership as may be proven by the fact that said purchasers filed a sworn petition for consolidating their ownership barely ten days after the expiration of thirty days, that is, on April 13, 1946, and registered with the office of Register of Deeds for Cebu twelve days thereafter, or on May 15, 1946." The Court of Appeals in its decision affirming that of the trial court said:

"The Segarra spouses maintain that they are purchasers in good faith. We will now examine the record on this point. The alleged power of attorney executed by the late Lucio Tio in favor of appellant Fadullon was registered in the land record of the Register of Deeds of Cebu City and annotated at the back of Transfer Certificate of Title No. 10548 on December 29, 1939. On the same date, the deed of mortgage in favor of the Cebu Mutual Building and Loan Association was annotated in the said Torrens title (Exhibits 1 and 1-B). This encumbrance alone should have been sufficient to put the Segarra spouses upon an inquiry as to the authority of Fadullon to sell to them the same property six years later. For instance, the Segerras could have asked themselves this question: Did not the mortgage of P400 serve the purpose for which the power of attorney was executed? "The Segarras did not require Fadullon to produce his power of attorney. While it is true that said power of attorney is annotated at the back of the Torrens title of Tio, it was still incumbent upon the Segarras to ascertain the scope and authority of Fadullon under said power of attorney. Fadullon executed the sale with the right to repurchase within the extraordinary short period of 30 days. This circumstance, again, should have placed the Segarras on their guards, knowing, as they did, that they were dealing with an agent under a power of attorney executed before the war. These unusual circumstances would seem to engender in our minds the possibility of collusion between the appellants, to hasten the registration of the title of the Segarras to the land in dispute . . . ". . . the transfer of dominion on the property in question to the Segarras was null and void and of no effect. The new Certificate of Torrens Title No. 392 on the property now in the name of the Segarras is hereby ordered cancelled and that a new one issued in the name of Lucio Tio and his wife Salvacion Miranda; ordering the Segarras to return the possession of said property to plaintiff; "The defendants Segarras are furthermore required to pay plaintiff the reasonable rentals on the property from the filing of this action until such time as the said property shall have been returned to plaintiff . . ." Although neither the trial court nor the Court of Appeals did expressly say and in so many words that the defendants-appellees were possessors in bad faith, from a reading of their decisions particularly those we have just quoted, one can logically infer that was the conclusion of the two courts, or to say it more mildly, that the defendants were not possessors in good faith. Moreover, the very fact that the Court of Appeals sentenced the defendants to pay rentals is an indication, even proof that defendants were considered possessors and builders in bad faith, or at least that they were not possessors and builders in good faith. A builder in good faith may not be required to pay rentals. He has a right to retain the land on which he has built in good faith until he is reimbursed the expenses incurred by him. Possibly he might be required to pay rental only when the owner of the land chooses not to appropriate the improvement and requires the builder in good faith to pay for the land, but that the builder is unwilling or unable to buy the land, and then they decide to leave things as they are and assume the relation of lessor and lessee, and should they disagree as to the amount of the rental then they can go to the court to fix that amount. Furthermore, plaintiff-appellant in her brief (page 7) says without denial or refutation on the part of defendants-appellees that they (defendants) applied for a building permit to construct the improvements in question on December 4, 1946 and the permit was granted on January 11, 1947, all this about seven months after they received the summons on June 10, 1946, meaning to say that the improvements were introduced long after their alleged good faith as possessors had ended.

"The attorney for the plaintiff has been accordingly served with copy of defendant's motion of July 31, 1952, filed through counsel. "As prayed for, without opposition, the plaintiff is hereby ordered to either pay the defendant spouses, Dionisio Segarra and Clemencia N. Segarra (possessors in good faith) the sum of P5,300, value of the building erected on the land in question, or otherwise allow said defendants to purchase the aforementioned lot." The plaintiff filed a motion for reconsideration claiming that the Segarras were possessors and builders in bad faith and so were not entitled to reimbursement for the value of the improvements; that the reason he (plaintiff) did not file an opposition to the motion of the defendants asking for reimbursement was that he thought that the trial court was sufficiently informed and impressed with the bad faith with which defendants bought the land and introduced improvements thereon and that it would consequently deny their motion; and in support of his motion for reconsideration plaintiff quoted portions of the decision of the trial court and the Court of Appeals. Upon the denial of his motion for reconsideration, he took the present appeal. After a careful review of the record we agree with the plaintiff- appellant. The trial court in its decision declaring the sale of the land to the defendants null and void and commenting on the alleged good faith of defendants in buying the property said the following: "There are two circumstances which seem to stubbornly belie the professed good faith on the part of the Segarras in buying this property; namely the circumstances of the power-of-attorney appearing on the back of the title as of five or six years previous and the other circumstances of the comparatively limited period of one month granted vendor Fadullon to redeem the property. Above all these, is the further circumstance that the said property had already been mortgaged in favor of the Cebu Mutual Building and Loan Association by virtue of that power-ofattorney.

Page 22 of 46

In view of the foregoing, the appealed order of August 28, 1952 and the order of October 15, 1952, denying plaintiff's motion for reconsideration are set aside. With costs against appellees. [1946V18E] DAMIAN IGNACIO, FRANCISCO IGNACIO and LUIS IGNACIO, petitioners, vs. ELIAS HILARIO and his wife DIONISIA DRES, and FELIPE NATIVIDAD, Judge of First Instance of Pangasinan, respondents.1946 Apr 30En BancG.R. No. L-175D E C I S I O N MORAN, C.J: This is a petition for certiorari arising from a case in the Court of First Instance of Pangasinan between the herein respondents Elias Hilario and his wife Dionisia Dres as plaintiffs, and the herein petitioners Damian, Francisco and Luis surnamed Ignacio, as defendants, concerning the ownership of a parcel of land, partly rice-land and partly residential. After the trial of the case, the lower court, presided over by Hon. Alfonso Felix, rendered judgment holding plaintiffs as the legal owners of the whole property but conceding to defendants the ownership of the houses and granaries built by them on the residential portion with the rights of a possessor in good faith, in accordance with article 361 of the Civil Code. The dispositive part of the decision , hub of this controversy, follows: "Wherefore, judgment is hereby rendered declaring: "(1) That the plaintiffs are the owners of the whole property described in transfer certificate of title No. 12872 (Exhibit A) issued in their name, and entitled to the possession of the same; "(2) That the defendants are entitled to hold the possession of the residential lot until after they are paid the actual market value of their houses and granaries erected thereon, unless the plaintiffs prefer to sell them said residential lot, in which case defendants shall pay the plaintiffs the proportionate value of said residential lot taking as a basis the price paid for the whole land according to Exhibit B; and

"ART. 453. Necessary expenses shall be refunded to every possessor; but only the possessor in good faith may retain the thing until such expenses are made good to him. "Useful expenses shall be refunded to the possessor in good faith with the same right of retention, the person who has defeated him in the possession having the option of refunding the amount of the expenses or paying the increase in value which the thing may have acquired in consequence thereof."

The owner of the building erected in good faith on a land owned by another, is entitled to retain the possession of the land until he is paid the value of his building, under article 453. The owner of the land, upon the other hand, has the option, under article 361, either to pay for the building or to sell his land to the owner of the building. But he cannot, as respondents here did, refuse both to pay for the building and to sell the land and compel the owner of the building to remove it from the land where it is erected. He is entitled to such remotion only when, after having chosen to sell his land, the other party fails to pay for the same. But this is not the case before us. We hold, therefore, that the order of Judge Natividad compelling defendants-petitioners to remove their buildings from the land belonging to plaintiffs-respondents only because the latter chose neither to pay for such buildings nor to sell the land, is null and void, for it amends substantially the judgment sought to be executed and is, furthermore, offensive to articles 361 and 453 of the Civil Code. There is. however, in the decision of Judge Felix a question of procedure which calls for clarification, to avoid uncertainty and delay in the disposition of cases. In that decision, the rights of both parties are well defined under articles 361 and 453 of the Civil Code, but it fails to determine the value of the buildings and of the lot where they are erected as well as the periods of time within which the option may be exercised and payment should be made, these particulars having been left for determination apparently after the judgment has become final. This procedure is erroneous, for after the judgment has become final, no additions can be made thereto and nothing can be done therewith except its execution. And execution cannot be had, the sheriff being ignorant as to how, for how much, and within what time may the option be exercised, and certainty no authority is vested in him to settle these matters which involve exercise of judicial discretion. Thus the judgment rendered by Judge Felix has never become final, it having left matters to be settled for its completion in a subsequent proceeding, matters which remained unsettled up to the time the petition is filed in the instant case. For all the foregoing, the writ of execution issued by Judge Natividad is hereby set aside and the lower court ordered to hold a hearing in the principal case wherein it must determine the prices of the buildings and of the residential lot where they are erected, as well as the period of time within which the plaintiffs-respondents may exercise their option either to pay for the buildings or to sell their land, and, in the last instance, the period of time within which the defendants-petitioners may pay for the land, all these periods to be counted from the date the judgment becomes executory or unappealable. After such hearing, the court shall render a final judgment according to the evidence presented by the parties. The costs shall be paid by plaintiffs-respondents.

"(3) That upon defendant's failure to purchase the residential lot in question, said defendants shall remove their houses and granaries after this decision becomes final and within the periods of sixty (60) days from the date that the court is informed in writing of the attitude of the parties in this respect. "No pronouncement is made as to damages and costs. "Once this decision becomes final, the plaintiffs and defendants may appear again before this court for the purpose of determining their respective rights under article 361 of the Civil Code, if they cannot come to an extra-judicial settlement with regard to said rights." Subsequently, in a motion filed in the same Court of First Instance but now presided over by the herein respondent Judge Hon. Felipe Natividad, the plaintiffs prayed for an order of execution alleging that since they chose neither to pay defendants for the buildings nor to sell to them the residential lot, said defendants should be ordered to remove the structure at their own expense and to restore plaintiffs in the possession of said lot. Defendants objected to this motion which, after hearing, was granted by Judge Natividad. Hence, this petition by defendants praying for (a) a restraint and annulment of the order of execution issued by Judge Natividad; (b) an order to compel plaintiffs to pay them the sum of P2,000 for the buildings, or sell to them the residential lot for P45; or (c) a rehearing of the case for a determination of the rights of the parties upon failure of extra-judicial settlement. The judgment rendered by Judge Felix is founded on articles 361 and 453 of the Civil Code which are as follows: "ART. 361. The owner of land on which anything has been built, sown or planted in good faith, shall have the right to appropriate as his own the work, sowing or planting, after the payment of the indemnity stated in articles 453 and 454, or to oblige the one who built or planted to pay the price of the land, and the one who sowed, the proper rent.

FRANCISCO DEPRA, plaintiff-appellee, vs. AGUSTIN DUMLAO, defendant-appellant.

MELENCIO-HERRERA, J.:

This is an appeal from the Order of the former Court of First Instance of Iloilo to the then Court of Appeals, which the latter certified to this instance as involving pure questions of law

Plaintiff-appellee, Francisco Depra, is the owner of a parcel of land registered under Transfer Certificate of Title No. T3087, known as Lot No. 685, situated in the municipality of Dumangas, Iloilo, with an area of approximately 8,870

Page 23 of 46

square meters. Agustin Dumlao, defendant-appellant, owns an adjoining lot, designated as Lot No. 683, with an approximate area of 231 sq. ms.

of action was based on ownership. Furthermore, Sec. 7, Rule 70 of the Rules of Court explicitly provides that judgment in a detainer case "shall not bar an action between the same parties respecting title to the land. " 4

Sometime in 1972, when DUMLAO constructed his house on his lot, the kitchen thereof had encroached on an area of thirty four (34) square meters of DEPRA's property, After the encroachment was discovered in a relocation survey of DEPRA's lot made on November 2,1972, his mother, Beatriz Depra after writing a demand letter asking DUMLAO to move back from his encroachment, filed an action for Unlawful Detainer on February 6,1973 against DUMLAO in the Municipal Court of of Dumangas, docketed as Civil Case No 1, Said complaint was later amended to include DEPRA as a party plain. plaintiff.

Conceded in the Stipulation of Facts between the parties is that DUMLAO was a builder in good faith. Thus,

8. That the subject matter in the unlawful detainer case, Civil Case No. 1, before the Municipal Court of Dumangas, Iloilo involves the same subject matter in the present case, the Thirty-four (34) square meters portion of land and built thereon in good faith is a portion of defendant's kitchen and has been in the possession of the defendant since 1952 continuously up to the present; ... (Emphasis ours)

After trial, the Municipal Court found that DUMLAO was a builder in good faith, and applying Article 448 of the Civil Code, rendered judgment on September 29, 1973, the dispositive portion of which reads: Consistent with the principle that our Court system, like any other, must be a dispute resolving mechanism, we accord legal effect to the agreement of the parties, within the context of their mutual concession and stipulation. They have, thereby, chosen a legal formula to resolve their dispute to appeal ply to DUMLAO the rights of a "builder in good faith" and to DEPRA those of a "landowner in good faith" as prescribed in Article 448. Hence, we shall refrain from further examining whether the factual situations of DUMLAO and DEPRA conform to the juridical positions respectively defined by law, for a "builder in good faith" under Article 448, a "possessor in good faith" under Article 526 and a "landowner in good faith' under Article 448.

Ordering that a forced lease is created between the parties with the plaintiffs, as lessors, and the defendants as lessees, over the disputed portion with an area of thirty four (34) square meters, the rent to be paid is five (P5.00) pesos a month, payable by the lessee to the lessors within the first five (5) days of the month the rent is due; and the lease shall commence on the day that this decision shall have become final.

From the foregoing judgment, neither party appeal so that, ff it were a valid judgment, it would have ordinarily lapsed into finality, but even then, DEPRA did not accept payment of rentals so that DUMLAO deposited such rentals with the Municipal Court.

In regards to builders in good faith, Article 448 of the Civil Code provides:

ART. 448. The owner of the land on which anything has been built sown or planted in good faith, On July 15,1974, DEPRA filed a Complaint for Quieting of Title against DUMLAO before the then Court of First Instance of Iloilo, Branch IV (Trial Court), involving the very same 34 square meters, which was the bone of contention in the Municipal Court. DUMLAO, in his Answer, admitted the encroachment but alleged, in the main, that the present suit is barred by res judicata by virtue of the Decision of the Municipal Court, which had become final and executory.

shall have the right

to appropriate as his own the works, sowing or planting, after payment of the indemnity provided for in articles 546 and 548, or

After the case had been set for pre-trial, the parties submitted a Joint Motion for Judgment based on the Stipulation of Facts attached thereto. Premised thereon, the Trial Court on October 31, 1974, issued the assailed Order, decreeing:

to oblige the one who built or planted to pay the price of the land, and the one who sowed, the proper rent.

WHEREFORE, the Court finds and so holds that the thirty four (34) square meters subject of this litigation is part and parcel of Lot 685 of the Cadastral Survey of Dumangas of which the plaintiff is owner as evidenced by Transfer Certificate of Title No. 3087 and such plaintiff is entitled to possess the same.

However, the builder or planter cannot be obliged to buy the land if its value is considerably more than that of the building or trees. In such case, he shall pay reasonable rent, if the owner of the land does not choose to appropriate the building or trees after proper indemnity. The parties shall agree upon the terms of the lease and in case of disagreement, the court shall fix the terms thereof (Paragraphing supplied)

Without pronouncement as to costs. Pursuant to the foregoing provision, DEPRA has the option either to pay for the encroaching part of DUMLAO's kitchen, or to sell the encroached 34 square meters of his lot to DUMLAO. He cannot refuse to pay for the encroaching part of the building, and to sell the encroached part of his land, 5 as he had manifested before the Municipal Court. But that manifestation is not binding because it was made in a void proceeding.

SO ORDERED.

Rebutting the argument of res judicata relied upon by DUMLAO, DEPRA claims that the Decision of the Municipal Court was null and void ab initio because its jurisdiction is limited to the sole issue of possession, whereas decisions affecting lease, which is an encumbrance on real property, may only be rendered by Courts of First Instance.

However, the good faith of DUMLAO is part of the Stipulation of Facts in the Court of First Instance. It was thus error for the Trial Court to have ruled that DEPRA is "entitled to possession," without more, of the disputed portion implying thereby that he is entitled to have the kitchen removed. He is entitled to such removal only when, after having chosen to sell his encroached land, DUMLAO fails to pay for the same. 6 In this case, DUMLAO had expressed his willingness to pay for the land, but DEPRA refused to sell.

Addressing out selves to the issue of validity of the Decision of the Municipal Court, we hold the same to be null and void. The judgment in a detainer case is effective in respect of possession only (Sec. 7, Rule 70, Rules of Court). 1 The Municipal Court over-stepped its bounds when it imposed upon the parties a situation of "forced lease", which like "forced co-ownership" is not favored in law. Furthermore, a lease is an interest in real property, jurisdiction over which belongs to Courts of First Instance (now Regional Trial Courts) (Sec. 44(b), Judiciary Act of 1948; 2 Sec. 19 (2) Batas Pambansa Blg. 129). 3 Since the Municipal Court, acted without jurisdiction, its Decision was null and void and cannot operate as res judicata to the subject complaint for Queting of Title. Besides, even if the Decision were valid, the rule on res judicata would not apply due to difference in cause of action. In the Municipal Court, the cause of action was the deprivation of possession, while in the action to quiet title, the cause

The owner of the building erected in good faith on a land owned by another, is entitled to retain the possession of the land until he is paid the value of his building, under article 453 (now Article 546). The owner of the land, upon the other hand, has the option, under article 361 (now Article 448), either to pay for the building or to sell his land to the owner of the building. But he cannot as respondents here did refuse both to pay for the building and to sell the land and compel the owner of the building to remove it from the

Page 24 of 46

land where it erected. He is entitled to such remotion only when, after having chosen to sell his land. the other party fails to pay for the same (italics ours).

The fairness of the rules in Article 448 has also been explained as follows:

We hold, therefore, that the order of Judge Natividad compelling defendants-petitioners to remove their buildings from the land belonging to plaintiffs-respondents only because the latter chose neither to pay for such buildings nor to sell the land, is null and void, for it amends substantially the judgment sought to be executed and is. furthermore, offensive to articles 361 (now Article 448) and 453 (now Article 546) of the Civil Code. (Ignacio vs. Hilario, 76 Phil. 605, 608[1946]).

A word anent the philosophy behind Article 448 of the Civil rode.

Where the builder, planter or sower has acted in good faith, a conflict of rights arises between the owners, and it becomes necessary to protect the owner of the improvements without causing injustice to the owner of the land. In view of the impracticability of creating a state of forced co-ownership, the law has provided a just solution by giving the owner of the land the option to acquire the improvements after payment of the proper indemnity, or to oblige the builder or planter to pay for the land and the sower to pay for the proper rent. It is the owner of the land who is authorized to exercise the option, because his right is older, and because, by the principle of accession, he is entitled to the ownership of the accessory thing. (3 Manresa 213; Bernardo vs. Bataclan, 37 Off. Gaz. 1382; Co Tao vs. Chan Chico, G.R. No. 49167, April 30, 1949; Article applied: see Cabral, et al vs. Ibanez [S.C.] 52 Off. Gaz. 217; Marfori vs. Velasco, [C.A.] 52 Off. Gaz. 2050). 8

The original provision was found in Article 361 of the Spanish Civil Code; which provided: WHEREFORE, the judgment of the trial Court is hereby set aside, and this case is hereby ordered remanded to the Regional Trial Court of Iloilo for further proceedings consistent with Articles 448 and 546 of the Civil Code, as follows:

ART. 361. The owner of land on which anything has been built, sown or planted in good faith, shall have the right to appropriate as his own the work, sowing or planting, after the payment of the indemnity stated in Articles 453 and 454, or to oblige the one who built or planted to pay the price of the land, and the one who sowed, the proper rent.

1. The trial Court shall determine As will be seen, the Article favors the owner of the land, by giving him one of the two options mentioned in the Article. Some commentators have questioned the preference in favor of the owner of the land, but Manresa's opinion is that the Article is just and fair.

a) the present fair price of DEPRA's 34 square meter area of land;

b) the amount of the expenses spent by DUMLAO for the building of the kitchen; . . . es justa la facultad que el codigo da al dueno del suelo en el articulo 361, en el caso de edificacion o plantacion? Algunos comentaristas la conceptuan injusta, y como un extraordinario privilegio en favor de la propiedad territorial. Entienden que impone el Codigo una pena al poseedor de buena fe y como advierte uno de los comentaristas aludidos 'no se ve claro el por que de tal pena . . . al obligar al que obro de buena fe a quedarse con el edificio o plantacion, previo el pago del terreno que ocupa, porque si bien es verdad que cuando edifico o planto demostro con este hecho, que queria para si el edificio o plantio tambien lo es que el que edifico o planto de buena fe lo hizo en la erronea inteligencia de creerse dueno del terreno Posible es que, de saber lo contrario, y de tener noticia de que habia que comprar y pagar el terreno, no se hubiera decidido a plantar ni a edificar. La ley obligandole a hacerlo fuerza su voluntad, y la fuerza por un hecho inocente de que no debe ser responsable'. Asi podra suceder pero la realidad es que con ese hecho voluntario, aunque sea inocente, se ha enriquecido torticeramente con perjuicio de otro a quien es justo indemnizarle,

c) the increase in value ("plus value") which the said area of 34 square meters may have acquired by reason thereof, and

d) whether the value of said area of land is considerably more than that of the kitchen built thereon.

2. After said amounts shall have been determined by competent evidence, the Regional, Trial Court shall render judgment, as follows:

En nuestra opinion, el Codigo ha resuelto el conflicto de la manera mas justa y equitativa y respetando en lo possible el principio que para la accesion se establece en el art. 358. 7

Our own Code Commission must have taken account of the objections to Article 361 of the Spanish Civil Code. Hence, the Commission provided a modification thereof, and Article 448 of our Code has been made to provide:

a) The trial Court shall grant DEPRA a period of fifteen (15) days within which to exercise his option under the law (Article 448, Civil Code), whether to appropriate the kitchen as his own by paying to DUMLAO either the amount of tile expenses spent by DUMLAO f or the building of the kitchen, or the increase in value ("plus value") which the said area of 34 square meters may have acquired by reason thereof, or to oblige DUMLAO to pay the price of said area. The amounts to be respectively paid by DUMLAO and DEPRA, in accordance with the option thus exercised by written notice of the other party and to the Court, shall be paid by the obligor within fifteen (15) days from such notice of the option by tendering the amount to the Court in favor of the party entitled to receive it;

ART. 448. The owner of the land on which anything has been built, sown or planted in good faith, shall have the right to appropriate as his own the works, sowing or planting, after payment of the indemnity provided for in articles 546 and 548, or to oblige the one who built or planted to pay the price of the land, and the one who sowed, the proper rent. However, the builder or planter cannot be obliged to buy the land if its value is considerably more than that of the building or trees. In such case, he shall pay reasonable rent, if the owner of the land does not choose to appropriate the building or trees after proper indemnity. The parties shall agree upon the terms of the lease and in case of disagreement, the court shall fix the terms thereof.

Additional benefits were extended to the builder but the landowner retained his options.

b) The trial Court shall further order that if DEPRA exercises the option to oblige DUMLAO to pay the price of the land but the latter rejects such purchase because, as found by the trial Court, the value of the land is considerably more than that of the kitchen, DUMLAO shall give written notice of such rejection to DEPRA and to the Court within fifteen (15) days from notice of DEPRA's option to sell the land. In that event, the parties shall be given a period of fifteen (15) days from such notice of rejection within which to agree upon the terms of the lease, and give the Court formal written notice of such agreement and its provisos. If no agreement is reached by the parties, the trial Court, within fifteen (15) days from and after the termination of the said period fixed for negotiation, shall then fix the terms of the lease, provided that the monthly rental to be fixed by the Court shall not be less than Ten Pesos (P10.00) per month, payable within the first five (5) days of each calendar month. The period for the forced lease shall not

Page 25 of 46

be more than two (2) years, counted from the finality of the judgment, considering the long period of time since 1952 that DUMLAO has occupied the subject area. The rental thus fixed shall be increased by ten percent (10%) for the second year of the forced lease. DUMLAO shall not make any further constructions or improvements on the kitchen. Upon expiration of the two-year period, or upon default by DUMLAO in the payment of rentals for two (2) consecutive months, DEPRA shall be entitled to terminate the forced lease, to recover his land, and to have the kitchen removed by DUMLAO or at the latter's expense. The rentals herein provided shall be tendered by DUMLAO to the Court for payment to DEPRA, and such tender shall constitute evidence of whether or not compliance was made within the period fixed by the Court.

Officer, and who in turn rendered a decision on April 9, 1958, dismissing plaintiff's claim and giving due course to defendants' sales applications on the ground that the relinquishment of the homestead rights of Martin Dolorico I in favor of Comintan and Zamora is proper, the former having been designated as successor in interest of the original homestead applicant and that because plaintiff failed to participate in the public auction, he is forever barred to claim the property; that plaintiff filed a motion for reconsideration of this decision which was denied by the Director of Lands in his order dated June 10, 1959; that finally, on appeal to the Secretary of Agriculture and Natural Resources, the decision rendered by the Regional Land Officer was affirmed in toto." 1 On March 22, 1966, respondent Court rendered judgment in the afore-mentioned civil case, the dispositive portion of which reads as follows: "IN VIEW OF THE FOREGOING CONSIDERATIONS, judgment is hereby rendered awarding Lot No. 5785-A of PLS-45, (Calauag Public Land Subdivision) one-half portion of the property in litigation located at Bo. Cabuluan, Calauag, Quezon in favor of defendant QUIRINO COMINTAN, being the successful bidder in the public auction conducted by the Bureau of Lands on April 18, 1955, and hereby giving due course to the Sales Application No. 9258 of defendant Eleuterio Zamora over the other half, Lot No. 5785-B of PLS-45, Calauag, without prejudice to the right of plaintiff BARTOLOME ORTIZ to participate in the public bidding of the same to be announced by the Bureau of Lands, Manila. However, should plaintiff Bartolome Ortiz be not declared the successful bidder thereof defendants Quirino Comintan and Eleuterio Zamora are ordered to reimburse jointly said plaintiff the improvements he has introduced on the whole property in the amount of THIRTEEN THOUSAND SIX HUNDRED THIRTY-TWO (P13,632.00) PESOS, the latter having the right to retain the property until after he has been fully paid therefor, without interest since he enjoys the fruits of the property in question, with prejudice and with costs against the plaintiff." 2 Plaintiff appealed the decision to the Court of Appeals. Two (2) years after the rendition of the judgment by the court a quo, while the case was pending appeal and upon petition of private respondents Quirino Comintan and Eleuterio Zamora, respondent Court appointed respondent Vicente Ferro, Clerk of Court, as Receiver to collect tolls on a portion of the property used as a diversion road. On August 19, 1969, the Court of Appeals issued a Resolution annulling the Order appointing the Receiver. Subsequently, on February 19, 1970, the Appellate Court affirmed the decision of the trial court. A petition for review on certiorari of the decision of the Court of Appeals was denied by this Court on April 6, 1970. At this point, private respondents filed a petition for appointment of a new receiver with the court a quo. This petition was granted and the receiver was reappointed. Petitioner sought the annulment of this Order with the Court of Appeals, but said Court ruled that its decision had already become final and that the records of the case were to be remanded to the trial court.

c) In any event, DUMLAO shall pay DEPRA an amount computed at Ten Pesos (P10.00) per month as reasonable compensation for the occupancy of DEPRA's land for the period counted from 1952, the year DUMLAO occupied the subject area, up to the commencement date of the forced lease referred to in the preceding paragraph;

d) The periods to be fixed by the trial Court in its Precision shall be inextendible, and upon failure of the party obliged to tender to the trial Court the amount due to the obligee, the party entitled to such payment shall be entitled to an order of execution for the enforcement of payment of the amount due and for compliance with such other acts as may be required by the prestation due the obligee.

No costs,

SO ORDERED.

BARTOLOME ORTIZ, petitioner, vs. HON. UNION C. KAYANAN, in his capacity as Judge of the Court of First Instance of Quezon, Branch IV; ELEUTERIO ZAMORA, QUIRINO COMINTAN, VICENTE FERRO, AND GREGORIO PAMISARAN, respondents.1979 July 302nd DivisionG.R. No. L-32974D E C I S I O N ANTONIO, J: Petition for Certiorari and Prohibition with Preliminary Injunction to nullify the Order of respondent Judge directing the execution of the final judgment in Civil Case No. C-90, entitled "Bartolome Ortiz vs. Secretary of Agriculture and Natural Resources, et al.," and the Writ of Execution issued to implement said Order, allegedly for being inconsistent with the Judgment sought to be enforced. Civil Case No. C-90 was filed by Bartolome Ortiz who sought the review and/or annulment of the decision of the Secretary of Agriculture and Natural Resources, giving preference to the sales applications of private respondents Quirino Comintan and Eleuterio Zamora over Lot No. 5785, PLS-45, located at Barrio Cabuluan, Calauag, Quezon. The factual background of the case, as found by respondent Court, is as follows: ". . . The lot in controversy was formerly the subject of Homestead Application No. 122417 of Martin Dolorico II, plaintiff's ward who died on August 20, 1931; that since then it was plaintiff who continued the cultivation and possession of the property, without however filing any application to acquire title thereon; that in the Homestead Application No. 122417, Martin Dolorico II named his uncle, Martin Dolorico I as his heir and successor in interest, so that in 1951 Martin Dolorico I executed an affidavit relinquishing his rights over the property in favor of defendants Quirino Comintan and Eleuterio Zamora, his grandson and son-in-law, respectively, and requested the Director of Lands to cancel the homestead application; that on the strength of the affidavit, Homestead Application No. 122417 was cancelled and thereafter, defendants Comintan and Zamora filed their respective sales applications Nos. 8433 and 9258; that plaintiff filed his protest on November 26, 1951 alleging that he should be given preference to purchase the lot inasmuch as he is the actual occupant and has been in continuous possession of the same since 1931; and inspite of plaintiff's opposition, 'Portion A' of the property was sold at public auction wherein defendant Comintan was the only bidder; that on June 8, 1957, investigation was conducted on plaintiff's protest by Assistant Public Lands Inspector Serapion Bauzon who submitted his report to the Regional Land

Not satisfied with such denial, petitioner filed a petition for certiorari, prohibition and mandamus with preliminary injunction before this Court, 3 praying for the annulment of the Order reappointing the Receiver. On July 13, 1970, the petition was dismissed by this Court on the ground of insufficient showing of grave abuse of discretion. II The judgment having become final and executory private respondents filed a motion for the execution of the same, praying as follows: "WHEREFORE, it is respectfully prayed of this Honorable Court to order the issuance of a writ of execution in accordance with the judgment of this Honorable Court, confirmed by the Court of Appeals and the Supreme Court, commanding any lawful officer to deliver to defendants Comintan and Zamora the land subject of the decision in this case but allowing defendants to file a bond in such amount as this Honorable Court may fix, in lieu of the P13,632.00 required to be paid to plaintiff, conditioned that after the accounting of the tools collected by plaintiff, there is still an amount due and payable to said plaintiff, then if such amount is not paid on demand, including the legal interests, said bond shall be held answerable.

Page 26 of 46

"Ordering further the plaintiff to render an accounting of the tolls he collected from March of 1967 to December 31, 1968 and from September 1969 to March 31, 1970, and deliver said tolls collected to the receiver and if judgment is already executed, then to Quirino Comintan and Eleuterio Zamora; and, "Finally, to condemn plaintiff to pay moral damages for withholding the tools which belong to your movant in an amount this Court may deem just in the premises." 4 Acting upon the foregoing motion, respondent Judge issued an Order, dated September 23, 1970, stating, among others, the following: "The records further disclosed that from March 1967 to December 31, 1968, plaintiff Bartolome Ortiz collected tolls on a portion of the property in question wherein he has not introduced any improvement particularly on Lot No. 5785-A; PLS-45 awarded to defendant Quirino Comintan, thru which vehicular traffic was detoured or diverted, and again from September 1969 to March 31, 1970, the plaintiff resumed the collection of tools on the same portion without rendering any accounting on said tolls to the Receiver, who was reappointed after submitting the required bond and specifically authorized only to collect tolls leaving the harvesting of the improvements to the plaintiff. xxxxxxxxx

"WHEREFORE, finding the Motion for Execution filed by the defendants to be meritorious, the same is granted; provided, however, that they put up a bond equal the adjudicated amount of P13,632.00 accruing in favor of the plaintiff, from a reputable or recognized bonding or surety company, conditioned that after an accounting of the tolls collected by the plaintiff should there be found out any balance due and payable to him after reckoning said obligation of P13,632.00 the bond shall be held answerable therefor." 5 Accordingly, a Writ of Execution was issued after private respondent Quirino Comintan had filed the required bond. The writ directed the Sheriff to enforce the decision of the Court, and stated, in part, the following: "But should there be found any amount collectible after accounting and deducting the amount of P13,632.00, you are hereby ordered that of the goods and chattels of Bartolome Ortiz of Bo. Kabuluan, Calauag, Quezon, be caused to be made any excess in the abovementioned amount together with your lawful fees and that you render same to defendant Quirino Comintan. If sufficient personal property cannot be found thereof to satisfy this execution and lawful fees thereon, then you are commanded that of the lands and buildings of the said BARTOLOME ORTIZ you make the said excess amount in the manner required by the Rules of Court, and make return of your proceedings within this Court within sixty (60) days from date of service. "You are also ordered to cause Bartolome Ortiz to vacate the property within fifteen (15) days after service thereof the defendant Quirino Comintan having filed the required bond in the amount of THIRTEEN THOUSAND SIX HUNDRED THIRTY-TWO (P13,632.00) PESOS." 6 On October 12, 1970, petitioner filed a Motion for Reconsideration of the aforesaid Order and Writ of Execution, alleging: "(a) That the respondent judge has no authority to place respondents in possession of the property; "(b) That the Supreme Court has never affirmed any decision of the trial court that tolls collected from the diversionary road on the property, which is public land, belong to said respondents; " That to assess petitioner a P25,000.00 liability for damages is purely punitive imposition without factual or legal justification." The foregoing Motion for Reconsideration was denied by respondent Judge per Order dated November 18, 1970. Said Order states, in part:

"In virtue of the findings of this Court as contained in the dispositive portion of its decision, the defendants are jointly obligated to pay the plaintiff in the amount of P13,632.00 as reasonable value of the improvements he introduced on the whole property in question, and that he has the right of retention until fully paid. It can be gleaned from the motion of the defendants that if plaintiff submits an accounting of the tolls he collected during the periods above alluded to, their damages of about P25,000.00 can more than offset their obligation of P13,362.00 in favor of the plaintiff, thereafter the possession of the land he delivered to the defendants since the decision of the Supreme Court has already become final and executory, but in the interregnum pending such accounting and recovery by the Receiver of the tolls collected by the plaintiff, the defendants pray that they allowed to put up a bond in lieu of the said P13,632.00 to answer for damages of the former, if any. "On the other hand, plaintiff contends in his opposition, admitting that the decision of the Supreme Court has become final and executory; (1) the offer of a bond in lieu of payment of P13,632.00 does not, and cannot, satisfy the condition imposed in the decision of this Court which was affirmed in toto; (2) the public sale of Portion 'B' of the land has still to take place as ordained before the decision could be executed; and, (3) that whatever sums plaintiff may derive from the property cannot be set off against what is due him for the improvements he made, for which he has to be reimbursed as ordered. xxxxxxxxx "Let it be known that plaintiff does not dispute his having collected tolls during the periods from March 1967 to December 31, 1968 and from September 1969 to March 31, 1970. The Supreme Court affirmed the decision of this Court in its findings that said tolls belong to the defendants, considering that the same were collected on a portion of the land in question where the plaintiff did not introduce any improvement. The reimbursement to the plaintiff pertains only to the value of the improvements, like coconut trees and other plants which he introduced on the whole property. The tolls collected by the plaintiff on an unimproved portion naturally belong to the defendants, following the doctrine on accretion. Further, the reappointment of a Receiver by this Court was upheld by the Supreme Court when it denied the petition for certiorari filed by the plaintiff, bolstering the legal claim of defendants over said tolls. Thus, the decision of the Supreme Court rendered the decision of this Court retroactive from March 22, 1966 although pending appeal its implementation was suspended. It is our honest conviction, therefore, that the putting up of a bond by the defendants pending accounting of the tolls collected by the plaintiff is justified and will not prejudice anybody, but certainly would substantially satisfy the conditions imposed in the decision. However, insofar as the one-half portion 'B' of the property, the decision may he executed only after public sale by the Bureau of Lands shall be accomplished.

"It goes without saying that defendant Comintan is entitled to be placed in possession of Lot No. 5785-A of PLS-45 (Calauag Public Land Subdivision) and enjoyment of the tolls from March, 1967 to March, 1968 and from September, 1969 to March 31, 1970 which were received by plaintiff Bartolome Ortiz, collected from the property by reason of the diversion road where vehicular traffic was detoured. To defendant Comintan belongs the tolls thus collected from a portion of the land awarded to him used as a diversionary road by the doctrine of accretion and his right over the same is ipso jure, there being no need of any action to possess said addition. It is so because as consistently maintained by the Supreme Court, an applicant who has complied with all the terms and conditions which entitle him to a patent for a particular tract of public land, acquires a vested right therein and is to be regarded as equitable owner thereof so that even without a patent, a perfected homestead or sales application is a property right in the fullest sense, unaffected by the fact that the paramount title is still in the Government and no subsequent law can deprive him of that vested right. The question of the actual damages suffered by defendant Comintan by reason of the unaccounted tolls received by plaintiff had already been fully discussed in the order of September 23, 1970 and the Court is honestly convinced and believes it to be proper and regular under the circumstances. "Incidentally, the Court stands to correct itself when in the same order, it directed the execution of the decision with respect to the one-half portion 'B' of the property only after the public sale by the Bureau of Lands, the same being an oversight, it appearing that the Sales Application of defendant Eleuterio Zamora had already been recognized and fully confirmed by the Supreme Court.

Page 27 of 46

"In view thereof, finding the motion filed by plaintiff to be without merit, the Court hereby denies the same and the order of September 23, 1970 shall remain in full force subject to the amendment that the execution of the decision with respect to the one-half portion 'B' shall not be conditioned to the public sale by the Bureau of Lands. "SO ORDERED." 7 III Petitioner thus filed the instant petition, contending that in having issued the Order and Writ of Execution, respondent Court "acted without or in excess of jurisdiction, and/or with grave abuse of discretion, because the said order and writ in effect vary the terms of the judgment they purportedly seek to enforce." He argued that since said judgment declared the petitioner a possessor in good faith, he is entitled to the payment of the value of the improvements introduced by him on the whole property, with right to retain the land until he has been fully paid such value. He likewise averred that no payment for improvements has been made and, instead, a bond therefor had been filed by defendants (private respondents), which, according to petitioner, is not the payment envisaged in the decision which would entitle private respondents to the possession of the property. Furthermore, with respect to portion "B", petitioner alleges that, under the decision, he has the right to retain the same until after he has participated and lost in the public bidding of the land to be conducted by the Bureau of Lands. It is claimed that it is only in the event that he loses in the bidding that he can be legally dispossessed thereof. It is the position of petitioner that all the fruits of the property, including the tolls collected by him from the passing vehicles, which according to the trial court amounts to P25,000.00, belongs to petitioner and not to defendant/private respondent Quirino Comintan, in accordance with the decision itself, which decreed that the fruits of the property shall be in lieu of interest on the amount to be paid to petitioner as reimbursement for improvements. Any contrary opinion, in his view, would be tantamount to an amendment of a decision which has long become final and executory and, therefore, cannot be lawfully done. Petitioner, therefore, prayed that (1) a Writ of Preliminary Injunction be issued enjoining the enforcement of the Orders of September 23, 1970 and November 18, 1970, and the Writ of Execution issued thereto, or restoring to petitioner the possession of the property if the private respondents had been placed in possession thereof; (2) annulling said Orders as well as the Writ of Execution, dissolving the receivership established over the property; and (3) ordering private respondents to account to petitioner all the fruits they may have gathered or collected from the property in question from the time of petitioner's illegal dispossession thereof. On January 29, 1971, this Court issued the Writ of Preliminary Injunction. On January 30, 1971, private respondents filed a Motion for Reconsideration and/or Modification of the Order dated January 29, 1971. This was followed by a Supplemental Motion for Reconsideration and Manifestation on February 3, 1971. In the latter motion, private respondents manifested that the amount of P14,040.96, representing the amount decreed in the judgment as reimbursement to petitioner for the improvements, plus interest for six months, has already been deposited by them in court, "with the understanding that said amount shall be turned over to the plaintiff after the court a quo shall have determined the improvement on Lot 5785-A, and subsequently the remaining balance of the deposit shall be delivered to the petitioner (plaintiff therein) in the event he loses the bid for Lot 5785-B in favor of private respondent Eleuterio Zamora." 8 The deposit is evidenced by a certification made by the Clerk of the Court a quo. 9 Contending that said deposit was a faithful compliance with the judgment of the trial court, private respondent Quirino Comintan prayed for the dissolution of the Writ of Injunction.

not constitute sufficient compliance with the judgment sought to be enforced, neither was it legally and validly made because the requisites for consignation had not been complied with; that the tender of legal interest for six months cannot substitute petitioner's enjoyment of the fruits of the property as long as the judgment in Civil Case No. C-90 has not been implemented in the manner decreed therein; that contrary to the allegations of private respondents, the value of the improvements on the whole property had been determined by the lower court, and the segregation of the improvements for each lot should have been raised by them at the opportune moment by asking for the modification of the decision before it became final and executory; and that the tolls on the property constituted "civil fruits" to which the petitioner is entitled under the terms of the decision. IV The issue decisive of the controvercy is after the rendition by the trial court of its judgment in Civil Case No. C-90 on March 22, 1966 confirming the award of one-half of the property to Quirino Comintan whether or not petitioner is still entitled to retain for his own exclusive benefit all the fruits of the property, such as the tolls collected by him from March 1967 to December 1968, and September 1969 to March 31, 1970, amounting to about P25,000.00. In other words, petitioner contends that so long as the aforesaid amount of P13,632.00 decreed in the judgment representing the expenses for clearing the land and the value of the coconuts and fruit trees planted by him remains unpaid, he can appropriate for his exclusive benefit all the fruits which he may derive from the property, without any obligation to apply any portion thereof to the payment of the interest and the principal of the debt. We find this contention untenable. There is no question that a possessor in good faith is entitled to the fruits received before the possession is legally interrupted. 11 Possession in good faith ceases or is legally interrupted from the moment defects in the title are made known to the possessor, by extraneous evidence or by the filing of an action in court by the true owner for the recovery of the property. 12 Hence, all the fruits that the possessor may receive from the time he is summoned in court, or when he answers the complaint, must be delivered and paid by him to the owner or lawful possessor. 13

It appears that as a consequence of the deposit made by private respondents, the Deputy Sheriff of Calauag, Quezon ousted petitioner's representative from the land in question and put private respondents in possession thereof. 10 On March 10, 1971, petitioner filed a "Comment on Respondents' 'Motion for Reconsideration' dated January 29, 1971' and 'Supplemental Motion for Reconsideration and Manifestation,"' contending that the tender of deposit mentioned in the Supplemental Motion was not really and officially made, "'inasmuch as the same is not supported by any official receipt from the lower court, or from its clerk or cashier, as required by law;" that said deposit does

However, even after his good faith ceases, the possessor in fact can still retain the property, pursuant to Article 546 of the New Civil Code, until he has been fully reimbursed for all the necessary and useful expenses made by him on the property. This right of retention has been considered as one of the conglomerate of measures devised by the law for the protection of the possessor in good faith. Its object is to guarantee the reimbursement of the expenses, such as those for the preservation of the property, 14 or for the enhancement of its utility or productivity. 15 It permits the actual possessor to remain in possession while he has not been reimbursed by the person who defeated him in the possession for those necessary expenses and useful improvements made by him on the thing possessed. The principal characteristic of the right of retention is its accessory character. It is accessory to a principal obligation. Considering that the right of the possessor to receive the fruits terminates when his good faith ceases, it is necessary, in order that this right to retain may be useful, to concede to the creditor the right to secure reimbursement from the fruits of the property by utilizing its proceeds for the payment of the interest as well as the principal of the debt while he remains in possession. This right of retention of the property by the creditor, according to Scaevola, in the light of the provisions of Article 502 of the Spanish Civil Code, 16 is considered not a coercive measure to oblige the debtor to pay, depriving him temporarily of the enjoyment of the fruits of his property, but as a means of obtaining compensation for the debt. The right of retention in this case is analogous to a contract of antichresis and it can be considered as a means of extinguishing the obligation, inasmuch as the right to retain the thing lasts only for the period necessary to enable the creditor to be reimbursed from the fruits for the necessary and useful expenses. 17 According to Manresa, the right of retention is, therefore, analogous to that of a pledge, if the property retained is a movable, and to that of antichresis, if the property held is immovable. 18 This construction appears to be in harmony with similar provisions of the civil law which employs the right of retention as a means or device by which a creditor is able to obtain the payment of a debt. Thus, under Article 1731 of the New Civil Code, any person who has performed work upon a movable has a right to retain it by way of pledge until he is paid. Similarly, under Article 1914 of the same Code, the agent may retain in pledge the things which are the object of the agency until the principal effects reimbursement of the funds advanced by the former for the execution of the agency, or he is

Page 28 of 46

indemnified for all damages which he may have suffered as a consequence of the execution of the agency, provided he is free from fault. To the same effect, the depository, under Article 1994 of the same Code, may retain the thing in pledge until the full payment of what may be due him by reason of the deposit. The usufructuary, pursuant to Article 612 of the same Code, may retain the property until he is reimbursed for the amount paid for taxes levied on the capital (Article 597) and for extraordinary repairs (Article 594).

HON. INTERMEDIATE APPELLATE COURT, JUAN IGNAO, substituted by his Legal Heirs, and ISIDRO IGNAO, respondents.

FERNAN, C.J.:p

In all of these cases, the right of retention is used as a means of extinguishing the obligation. As amply observed by Manresa: "El derecho de retencion, lo hemos dicho, es el derecho de prenda o el de anticresis constituido por la ley con independencia de la voluntad de las partes." 19 In a pledge if the thing pledged earns or produces fruits, income, dividends or interests, the creditor shall compensate what he receives with those which are owing him. 20 In the same manner, in a contract of antichresis, the creditor acquires the right to receive the fruits of an immovable of his debtor with the obligation to apply them to the payment of the interest, if owing, and thereafter to the principal of his credit. 21 The debtor can not reacquire enjoyment of the immovable until he has actually paid what he owes the creditor. 22 Applying the afore-cited principles to the case at bar, petitioner cannot appropriate for his own exclusive benefit the tolls which he collected from the property retained by him. It was his duty under the law, after deducting the necessary expenses for his administration, to apply such amount collected to the payment of the interest, and the balance to the payment of the principal of the obligation. We hold, therefore, that the disputed tolls, after deducting petitioner's expenses for administration, belong to Quirino Comintan, owner of the land through which the toll road passed, further considering that the same was on portions of the property on which petitioner had not introduced any improvement. The trial court itself clarified this matter when it placed the toll road under receivership. The omission of any mention of the tolls in the decision itself may be attributed to the fact that the tolls appear to have been collected after the rendition of the judgment of the trial court. The records further reveal that earnest efforts have been made by private respondents to have the judgment executed in the most practicable manner. They deposited in court the amount of the judgment in the sum of P13,632.00 in cash, subject only to the accounting of the tolls collected by the petitioner so that whatever is due from him may be set off with the amount of reimbursement. This is just and proper under the circumstances and, under the law, compensation or set off may take place, either totally or partially. Considering that petitioner is the creditor with respect to the judgment obligation and the debtor with respect to the tolls collected, Comintan being the owner thereof, the trial court's order for an accounting and compensation is in accord with law. 23 With respect to the amount of reimbursement to be paid by Comintan, it appears that the dispositive portion of the decision was lacking in specificity, as it merely provided that Comintan and Zamora are jointly liable therefor. When two persons are liable under a contract or under a judgment, and no words appear in the contract or judgment to make each liable for the entire obligation, the presumption is that their obligation is joint or mancomunada, and each debtor is liable only for a proportionate part of the obligation. 24 The judgment debt of P13,632.00 should, therefore, be pro-rated in equal shares to Comintan and Zamora. Regarding Lot 5785-B, it appears that no public sale has yet been conducted by the Bureau of Lands and, therefore, petitioner is entitled to remain in possession thereof. This is not disputed by respondent Eleuterio Zamora. 25 After public sale is had and in the event that Ortiz is not declared the successful bidder, then he should be reimbursed by respondent Zamora in the corresponding amount for the improvements on Lot 5785-B. WHEREFORE, in view hereof, the Order of respondent Court of November 18, 1970 is hereby modified to conform to the foregoing judgment. The Writ of Preliminary Injunction, dated January 29, 1971, is hereby dissolved. Without special pronouncement as to costs.

In this petition for review by certiorari, petitioner seeks the reversal of the decision of the Intermediate Appellate Court (now Court of Appeals) affirming in toto the decision of the Court of First Instance of Cavite, ordering petitioner Florencio Ignao to sell to private respondents Juan and Isidro Ignao, that part of his property where private respondents had built a portion of their houses.

The antecedent facts are as follows:

Petitioner Florencio Ignao and his uncles private respondents Juan Ignao and Isidro Ignao were co-owners of a parcel of land with an area of 534 square meters situated in Barrio Tabon, Municipality of Kawit, Cavite. Pursuant to an action for partition filed by petitioner docketed as Civil Case No. N-1681, the then Court of First Instance of Cavite in a decision dated February 6, 1975 directed the partition of the aforesaid land, alloting 133.5 square meters or 2/8 thereof to private respondents Juan and Isidro, and giving the remaining portion with a total area of 266.5 square meters to petitioner Florencio. However, no actual partition was ever effected. 1

On July 17, 1978, petitioner instituted a complaint for recovery of possession of real property against private respondents Juan and Isidro before the Court of First Instance of Cavite, docketed as Civil Case No. 2662. In his complaint petitioner alleged that the area occupied by the two (2) houses built by private respondents exceeded the 133.5 square meters previously alloted to them by the trial court in Civil Case No. N-1681.

Consequently, the lower court conducted an ocular inspection. It was found that the houses of Juan and Isidro actually encroached upon a portion of the land belonging to Florencio. Upon agreement of the parties, the trial court ordered a licensed geodetic engineer to conduct a survey to determine the exact area occupied by the houses of private respondents. The survey subsequently disclosed that the house of Juan occupied 42 square meters while that of Isidro occupied 59 square meters of Florencio's land or a total of 101 square meters.

In its decision, the trial court (thru Judge Luis L. Victor) ruled that although private respondents occupied a portion of Florencio's property, they should be considered builders in good faith. The trial court took into account the decision of the Court of First Instance of Cavite in the action for partition 2 and quoted:

. . . . Hence, it is the well-considered opinion of the Court that although it turned out that the defendants had, before partition, been in possession of more than what rightfully belongs to them, their possession of what is in excess of their rightful share can at worst be possession in good faith which exempts them from being condemned to pay damages by reason thereof. 3

Furthermore, the trial court stated that pursuant to Article 448 of the Civil Code, the owner of the land (Florencio) should have the choice to either appropriate that part of the house standing on his land after payment of indemnity or oblige the builders in good faith (Juan and Isidro) to pay the price of the land. However, the trial court observed that based on the facts of the case, it would be useless and unsuitable for Florencio to exercise the first option since this would render the entire houses of Juan and Isidro worthless. The trial court then applied the ruling in the similar case of Grana vs. Court of Appeals, 4 where the Supreme Court had advanced a more "workable solution". Thus, it ordered Florencio to sell to Juan and Isidro those portions of his land respectively occupied by the latter. The dispositive portion of said decision reads as follows:

WHEREFORE, judgment is hereby rendered in favor of the defendants and

FLORENCIO IGNAO, petitioner, vs.

(a) Ordering the plaintiff Florencio Ignao to sell to the defendants Juan and Isidro Ignao that portion of his property with an area of 101 square meters at P40.00 per square meter, on which part the defendants had built their houses; and

Page 29 of 46

(b) Ordering the said plaintiff to execute the necessary deed of conveyance to the defendants in accordance with paragraph (a) hereof.
5

Without pronouncement as to costs.

Petitioner Florencio Ignao appealed to the Intermediate Appellate Court. On August 27, 1985, the Appellate Court, Second Civil Cases Division, promulgated a decision, 6 affirming the decision of the trial court.

Art. 448. The owner of the land on which anything has been built, sown or planted in good faith, shall have the right to appropriate as his own the works, sowing or planting, after payment of the indemnity provided for in articles 546 and 548, or to oblige the one who built or planted to pay the price of the land, and the one who sowed, the proper rent. However, the builder or planter cannot be obliged to buy the land if its value is considerably more than that of the building or trees. In such case, he shall pay reasonable rent, if the owner of the land does not choose to appropriate the building or trees after proper indemnity. The parties shall agree upon the terms of the lease and in case of disagreement, the court shall fix the terms thereof.

Hence the instant petition for review which attributes to the Appellate Court the following errors:

Whether or not the provisions of Article 448 should apply to a builder in good faith on a property held in common has been resolved in the affirmative in the case of Spouses del Campo vs. Abesia, 10 wherein the Court ruled that:

1. That the respondent Court has considered private respondents builders in good faith on the land on question, thus applying Art. 448 of the Civil Code, although the land in question is still owned by the parties in co-ownership, hence, the applicable provision is Art. 486 of the Civil Code, which was not applied.

The court a quo correctly held that Article 448 of the Civil Code cannot apply where a co-owner builds, plants or sows on the land owned in common for then he did not build, plant or sow upon land that exclusively belongs to another but of which he is a coowner. The co-owner is not a third person under the circumstances, and the situation is governed by the rules of co-ownership.

2. That, granting for the sake of argument that Art. 448 . . . is applicable, the respondent Court has adjudged the working solution suggested in Grana and Torralba vs. CA. (109 Phil. 260), which is just an opinion by way of passing, and not the judgment rendered therein, which is in accordance with the said provision of the Civil Code, wherein the owner of the land to buy (sic) the portion of the building within 30 days from the judgment or sell the land occupied by the building.

However, when, as in this case, the ownership is terminated by the partition and it appears that the home of defendants overlaps or occupies a portion of 5 square meters of the land pertaining to plaintiffs which the defendants obviously built in good faith, then the provisions of Article 448 of the new Civil Code should apply. Manresa and Navarro Amandi agree that the said provision of the Civil Code may apply even when there is a co-ownership if good faith has been established. 11

3. That, granting that private respondents could buy the portion of the land occupied by their houses, the price fixed by the court is unrealistic and pre-war price. 7 In other words, when the co-ownership is terminated by a partition and it appears that the house of an erstwhile coowner has encroached upon a portion pertaining to another co-owner which was however made in good faith, then the provisions of Article 448 should apply to determine the respective rights of the parties.

The records of the case reveal that the disputed land with an area of 534 square meters was originally owned by Baltazar Ignao who married twice. In his first marriage, he had four children, namely Justo (the father of petitioner Florencio), Leon and private respondents Juan and Isidro. In his second marriage, Baltazar had also four children but the latter waived their rights over the controverted land in favor of Justo. Thus, Justo owned 4/8 of the land which was waived by his half-brothers and sisters plus his 1/8 share or a total of 5/8. Thereafter, Justo acquired the 1/8 share of Leon for P500.00 which he later sold to his son Florencio for the same amount. When Justo died, Florencio inherited the 5/8 share of his father Justo plus his 1/8 share of the land which he bought or a total of 6/8 (representing 400.5 square meters). Private respondents, Juan and Isidro, on the other hand, had 1/8 share (66.75 square meters) each of the land or a total of 133.5 square meters.

Petitioner's second assigned error is however well taken. Both the trial court and the Appellate Court erred when they peremptorily adopted the "workable solution" in the case of Grana vs. Court of appeals, 12 and ordered the owner of the land, petitioner Florencio, to sell to private respondents, Juan and Isidro, the part of the land they intruded upon, thereby depriving petitioner of his right to choose. Such ruling contravened the explicit provisions of Article 448 to the effect that "(t)he owner of the land . . . shall have the right to appropriate . . .or to oblige the one who built . . . to pay the price of the land . . . ." The law is clear and unambiguous when it confers the right of choice upon the landowner and not upon the builder and the courts. Thus, in Quemuel vs. Olaes, 13 the Court categorically ruled that the right to appropriate the works or improvements or to oblige the builder to pay the price of the land belongs to the landowner.

Before the decision in the partition case was promulgated, Florencio sold 134 square meters of his share to a certain Victa for P5,000.00 on January 27, 1975. When the decision was handed down on February 6,1975, the lower court alloted 2/8 of the land to private respondents Juan and Isidro, or a total of 133.5 square meters.

It should be noted that prior to partition, all the co-owners hold the property in common dominion but at the same time each is an owner of a share which is abstract and undetermined until partition is effected. As cited in Eusebio vs. Intermediate Appellate Court, 8 "an undivided estate is co-ownership by the heirs."

As to the third assignment of error, the question on the price to be paid on the land need not be discussed as this would be premature inasmuch as petitioner Florencio has yet to exercise his option as the owner of the land.

As co-owners, the parties may have unequal shares in the common property, quantitatively speaking. But in a qualitative sense, each co-owner has the same right as any one of the other co-owners. Every co-owner is therefore the owner of the whole, and over the whole he exercises the right of dominion, but he is at the same time the owner of a portion which is truly abstract, because until division is effected such portion is not concretely determined. 9

Petitioner Florencio, in his first assignment of error, asseverates that the court a quo erred in applying Article 448 of the Civil Code, since this article contemplates a situation wherein the land belongs to one person and the thing built, sown or planted belongs to another. In the instant case, the land in dispute used to be owned in common by the contending parties.

WHEREFORE, the decision appealed from is hereby MODIFIED as follows: Petitioner Florencio Ignao is directed within thirty (30) days from entry of judgment to exercise his option to either appropriate as his own the portions of the houses of Juan and Isidro Ignao occupying his land upon payment of indemnity in accordance with Articles 546 and 548 of the Civil Code, or sell to private respondents the 101 square meters occupied by them at such price as may be agreed upon. Should the value of the land exceed the value of the portions of the houses that private respondents have erected thereon, private respondents may choose not to buy the land but they must pay reasonable rent for the use of the portion of petitioner's land as may be agreed upon by the parties. In case of disagreement, the rate of rental and other terms of the lease shall be determined by the trial court. Otherwise, private respondents may remove or demolish at their own expense the said portions of their houses encroaching upon petitioner's land. 14 No costs.

SO ORDERED. Article 448 provides:

Page 30 of 46

MANOTOK REALTY, INC., petitioner, vs. THE HONORABLE JOSE H. TECSON, Judge of the Court of First Instance of Manila and NILO MADLANGAWA, respondents.

WHEREFORE, and for lack of merit, the instant motion for approval of the plaintiff's fairn of option and for satisfaction of judgment should be, as hereby it is, denied. (pp. 4546, Rollo)

GUTIERREZ, JR., J.:

In a complaint filed by the petitioner for recovery of possession and damages against the private respondent, the then Court of First Instance of Manila rendered judgment, the dispositive portion of which provides inter alia:

After a denial of its motion for reconsideration, the petitioner filed the present petition for mandamus alleging that the respondent judge committed grave abuse of discretion in denying his motion to exercise option and for execution of judgment on the grounds that under Articles 448 and 546 of the Civil Code, the exercise of option belongs to the owner of the property, who is the petitioner herein, and that upon finality of judgment, the prevailing party is entitled, as a matter of right, to its execution which is only a ministerial act on the part of the respondent judge.

WHEREFORE, judgment is hereby rendered:

xxx xxx xxx

(c) In Civil Case No. 72872, declaring the defendant Nilo Madlangawa as a builder or possessor in good faith; ordering the plaintiff to recognize the right of said defendant to remain in Lot No. 345, Block 1, of the Clara Tambunting Subdivision until after he shall have been reimbursed by the plaintiff the sum of P7,500.00, without pronouncement as to costs. (p. 24, Rollo)

On April 15, 1978, the private respondent filed his comment on the petition alleging that the same has already become moot and academic for two reasons: first, fire gutted not only the house of the private respondent but the majority of the houses in Tambunting Estate; and second, as a result of the said fire, the then First Lady and Metro Manila Governor Imelda R. Marcos has placed the disputed area under her Zonal Improvement Project, thereby allowing the victims of the fire to put up new structures on the premises, so that the willingness and readiness of the petitioner to exercise the alleged option can no longer be exercised since the subject-matter thereof has been extinguished by the fire. Furthermore, the President of the Philippines has already issued a Presidential Decree for the expropriation of certain estates in Metro Manila including the Tambunting Estate. Therefore, the beneficient and humanitarian purpose of the Zonal Improvement Project and the expropriation proceeding would be defeated if petitioner is allowed to exercise an option which would result in the ejectment of the private respondent.

Not satisfied with the trial court's decision, the petitioner appealed to the Court of Appeals and upon affirmance by the latter of the decision below, the petitioner elevated its case to this Court.

On December 28, 1980, Presidential Decree (P.D.) No. 1669 was issued providing for the expropriation of the Tambunting Estate. However, this decree was challenged before this Court in G.R. No, 55166 entitled The "Elisa R. Manotok, et al. v. National Housing Authority, et al." Hence, we decided to hold the decision on this petition pending the resolution of the above entitled case.

On July 13, 1977, we issued a resolution dated July 11, 1977 denying the petitioner's petition for lack of merit. Hence, on August 5, 1977, the petitioner filed with the trial court, presided over by respondent Judge Jose H. Tecson, a motion for the approval of petitioner's exercise of option and for satisfaction of judgment, praying that the court issue an order: a) approving the exercise of petitioner's option to appropriate the improvements introduced by the private respondent on the property; b) thereafter, private respondent be ordered to deliver possession of the property in question to the petitioner.

On May 21, 1987, the Court rendered a decision in the Elisa Manotok case (Manotok v. National Housing Authority, 150 SCRA 89) ruling that P.D. 1669 is unconstitutional for being violative of the due process clause. Thus, since the present petition has not been rendered moot and academic by the decision in said case, we will now decide on its merits.

On October 7, 1977, the respondent judge issued the disputed order, to wit:

As stated earlier, the petitioner argues that since the judgment of the trial court has already become final, it is entitled to the execution of the same and that moreover, since the house of the private respondent was gutted by fire, the execution of the decision would now involve the delivery of possession of the disputed area by the private respondent to the petitioner.

Acting on the motion for approval of plaintiffs exercise of option and for satisfaction of judgment filed by the plaintiff, and the opposition thereto interposed by the defendant, both through counsels, and after a judicious review of all the facts and circumstances obtaining in this case, in the light of statutory provisions (Art. 6, New Civil Code) and jurisprudential doctrines (Vide, Benares v. Capitol Subdivision, Inc., L-7330 (Nov. 29, 1960), and considering further the definitive ruling of our Supreme Tribunal in the case of Jose C. Cristobal v. Alejandro Melchor, G.R. No. L-43203 promulgated on July 29, 1977, wherein the Court says:

We find merit in these arguments.

When the decision of the trial court became final and executory, it became incumbent upon the respondent judge to issue the necessary writ for the execution of the same. There is, therefore, no basis for the respondent judge to deny the petitioner's motion to avail of its option to approriate the improvements made on its property.

"This Court, applying the principle of equity, need not be bound to a rigid application of the law but rather its action should conform to the conditions or exigencies of a given problem or situation in order to grant relief that it will serve the ends of justice."

In the case of Duenas v. Mandi (151 SCRA 530, 545), we said:

xxx xxx xxx xxx xxx xxx ...Likewise settled is the rule that after a judgment has become final, no additions can be made thereto, and nothing can be done therewith except its execution, otherwise there would be no end to legal processes. (Fabular v. Court of Appeals, 11 9 SCRA 329)

the Court is of the considered view that under the peculiar circumstances which supervened after the institution of this case, like, for instance, the introduction of certain major repairs of and other substantial improvements on the controverted property, the instant motion of the plaintiff is not well-taken and therefore not legally proper and tenable.

Neither can the respondent judge deny the issuance of a writ of execution because the private respondent was adjudged a builder in good faith or on the ground of "peculiar circumstances which supervened after the institution of this case, like, for instance, the introduction of certain major repairs of and other substantial improvements..." because the option given by law either to retain the premises and pay for the improvements thereon or to sell the

Page 31 of 46

said premises to the builder in good faith belongs to the owner of the property. As we have in Quemel v. Olaes (1 SCRA 1159,1163):

SPOUSES CONCEPCION FERNANDEZ DEL CAMPO and ESTANISLAO DEL CANTO, plaintiffs-appellees, vs. BERNARDA FERNANDEZ ABESIA, defendant-appellant.

xxx xxx xxx GANCAYCO, J.: ...The plaintiffs claim that their second cause of action is based on Article 448 in connection with Art. 546, of the new Civil Code. A cursory reading of these provisions, however, will show that they are not applicable to plaintiff's case. Under Article 448, the right to appropriate the works or improvements or to oblige the one who built or planted to pay the price of the land' belongs to the owner of the land. The only right given to the builder in good faith is the right to reimbursement for the improvements; the builder, cannot compel the owner of the land to sell such land to the former. ...

In this appeal from the decision of the Court of First Instance (CFI) of Cebu, certified to this Court by the Court of Appeals on account of the question of law involved, the sole issue is the applicability of the provisions of Article 448 of the Civil Code relating to a builder in good faith when the property involved is owned in common.

Again, in the recent case of Paz Mercado, et al. v. Hon. Court of Appeals, et al., (G.R. No. L- 44001, June 10, 1988), we said:

... To be deemed a builder in good faith, it is essential that a person assert title to the land on which he builds; i.e., that he be a possessor in concept of owner (Art. 525, Civil Code; Lopez, Inc. v. Phil. Eastern Trading Co., Inc., 98 Phil. 348) and that he be unaware 'that there exists in his title or mode of acquisition any flaw which invalidates it.' (Art. 526, Civil Code; Granados v. Monton, 86 Phil. 42; Arriola v. Gomez de la Serna, 14 Phil. 627; See also Manotok Realty, Inc. v. C.A., 134 SCRA 329, citing Caram v. Laureta, 103 SCRA 7) It is such a builder in good faith who is given the 1ight to retain the thing, even as against the real owner, until he has been reimbursed in full not only for the necessary expenses but also for useful expenses. (Art. 546, Civil Code; Policarpio v. CA., 129 SCRA 51; Sarmiento v. Agana, 129 SCRA 1221; cf, Queto v. C.A., 122 SCRA 206) ...

This case involves a parcel of land, Lot No. 1161 of the Cadastral Survey of Cebu, with an area of only about 45 square meters, situated at the corner of F. Flores and Cavan Streets, Cebu City covered by TCT No. 61850. An action for partition was filed by plaintiffs in the CFI of Cebu. Plaintiffs and defendants are co-owners pro indiviso of this lot in the proportion of and 1/3 share each, respectively. The trial court appointed a commissioner in accordance with the agreement of the parties. ,the Id commissioner conducted a survey, prepared a sketch plan and submitted a report to the trial court on May 29, 1976, recommending that the property be divided into two lots: Lot 1161-A with an area of 30 square meters for plaintiffs and Lot No. 1161-B with an area of 15 square meters for the defendants. The houses of plaintiffs and defendants were surveyed and shown on the sketch plan. The house of defendants occupied the portion with an area of 5 square meters of Lot 1161-A of plaintiffs. The parties manifested their conformity to the report and asked the trial court to finally settle and adjudicate who among the parties should take possession of the 5 square meters of the land in question.

In solving the issue the trial court held as follows:

Furthermore, the private respondent's good faith ceased after the filing of the complaint below by the petitioner. In the case of Mindanao Academy, Inc. v. Yap (13 SCRA 190,196), we ruled:

xxx xxx xxx

...Although the bad faith of one party neutralizes that of the other and hence as between themselves their rights would be as if both of them had acted in good faith at the time of the transaction, this legal fiction of Yap's good faith ceased when the complaint against him was filed, and consequently the court's declaration of liability for the rents thereafter is correct and proper . A possessor in good faith is entitled to the fruits only so long as his possession is not legally interrupted, and such interruption takes place upon service of judicial summons (Arts. 544 and 1123, Civil Code).

The Court believed that the plaintiffs cannot be obliged to pay for the value of the portion of the defendants' house which has encroached an area of five (5) sq. meters of the land alloted to them. The defendants cannot also be obliged to pay for the price of the said five (5) square meters. The rights of a builder in good faith under Article 448 of the New Civil Code does (sic) not apply to a case where one co-owner has built, planted or sown on the land owned in common. "Manresa agreeing with Sanchez Roman, says that as a general rule this article is not applicable because the matter should be governed more by the provisions on co-ownership than on accession. Planiol and Ripert are also of the opinion that this article is not applicable to a co-owner who constructs, plants or sows on the community property, even if the land where the construction, planting or sowing is made is a third person under the circumstances, and the situation is governed by the rules of co-ownership. Our Court of Appeals has held that this article cannot be invoked by one co-owner against another who builds, plants or sows upon their land, since the latter does not do so on land not belonging to him. (C.A.), O.G. Supp., Aug. 30, 194, p. 126). In the light of the foregoing authorities and considering that the defendants have expressed their conformity to the partition that was made by the commissioner as shown in the sketch plan attached to the commissioner's report, said defendants have no other alternative except to remove and demolish part of their house that has encroached an area of five (5) sq. meters of the land allotted to the plaintiffs.

Thus, the repairs and improvements introduced by the said respondents after the complaint was filed cannot be considered to have been built in good faith, much less, justify the denial of the petitioner's fai-rn of option. WHEREFORE, judgment is hereby rendered assigning Lot 1161-A with an area of thirty (30) sq. meters to the plaintiffs spouses Concepcion Fernandez Abesia, Lourdes Fernandez Rodil, Genaro Fernandez and Dominga A. Fernandez, in the respective metes and bounds as shown in the subdivision sketch plan attached to the Commissioner's Report dated may 29, 1976 prepared by the Commissioner, Geodetic Engineer Espiritu Bunagan. Further, the defendants are hereby ordered at their expense to remove and demolish part of their house which has encroached an area of five (5) square meters from Lot 1161-A of the plaintiffs; within sixty (60) days from date hereof and to deliver the possession of the same to the plaintiffs. For the Commissioner's fee of P400.00, the defendants are ordered to pay, jointly and severally, the sum of P133.33 and the balance thereof to be paid by the plaintiffs. The costs of suit shall be paid by the plaintiffs and the defendants in the proportion of twothirds (2/3) and one-third (1/3) shares respectively. A certified copy of this judgment shall be recorded in the office of the Register of Deeds of the City of Cebu and the expense of such recording shall be taxed as a part of the costs of the action.

Since the improvements have been gutted by fire, and therefore, the basis for private respondent's right to retain the premises has already been extinguished without the fault of the petitioner, there is no other recourse for the private respondent but to vacate the premises and deliver the same to herein petitioner.

WHEREFORE, IN VIEW OF THE FOREGOING, the petition is GRANTED and the respondent judge is hereby ordered to immediately issue a writ of execution ordering the private respondent to vacate the disputed premises and deliver possession of the same to the petitioner.

SO ORDERED.

Page 32 of 46

Hence, this appeal interposed by the defendants with the following assignments of errors:

SO ORDERED.

PEDRO P. PECSON, petitioner, vs. COURT OF APPEALS, SPOUSES JUAN NUGUID and ERLINDA NUGUID, respondents.

THE TRIAL COURT ERRED IN NOT APPLYING THE RIGHTS OF A BUILDER IN GOOD FAITH UNDER ART. 448 OF THE NEW CIVIL CODE TO DEFENDANTSAPPELLANTS WITH RESPECT TO THAT PART OF THEIR HOUSE OCCUPYING A PROTION OF THE LOT ASSIGNED TO PLAINTIFFS-APPELLEES.

DAVIDE, JR., J.: This petition for review on certiorari seeks to set aside the decision 1 of the Court of Appeals in CA-G.R. SP No. 32679 affirming in part the order 2 of the Regional Trial Court (RTC) of Quezon City, Branch 101, in Civil Case No. Q-41470.

II

THE TRIAL COURT ERRED IN ORDERING DEFENDANTS-APPELLANTS TO REMOVE AND DEMOLISH AT THEIR EXPENSE, THAT PART OF THEIR HOUSE WHICH HAS ENCROACHED ON AN AREA OF FIVE SQUARE METERS OF LOT 1161-A OF PLAINTIFFS-APPELLEES.

The factual and procedural antecedents of this case as gathered from the record are as follows:

Article 448 of the New Civil Code provides as follows:

Petitioner Pedro P. Pecson was the owner of a commercial lot located in Kamias Street, Quezon City, on which he built a four-door two-storey apartment building. For his failure to pay realty taxes amounting to twelve thousand pesos (P12,000.00), the lot was sold at public auction by the city Treasurer of Quezon City to Mamerto Nepomuceno who in turn sold it on 12 October 1983 to the private respondents, the spouses Juan Nuguid and Erlinda Tan-Nuguid, for one hundred three thousand pesos (P103,000.00).

Art. 448. The owner of the land on which anything has been built, sown, or planted in good faith, shall have the right to appropriate as his own the works, sowing or planting, after payment of the indemnity provided for in articles 546 and 548, or to oblige the one who built or planted to pay the price of the land, and the one who sowed, the proper rent. However, the builder or planter cannot be obliged to buy the land if its value is considerably more than that of the building or trees. In such case, he shall pay reasonable rent, if the owner of the land does not choose to appropriate the building or trees after proper indemnity. The parties shall agree upon the terms of the lease and in case of disagreement, the court shall fix the terms thereof.

The petitioner challenged the validity of the auction sale in Civil Case No. Q-41470 before the RTC of Quezon City. In its decision of 8 February 1989, the RTC dismissed the complaint, but as to the private respondents' claim that the sale included the apartment building, it held that the issue concerning it was "not a subject of the . . . litigation." In resolving the private respondents' motion to reconsider this issue, the trial court held that there was no legal basis for the contention that the apartment building was included in the sale. 3

The court a quo correctly held that Article 448 of the Civil Code cannot apply where a co-owner builds, plants or sows on the land owned in common for then he did not build, plant or sow upon land that exclusively belongs to another but of which he is a co-owner. The co-owner is not a third person under the circumstances, and the situation is governed by the rules of co-ownership. 1

Both parties then appealed the decision to the Court of Appeals. The case was docketed as CA-G.R. CV No. 2931. In its decision of 30 April 1992, 4 the Court of Appeals affirmed in toto the assailed decision. It also agreed with the trial court that the apartment building was not included in the auction sale of the commercial lot. Thus: Indeed, examining the record we are fully convinced that it was only the land without the apartment building which was sold at the auction sale, for plaintiff's failure to pay the taxes due thereon. Thus, in the Certificate of Sale of Delinquent Property To Purchaser (Exh. K, p. 352, Record) the property subject of the auction sale at which Mamerto Nepomuceno was the purchaser is referred to as Lot No. 21-A, Block No. K34, at Kamias, Barangay Piahan, with an area of 256.3 sq. m., with no mention whatsoever, of the building thereon. The same description of the subject property appears in the Final Notice To Exercise The Right of Redemption (over subject property) dated September 14, 1981 (Exh. L, p. 353, Record) and in the Final Bill of Sale over the same property dated April 19, 1982 (Exh. P, p. 357, Record). Needless to say, as it was only the land without any building which Nepomuceno had acquired at the auction sale, it was also only that land without any building which he could have legally sold to the Nuguids. Verily, in the Deed of Absolute Sale of Registered Land executed by Mamerto Nepomuceno in favor of the Nuguids on October 25, 1983 (Exh. U, p. 366, Record) it clearly appears that the property subject of the sale for P103,000.00 was only the parcel of land, Lot 21-A, Blk. K-34 containing an area of 256.3 sq. meters, without any mention of any improvement, much less any building thereon. (emphases supplied) The petition to review the said decision was subsequently denied by this Court. 5 Entry of judgment was made on 23 June 1993. 6

However, when, as in this case, the co-ownership is terminated by the partition and it appears that the house of defendants overlaps or occupies a portion of 5 square meters of the land pertaining to plaintiffs which the defendants obviously built in good faith, then the provisions of Article 448 of the new Civil Code should apply. Manresa and Navarro Amandi agree that the said provision of the Civil Code may apply even when there was coownership if good faith has been established. 2

Applying the aforesaid provision of the Civil Code, the plaintiffs have the right to appropriate said portion of the house of defendants upon payment of indemnity to defendants as provided for in Article 546 of the Civil Code. Otherwise, the plaintiffs may oblige the defendants to pay the price of the land occupied by their house. However, if the price asked for is considerably much more than the value of the portion of the house of defendants built thereon, then the latter cannot be obliged to buy the land. The defendants shall then pay the reasonable rent to the plaintiff upon such terms and conditions that they may agree. In case of disagreement, the trial court shall fix the terms thereof. Of course, defendants may demolish or remove the said portion of their house, at their own expense, if they so decide.

WHEREFORE, the decision appealed from is hereby MODIFIED by ordering plaintiff to indemnify defendants for the value of the Id portion of the house of defendants in accordance with Article 546 of the Civil Code, if plaintiffs elect to appropriate the same. Otherwise, the defendants shall pay the value of the 5 square meters of land occupied by their house at such price as may be agreed upon with plaintiffs and if its value exceeds the portion of the house that defendants built thereon, the defendants may choose not to buy the land but defendants must pay a reasonable rental for the use of the portion of the land of plaintiffs As may be agreed upon between the parties. In case of disagreement, the rate of rental shall be determined by the trial court. Otherwise, defendants may remove or demolish at their own expense the said portion of their house. No costs.

On November 1993, the private respondents filed with the trial court a motion for delivery of possession of the lot and the apartment building, citing article 546 of the Civil Code. 7 Acting thereon, the trial court issued on 15 November 1993 the challenged order 8 which reads as follows:

Page 33 of 46

Submitted for resolution before this Court is an uncontroverted [sic] for the Delivery of Possession filed by defendants Erlinda Tan, Juan Nuguid, et al. considering that despite personal service of the Order for plaintiff to file within five (5) days his opposition to said motion, he did not file any.

possession of subject property located at No. 79 Kamias Road, Quezon City, with all the improvements thereon and to eject therefrom all occupants therein, their agents, assignees, heirs and representatives." 9

In support of defendant's motion, movant cites the law in point as Article 546 of the Civil Code . . .

The petitioner then filed with the Court of Appeals a special civil action for certiorari and prohibition assailing the order of 15 November 1993, which was docketed as CA-G.R. SP No. 32679. 10 In its decision of 7 June 1994, the Court of Appeals affirmed in part the order of the trial court citing Article 448 of the Civil Code. In disposing of the issues, it stated:

Movant agrees to comply with the provisions of the law considering that plaintiff is a builder in good faith and he has in fact, opted to pay the cost of the construction spent by plaintiff. From the complaint itself the plaintiff stated that the construction cost of the apartment is much more than the lot, which apartment he constructed at a cost of P53,000.00 in 1965 (par. 8 complaint). This amount of P53,000.00 is what the movant is supposed to pay under the law before a writ of possession placing him in possession of both the lot and apartment would be issued.

However, the complaint alleges in paragraph 9 that three doors of the apartment are being leased. This is further confirmed by the affidavit of the movant presented in support of the motion that said three doors are being leased at a rental of P7,000.00 a month each. The movant further alleges in his said affidavit that the present commercial value of the lot is P10,000.00 per square meter or P2,500,000.00 and the reasonable rental value of said lot is no less than P21,000.00 per month.

As earlier pointed out, private respondent opted to appropriate the improvement introduced by petitioner on the subject lot, giving rise to the right of petitioner to be reimbursed of the cost of constructing said apartment building, in accordance with Article 546 of the . . . Civil Code, and of the right to retain the improvements until he is reimbursed of the cost of the improvements, because, basically, the right to retain the improvement while the corresponding indemnity is not paid implies the tenancy or possession in fact of the land on which they are built . . . [2 TOLENTINO, CIVIL CODE OF THE PHILIPPINES (1992) p. 112]. With the facts extant and the settled principle as guides, we agree with petitioner that respondent judge erred in ordering that "the movant having been declared as the uncontested owner of the lot in question as per Entry of Judgment of the Supreme Court dated June 23, 1993, the plaintiff should pay rent to the movant of no less than P21,000 per month from said date as this is the very same amount paid monthly by the tenants occupying the lot.

The decision having become final as per Entry of Judgment dated June 23, 1993 and from this date on, being the uncontested owner of the property, the rents should be paid to him instead of the plaintiff collecting them. From June 23, 1993, the rents collected by plaintiff amounting to more than P53,000.00 from tenants should be offset from the rents due to the lot which according to movant's affidavit is more than P21,000.00 a month.

We, however, agree with the finding of respondent judge that the amount of P53,000.00 earlier admitted as the cost of constructing the apartment building can be offset from the amount of rents collected by petitioner from June 23, 1993 up to September 23, 1993 which was fixed at P7,000.00 per month for each of the three doors. Our underlying reason is that during the period of retention, petitioner as such possessor and receiving the fruits from the property, is obliged to account for such fruits, so that the amount thereof may be deducted from the amount of indemnity to be paid to him by the owner of the land, in line with Mendoza vs. De Guzman, 52 Phil. 164 . . . .

WHEREFORE, finding merit in the Motion, the Court hereby grants the following prayer that:

The Court of Appeals then ruled as follows:

1. The movant shall reimburse plaintiff the construction cost of P53,000.00.

2. The payment of P53,000.00 as reimbursement for the construction cost, movant Juan Nuguid is hereby entitled to immediate issuance of a writ of possession over the Lot and improvements thereon.

WHEREFORE, while it appears that private respondents have not yet indemnified petitioner with the cost of the improvements, since Annex I shows that the Deputy Sheriff has enforced the Writ of Possession and the premises have been turned over to the possession of private respondents, the quest of petitioner that he be restored in possession of the premises is rendered moot and academic, although it is but fair and just that private respondents pay petitioner the construction cost of P53,000.00; and that petitioner be ordered to account for any and all fruits of the improvements received by him starting on June 23, 1993, with the amount of P53,000.00 to be offset therefrom. IT IS SO ORDERED. 11

3. The movant having been declared as the uncontested owner of the Lot in question as per Entry of Judgment of the Supreme Court dated June 23, 1993, the plaintiff should pay rent to the movant of no less than P21,000.00 per month from said date as this is the very same amount paid monthly by the tenants occupying the lot.

Aggrieved by the Court of Appeals' decision, the petitioner filed the instant petition.

The parties agree that the petitioner was a builder in good faith of the apartment building on the theory that he constructed it at the time when he was still the owner of the lot, and that the key issue in this case is the application of Articles 448 and 456 of the Civil Code.

4. The amount of P53,000.00 due from the movant is hereby offset against the amount of rents collected by the plaintiff from June 23, 1993, to September 23, 1993.

The trial court and the Court of Appeals, as well as the parties, concerned themselves with the application of Articles 448 and 546 of the Civil Code. These articles read as follows:

SO ORDERED. Art. 448. The owner of the land on which anything has been built, sown or planted in good faith, shall have the right to appropriate as his own the works, sowing or planting, after payment of the indemnity provided for in articles 546 and 548, or to oblige the one who built or planted to pay the price of the land, and the one who sowed, the proper

The petitioner moved for the reconsideration of the order but it was not acted upon by the trial court. Instead, on 18 November 1993, it issued a writ of possession directing the deputy sheriff "to place said movant Juan Nuguid in

Page 34 of 46

rent. However, the builder or planter cannot be obliged to buy the land if its value is considerably more than that of the building or trees. In such case, he shall pay reasonable rent, if the owner of the land does not choose to appropriate the building or trees after proper indemnity. The parties shall agree upon the terms of the lease and in case of disagreement, the court shall fix the terms thereof. (361a)

not belong to him. Guided by this precept, it is therefore the current market value of the improvements which should be made the basis of reimbursement. A contrary ruling would unjustly enrich the private respondents who would otherwise be allowed to acquire a highly valued income-yielding four-unit apartment building for a measly amount. Consequently, the parties should therefore be allowed to adduce evidence on the present market value of the apartment building upon which the trial court should base its finding as to the amount of reimbursement to be paid by the landowner.

xxx xxx xxx The trial court also erred in ordering the petitioner to pay monthly rentals equal to the aggregate rentals paid by the lessees of the apartment building. Since the private respondents have opted to appropriate the apartment building, the petitioner is thus entitled to the possession and enjoyment of the apartment building, until he is paid the proper indemnity, as well as of the portion of the lot where the building has been constructed. This is so because the right to retain the improvements while the corresponding indemnity is not paid implies the tenancy or possession in fact of the land on which it is built, planted or sown. 18 The petitioner not having been so paid, he was entitled to retain ownership of the building and, necessarily, the income therefrom.

Art. 546. Necessary expenses shall be refunded to every possessor; but only the possessor in good faith may retain the thing until he has been reimbursed therefor.

Useful expenses shall be refunded only to the possessor in good faith with the same right of retention, the person who has defeated him in the possession having the option of refunding the amount of the expenses or of paying the increase in value which the thing may have acquired by reason thereof. (453a)

It follows, too, that the Court of Appeals erred not only in upholding the trial court's determination of the indemnity, but also in ordering the petitioner to account for the rentals of the apartment building from 23 June 1993 to 23 September 1993.

By its clear language, Article 448 refers to a land whose ownership is claimed by two or more parties, one of whom has built some works, or sown or planted something. The building, sowing or planting may have been made in good faith or in bad faith. The rule on good faith laid down in Article 526 of the Civil Code shall be applied in determining whether a builder, sower or planter had acted in good faith. 12

WHEREFORE, the decision of the Court of Appeals in CA-G.R. SP No. 32679 and the Order of 15 November 1993 of the Regional Trial Court, Branch 101, Quezon City in Civil Case No. Q-41470 are hereby SET ASIDE.

Article 448 does not apply to a case where the owner of the land is the builder, sower, or planter who then later loses ownership of the land by sale or donation. This Court said so in Coleongco vs. Regalado: 13

Article 361 of the old Civil Code is not applicable in this case, for Regalado constructed the house on his own land before he sold said land to Coleongco. Article 361 applies only in cases where a person constructs a building on the land of another in good or in bad faith, as the case may be. It does not apply to a case where a person constructs a building on his own land, for then there can be no question as to good or bad faith on the part of the builder.

The case is hereby remanded to the trial court for it to determine the current market value of the apartment building on the lot. For this purpose, the parties shall be allowed to adduce evidence on the current market value of the apartment building. The value so determined shall be forthwith paid by the private respondents to the petitioner otherwise the petitioner shall be restored to the possession of the apartment building until payment of the required indemnity.

No costs.

SO ORDERED. Elsewise stated, where the true owner himself is the builder of works on his own land, the issue of good faith or bad faith is entirely irrelevant.

SPOUSES JUAN NUGUID AND ERLINDA T. NUGUID, petitioners, vs. HON. COURT OF APPEALS AND PEDRO P. PECSON, respondents.

Thus in strict point of law, Article 448 is not apposite to the case at bar. Nevertheless, we believe that the provision therein on indemnity may be applied by analogy considering that the primary intent of Article 448 is to avoid a state of forced co-ownership and that the parties, including the two courts below, in the main agree that Articles 448 and 546 of the Civil Code are applicable and indemnity for the improvements may be paid although they differ as to the basis of the indemnity. QUISUMBING, J.: Article 546 does not specifically state how the value of the useful improvements should be determined. The respondent court and the private respondents espouse the belief that the cost of construction of the apartment building in 1965, and not its current market value, is sufficient reimbursement for necessary and useful improvements made by the petitioner. This position is, however, not in consonance with previous rulings of this Court in similar cases. In Javier vs. Concepcion, Jr., 14 this Court pegged the value of the useful improvements consisting of various fruits, bamboos, a house and camarin made of strong material based on the market value of the said improvements. In Sarmiento vs. Agana, 15 despite the finding that the useful improvement, a residential house, was built in 1967 at a cost of between eight thousand pesos (P8,000.00) to ten thousand pesos(P10,000.00), the landowner was ordered to reimburse the builder in the amount of forty thousand pesos (P40,000.00), the value of the house at the time of the trial. In the same way, the landowner was required to pay the "present value" of the house, a useful improvement, in the case of De Guzman vs. De la Fuente, 16 cited by the petitioner.

DECISION

This is a petition for review on certiorari of the Decision1 dated May 21, 2001, of the Court of Appeals in CA-G.R. CV No. 64295, which modified the Order dated July 31, 1998 of the Regional Trial Court (RTC) of Quezon City, Branch 101 in Civil Case No. Q-41470. The trial court ordered the defendants, among them petitioner herein Juan Nuguid, to pay respondent herein Pedro P. Pecson, the sum of P1,344,000 as reimbursement of unrealized income for the period beginning November 22, 1993 to December 1997. The appellate court, however, reduced the trial courts award in favor of Pecson from the said P1,344,000 to P280,000. Equally assailed by the petitioners is the appellate courts Resolution2 dated January 10, 2002, denying the motion for reconsideration.

The objective of Article 546 of the Civil Code is to administer justice between the parties involved. In this regard, this Court had long ago stated in Rivera vs. Roman Catholic Archbishop of Manila 17 that the said provision was formulated in trying to adjust the rights of the owner and possessor in good faith of a piece of land, to administer complete justice to both of them in such a way as neither one nor the other may enrich himself of that which does

It may be recalled that relatedly in our Decision dated May 26, 1995, in G.R. No. 115814, entitled Pecson v. Court of Appeals, we set aside the decision of the Court of Appeals in CA-G.R. SP No. 32679 and the Order dated November 15, 1993, of the RTC of Quezon City, Branch 101 and remanded the case to the trial court for the determination of the current market value of the four-door two-storey apartment building on the 256-square meter commercial lot.

The antecedent facts in this case are as follows:

Page 35 of 46

Pedro P. Pecson owned a commercial lot located at 27 Kamias Road, Quezon City, on which he built a four-door two-storey apartment building. For failure to pay realty taxes, the lot was sold at public auction by the City Treasurer of Quezon City to Mamerto Nepomuceno, who in turn sold it for P103,000 to the spouses Juan and Erlinda Nuguid.

Erlinda Nuguid] to the petitioner [Pedro Pecson] otherwise the petitioner shall be restored to the possession of the apartment building until payment of the required indemnity.

No costs. Pecson challenged the validity of the auction sale before the RTC of Quezon City in Civil Case No. Q-41470. In its Decision,3 dated February 8, 1989, the RTC upheld the spouses title but declared that the four-door two-storey apartment building was not included in the auction sale. 4 This was affirmed in toto by the Court of Appeals and thereafter by this Court, in its Decision5 dated May 25, 1993, in G.R. No. 105360 entitled Pecson v. Court of Appeals. SO ORDERED.12 [Emphasis supplied.]

On June 23, 1993, by virtue of the Entry of Judgment of the aforesaid decision in G.R. No. 105360, the Nuguids became the uncontested owners of the 256-square meter commercial lot.

In so ruling, this Court pointed out that: (1) Article 448 of the Civil Code is not apposite to the case at bar where the owner of the land is the builder, sower, or planter who then later lost ownership of the land by sale, but may, however, be applied by analogy; (2) the current market value of the improvements should be made as the basis of reimbursement; (3) Pecson was entitled to retain ownership of the building and, necessarily, the income therefrom; (4) the Court of Appeals erred not only in upholding the trial courts determination of the indemnity, but also in ordering Pecson to account for the rentals of the apartment building from June 23, 1993 to September 23, 1993. On the basis of this Courts decision in G.R. No. 115814, Pecson filed a Motion to Restore Possession and a Motion to Render Accounting, praying respectively for restoration of his possession over the subject 256-square meter commercial lot and for the spouses Nuguid to be directed to render an accounting under oath, of the income derived from the subject four-door apartment from November 22, 1993 until possession of the same was restored to him. In an Order13 dated January 26, 1996, the RTC denied the Motion to Restore Possession to the plaintiff averring that the current market value of the building should first be determined. Pending the said determination, the resolution of the Motion for Accounting was likewise held in abeyance. With the submission of the parties assessment and the reports of the subject realty, and the reports of the Quezon City Assessor, as well as the members of the duly constituted assessment committee, the trial court issued the following Order14 dated October 7, 1997, to wit:

As a result, the Nuguid spouses moved for delivery of possession of the lot and the apartment building.
6 7

In its Order of November 15, 1993, the trial court, relying upon Article 546 of the Civil Code, ruled that the Spouses Nuguid were to reimburse Pecson for his construction cost of P53,000, following which, the spouses Nuguid were entitled to immediate issuance of a writ of possession over the lot and improvements. In the same order the RTC also directed Pecson to pay the same amount of monthly rentals to the Nuguids as paid by the tenants occupying the apartment units or P21,000 per month from June 23, 1993, and allowed the offset of the amount of P53,000 due from the Nuguids against the amount of rents collected by Pecson from June 23, 1993 to September 23, 1993 from the tenants of the apartment. 8 Pecson duly moved for reconsideration, but on November 8, 1993, the RTC issued a Writ of Possession, 9directing the deputy sheriff to put the spouses Nuguid in possession of the subject property with all the improvements thereon and to eject all the occupants therein.

Aggrieved, Pecson then filed a special civil action for certiorari and prohibition docketed as CA-G.R. SP No. 32679 with the Court of Appeals. In its decision of June 7, 1994, the appellate court, relying upon Article 448 10 of the Civil Code, affirmed the order of payment of construction costs but rendered the issue of possession moot on appeal, thus:

On November 21, 1996, the parties manifested that they have arrived at a compromise agreement that the value of the said improvement/building is P400,000.00 The Court notes that the plaintiff has already receivedP300,000.00. However, when defendant was ready to pay the balance of P100,000.00, the plaintiff now insists that there should be a rental to be paid by defendants. Whether or not this should be paid by defendants, incident is hereby scheduled for hearing on November 12, 1997 at 8:30 a.m.

WHEREFORE, while it appears that private respondents [spouses Nuguid] have not yet indemnified petitioner [Pecson] with the cost of the improvements, since Annex I shows that the Deputy Sheriff has enforced the Writ of Possession and the premises have been turned over to the possession of private respondents, the quest of petitioner that he be restored in possession of the premises is rendered moot and academic, although it is but fair and just that private respondents pay petitioner the construction cost of P53,000.00; and that petitioner be ordered to account for any and all fruits of the improvements received by him starting on June 23, 1993, with the amount of P53,000.00 to be offset therefrom. IT IS SO ORDERED.11 [Underscoring supplied.]

Meantime, defendants are directed to pay plaintiff the balance of P100,000.00. SO ORDERED.15

On December 1997, after paying the said P100,000 balance to Pedro Pecson the spouses Nuguid prayed for the closure and termination of the case, as well as the cancellation of the notice of lis pendens on the title of the property on the ground that Pedro Pecsons claim for rentals was devoid of factual and legal bases. 16

Frustrated by this turn of events, Pecson filed a petition for review docketed as G.R. No. 115814 before this Court.

After conducting a hearing, the lower court issued an Order dated July 31, 1998, directing the spouses to pay the sum of P1,344,000 as reimbursement of the unrealized income of Pecson for the period beginning November 22, 1993 up to December 1997. The sum was based on the computation of P28,000/month rentals of the four-door apartment, thus: The Court finds plaintiffs motion valid and meritorious. The decision of the Supreme Court in the aforesaid case [Pecson vs. Court of Appeals, 244 SCRA 407] which set aside the Order of this Court of November 15, 1993 has in effect upheld plaintiffs right of possession of the building for as long as he is not fully paid the value thereof. It follows, as declared by the Supreme Court in said decision that the plaintiff is entitled to the income derived therefrom, thus

On May 26, 1995, the Court handed down the decision in G.R. No 115814, to wit:

WHEREFORE, the decision of the Court of Appeals in CA-G.R. SP No. 32679 and the Order of 15 November 1993 of the Regional Trial Court, Branch 101, Quezon City in Civil Case No. Q-41470 are hereby SET ASIDE.

The case is hereby remanded to the trial court for it to determine the current market value of the apartment building on the lot. For this purpose, the parties shall be allowed to adduce evidence on the current market value of the apartment building. The value so determined shall be forthwith paid by the private respondents [Spouses Juan and

...

Page 36 of 46

Records show that the plaintiff was dispossessed of the premises on November 22, 1993 and that he was fully paid the value of his building in December 1997. Therefore, he is entitled to the income thereof beginning on November 22, 1993, the time he was dispossessed, up to the time of said full payment, in December 1997, or a total of 48 months.

provides that a builder in good faith is entitled to full reimbursement for all the necessary and useful expenses incurred; it also gives him right of retention until full reimbursement is made.

The only question left is the determination of income of the four units of apartments per month. But as correctly pointed out by plaintiff, the defendants have themselves submitted their affidavits attesting that the income derived from three of the four units of the apartment building is P21,000.00 or P7,000.00 each per month, or P28,000.00 per month for the whole four units. Hence, at P28,000.00 per month, multiplied by 48 months, plaintiff is entitled to be paid by defendants the amount of P1,344,000.00. 17 The Nuguid spouses filed a motion for reconsideration but this was denied for lack of merit. 18 The Nuguid couple then appealed the trial courts ruling to the Court of Appeals, their action docketed as CA-G.R. CV No. 64295.

While the law aims to concentrate in one person the ownership of the land and the improvements thereon in view of the impracticability of creating a state of forced co-ownership,23 it guards against unjust enrichment insofar as the good-faith builders improvements are concerned. The right of retention is considered as one of the measures devised by the law for the protection of builders in good faith. Its object is to guarantee full and prompt reimbursement as it permits the actual possessor to remain in possession while he has not been reimbursed (by the person who defeated him in the case for possession of the property) for those necessary expenses and useful improvements made by him on the thing possessed. 24 Accordingly, a builder in good faith cannot be compelled to pay rentals during the period of retention 25 nor be disturbed in his possession by ordering him to vacate. In addition, as in this case, the owner of the land is prohibited from offsetting or compensating the necessary and useful expenses with the fruits received by the builder-possessor in good faith. Otherwise, the security provided by law would be impaired. This is so because the right to the expenses and the right to the fruits both pertain to the possessor, making compensation juridically impossible; and one cannot be used to reduce the other. 26

In the Court of Appeals, the order appealed from in CA-G.R. CV No. 64295, was modified. The CA reduced the rentals from P1,344,000 to P280,000 in favor of the appellee.19 The said amount represents accrued rentals from the determination of the current market value on January 31, 199720 until its full payment on December 12, 1997.

As we earlier held, since petitioners opted to appropriate the improvement for themselves as early as June 1993, when they applied for a writ of execution despite knowledge that the auction sale did not include the apartment building, they could not benefit from the lots improvement, until they reimbursed the improver in full, based on the current market value of the property. Despite the Courts recognition of Pecsons right of ownership over the apartment building, the petitioners still insisted on dispossessing Pecson by filing for a Writ of Possession to cover both the lot and the building. Clearly, this resulted in a violation of respondents right of retention. Worse, petitioners took advantage of the situation to benefit from the highly valued, income-yielding, four-unit apartment building by collecting rentals thereon, before they paid for the cost of the apartment building. It was only four years later that they finally paid its full value to the respondent. Petitioners interpretation of our holding in G.R. No. 115814 has neither factual nor legal basis. The decision of May 26, 1995, should be construed in connection with the legal principles which form the basis of the decision, guided by the precept that judgments are to have a reasonable intendment to do justice and avoid wrong. 27

Hence, petitioners state the sole assignment of error now before us as follows:

THE COURT OF APPEALS ERRED IN HOLDING PETITIONERS LIABLE TO PAY RENT OVER AND ABOVE THE CURRENT MARKET VALUE OF THE IMPROVEMENT WHEN SUCH WAS NOT PROVIDED FOR IN THE DISPOSITIVE PORTION OF THE SUPREME COURTS RULING IN G.R. No. 115814.

Petitioners call our attention to the fact that after reaching an agreed price of P400,000 for the improvements, they only made a partial payment of P300,000. Thus, they contend that their failure to pay the full price for the improvements will, at most, entitle respondent to be restored to possession, but not to collect any rentals. Petitioners insist that this is the proper interpretation of the dispositive portion of the decision in G.R. No. 115814, which states in part that "[t]he value so determined shall be forthwith paid by the private respondents [Spouses Juan and Erlinda Nuguid] to the petitioner [Pedro Pecson] otherwise the petitioner shall be restored to the possession of the apartment building until payment of the required indemnity." 21 Now herein respondent, Pecson, disagrees with herein petitioners contention. He argues that petitioners are wrong in claiming that inasmuch as his claim for rentals was not determined in the dispositive portion of the decision in G.R. No. 115814, it could not be the subject of execution. He points out that in moving for an accounting, all he asked was that the value of the fruits of the property during the period he was dispossessed be accounted for, since this Court explicitly recognized in G.R. No. 115814, he was entitled to the property. He points out that this Court ruled that "[t]he petitioner [Pecson] not having been so paid, he was entitled to retain ownership of the building and, necessarily, the income therefrom."22 In other words, says respondent, accounting was necessary. For accordingly, he was entitled to rental income from the property. This should be given effect. The Court could have very well specifically included rent (as fruit or income of the property), but could not have done so at the time the Court pronounced judgment because its value had yet to be determined, according to him. Additionally, he faults the appellate court for modifying the order of the RTC, thus defeating his right as a builder in good faith entitled to rental from the period of his dispossession to full payment of the price of his improvements, which spans from November 22, 1993 to December 1997, or a period of more than four years.

The text of the decision in G.R. No. 115814 expressly exempted Pecson from liability to pay rentals, for we found that the Court of Appeals erred not only in upholding the trial courts determination of the indemnity, but also in ordering him to account for the rentals of the apartment building from June 23, 1993 to September 23, 1993, the period from entry of judgment until Pecsons dispossession. As pointed out by Pecson, the dispositive portion of our decision in G.R. No. 115814 need not specifically include the income derived from the improvement in order to entitle him, as a builder in good faith, to such income. The right of retention, which entitles the builder in good faith to the possession as well as the income derived therefrom, is already provided for under Article 546 of the Civil Code.

Given the circumstances of the instant case where the builder in good faith has been clearly denied his right of retention for almost half a decade, we find that the increased award of rentals by the RTC was reasonable and equitable. The petitioners had reaped all the benefits from the improvement introduced by the respondent during said period, without paying any amount to the latter as reimbursement for his construction costs and expenses. They should account and pay for such benefits. We need not belabor now the appellate courts recognition of herein respondents entitlement to rentals from the date of the determination of the current market value until its full payment. Respondent is clearly entitled to payment by virtue of his right of retention over the said improvement.

It is not disputed that the construction of the four-door two-storey apartment, subject of this dispute, was undertaken at the time when Pecson was still the owner of the lot. When the Nuguids became the uncontested owner of the lot on June 23, 1993, by virtue of entry of judgment of the Courts decision, dated May 25, 1993, in G.R. No. 105360, the apartment building was already in existence and occupied by tenants. In its decision dated May 26, 1995 in G.R. No. 115814, the Court declared the rights and obligations of the litigants in accordance with Articles 448 and 546 of the Civil Code. These provisions of the Code are directly applicable to the instant case.

Under Article 448, the landowner is given the option, either to appropriate the improvement as his own upon payment of the proper amount of indemnity or to sell the land to the possessor in good faith. Relatedly, Article 546

WHEREFORE, the instant petition is DENIED for lack of merit. The Decision dated May 21, 2001 of the Court of Appeals in CA-G.R. CV No. 64295 is SET ASIDE and the Order dated July 31, 1998, of the Regional Trial Court, Branch 101, Quezon City, in Civil Case No. Q-41470 ordering the herein petitioners, Spouses Juan and Erlinda Nuguid, to account for the rental income of the four-door two-storey apartment building from November 1993 until December 1997, in the amount of P1,344,000, computed on the basis of Twenty-eight Thousand (P28,000.00) pesos monthly, for a period of 48 months, is hereby REINSTATED. Until fully paid, said amount of rentals should bear the legal rate of interest set at six percent (6%) per annum computed from the date of RTC judgment. If any portion thereof shall thereafter remain unpaid, despite notice of finality of this Courts judgment, said remaining

Page 37 of 46

unpaid amount shall bear the rate of interest set at twelve percent (12%) per annum computed from the date of said notice. Costs against petitioners.

SO ORDERED.

J.M. TUASON and CO., INC., petitioner, vs. ESTRELLA VDA. DE LUMANLAN and the COURT OF APPEALS (FIFTH DIVISION), respondents.

Tuason and Sison for petitioner. Jose Chuico and Wilfredo E. Dizon for respondents.

REYES, J.B.L., Actg. C.J.:

That the sales of the possessory rights claimed by the DEUDORS, are described in the lists submitted by them to the OWNERS which are attached hereto marked Annexes "B" and "C" and made part hereof. Whatever amounts may have been collected by the DEUDORS on account thereof, shall be deducted from the total sum of P1,201,063.00 to be paid to them. It shall be the joint and solidary obligation of the DEUDORS to make the buyer of the lots purportedly sold by them to recognize the title of the OWNERS over the property purportedly bought by them, and to make them sign, whenever possible, new contracts of purchase for said property at the current paces and terms specified by the OWNERS in their sales of lots in their subdivision known at "Sta. Mesa Heights Subdivision." The DEUDORS HEREBY advised the OWNERS that the buyer listed in Annex "B" herein with the annotation "continue" shall buy the lots respectively occupied by them and shall sign contracts, but the sums already paid by them to the DEUDORS amounting to P134,922.84 (subject to verification by the Court) shall be credited to the buyers and shall be deducted from the sums to be paid to the DEUDORS by the OWNERS. The DEUDORS also advise the OWNERS that, the buyers listed in Annex "C" herein with the annotation "Refund" have decided not to continue with their former contracts or purchases with the DEUDORS and the sums already paid by them to the DEUDORS TOTALLING P101,182.42 (subject to verification by the Court) shall be refunded to them by the OWNERS and deducted from the sums that may be due to the DEUDORS from the OWNERS (J.M. Tuason & Co., Inc. vs. Jaramillo, L-18932, Sept. 30, 1963);

J. M. Tuason & Co., Inc. petitioned for a review by certiorari of the decision issued by the Court of Appeals (Fifth Division) in its case CA-G.R. No. 27259-R, reversing the judgment rendered by the Court of First Instance of Rizal (Civil Case No. Q-4243) that ordered defendant (now respondent) Estrella Vda. de Lumanlan to vacate the lot occupied by her in Sta. Mesa Heights Subdivision, barrio Tatalon, Quezon City, and to remove therefrom the house and other structures constructed thereon, paying P240.00 a month until restoration of the premises to plaintiff.

The facts are stated in the decision of the Court of Appeals (accepted by both parties) in this wise:1wph1.t

. . . That in the complaint filed in this case by plaintiff, J. M. Tuason & Co., Inc., hereinafter called Tuason, on 30 April, 1969, the basis is that it being the registered owner of the property known as Santa Mesa Heights Subdivision, situated at Barrio North Tatalon, Quezon City, herein defendant sometime in April, 1949 unlawfully entered into possession of 800 square meters, and therein constructed his house so that plaintiff prayed for ejectment and damages for the occupancy; and defendant in her answer set forthaffirmative defense that on 12 March, 1949, she had bought the property she was occupying from one Pedro Deudor, and that in a compromise agreement between Pedro and Tuason on 16 March 1953, approved by the Court of First Instance of Quezon City, she was one of the buyers therein recognized, so that she asked that her rights be recognized and the complaint dismissed; but on the basis of the evidence presented by both parties in the trial, Lower Court sustained plaintiff, holding that Tuason being the registered owner, and the question being purely one of possession, therefore, defendant's said evidence was "completely immaterial". . . . (Page 2 of Decision, Annex "A" of Petition.)

Careful analysis of this paragraph of the compromise agreement will show that while the same created "a sort of contractual relation" between the J. M. Tuason & Co., Inc., and the Deudor vendees (as ruled by this Court in Evangelista vs. Deudor, ante), the same in no way obligated Tuason & Co. to sell to those buyers the lots occupied by them at the price stipulated with the Deudors, but at "the current prices and terms specified by the OWNERS (Tuason) in their sales of lots in their subdivision known as 'Sta. Mesa Heights Subdivision'". This is what is expressly provided. Further, the paragraph plainly imports that these buyers of the Deudors must "recognize the title of the OWNERS (Tuason) over the property purportedly bought by them" from the Deudors, and "sign, whenever possible, new contracts of purchase for said property"; and, if and when they do so, "the sums paid by them to the Deudors . . . shall be credited to the buyers." All that Tuason & Co. agreed to, therefore, was to grant the Deudor buyers preferential right to purchase "at current prices and terms" the lots occupied by them, upon their recognizing the title of Tuason & Co., Inc., and signing new contracts therefor; and to credit them for the amounts they had paid to the Deudors.

Upon the facts thus stated, the Fifth Division of the Court of Appeals held that, pursuant to this Supreme Court's ruling in Evangelista vs. Deudor, L-12826, September 10, 1959, the Compromise Agreement (Exh. 2) between the petitioner Tuason & Co. and the Deudors constituted a valid defense against the possessory action filed by Tuason & Co.; that under paragraph 7 of said Compromise Agreement, petitioner bound and committed itself to sell to respondent Lumanlan the lot occupied by her at a reasonable price; that said respondent had a right to compel petitioner to accept payment for the lot in question; and that the compromise agreement legalized the possession of respondent.

Nowhere in her answer did the respondent Estrella Vda. de Lumanlan claim that she had signed a new contract with J. M. Tuason & Co., Inc. for the purchase of the lot occupied. What is worse, instead of recognizing the title of the owners (Tuason & Co.) as required by the aforementioned compromise agreement, she charged in paragraph 6 of her special defense (Rec. on Appeal, p. 10) that "Pedro Deudor and his co-owners and the plaintiff herein . . .conspired together and helped each other . . . by entering into a supposed Compromise" whereby "Pedro Deudor and his co-owners renounced, ceded, waived and quitclaimed all their rights, title and interest in the property including the land sold to herein defendant, in favor of the plaintiff J. M. Tuason & Co., Inc., in consideration of the sum of P1,201,063.00, without the knowledge and consent, and much less the intervention of the herein defendant." In other words, the respondent Lumanlan in her answer repudiated and assailed the compromise between the Deudors and J. M. Tuason & Co. How then can she now claim to take advantage and derive rights from that compromise?

These pronouncements are assailed by the petitioner in this appeal as legally incorrect and contrary to the decisions of this Court.

Without the compromise agreement, Lumanlan must justify her possession on the basis of a pretended superiority of the Deudors' old Spanish informacion posesoria over Tuason's Certificate of Title No. 1267, traceable back to the original Certificate of Title No. 735 of Rizal, issued under the Registration Act No. 496. But, as ruled by this Court in previous cases, Lumanlan is by now barred from assailing the decree of registration in favor of Tuason & Co., Inc.'s predecessors twenty years after its issuance (Tiburcio vs. PHHC, L-13429, Oct. 31, 1959; Tuason & Co. vs. Bolaos, 95 Phil. 107; Tuason & Co. vs. Santiago, 99 Phil. 622-623; Tuason & Co. vs. Macalindong, supra; Tuason & Co. vs. Jaramillo, L-16827, Jan. 31, 1963).

The terms of the compromise agreement between the heirs of Telesforo Deudor and J. M. Tuason & Co. have been taken cognizance of in many decisions of this Court (Evangelista vs. Deudor, jam. cit; Deudor vs. J. M. Tuason & Co., L-18768, May 30, 1961, and L-20105, Oct. 31, 1963; J. M. Tuason vs. Jaramillo, et al., L-18932-34, Sept. 30, 1963; J. M. Tuason vs. Macalindong, L-15398, Dec. 29, 1962 and others). The Deudors had therein recognized the registered title of Tuason & Co. over the lands claimed by them, and received payment of certain sums of money; but as the Deudors had, prior to the compromise, sold their possessory rights to various persons, paragraph seventh of the compromise agreement (case Q-135 of the court of origin) provided:

It is thus apparent that no legal basis exists for the pronouncement in the appealed decision that Tuason & Co. had committed itself to sell to Lumanlan the lot occupied by her at a reasonable price, or that the compromise agreement legalized the possession of the respondent, since the latter does not rely on the compromise but, on the contrary, she assails it.

The Court of Appeals ruled that the price to be paid by Lumanlan to Tuason & Co., Inc., is governed by Article 1474 of the new Civil Code of the Philippines, which provides that:

Page 38 of 46

Where the price cannot be determined in accordance with the preceding articles, or in any other manner, the contract is inefficacious. However, if the thing or any part thereof has been delivered to and appropriated by the buyer, he must pay a reasonable price therefor. What is a reasonable price is a question of fact dependent on the circumstances of each particular case.

1. Dismissing the complaint for lack of cause of action;

2. Ordering Tecnogas to pay the sum of P2,000.00 per month as reasonable rental from October 4, 1979 until appellee vacates the land;

Since there has been no contract between petitioner Tuason & Co. and respondent Lumanlan for the sale of the lot occupied by the latter, and by paragraph 7 of the Compromise Agreement (assuming that respondent-appellee still has the right to invoke the same, and seek refuge thereunder), Tuason & Co. did not consider itself bound by the sales made by the Deudors, but demanded that the Deudor buyers should sign new contracts with it at current prices specified for the sales of lots in "Sta. Mesa Heights Subdivision" (ante) the aforequoted Article 1474 can have no bearing on the case, Lumanlan not being a buyer from Tuason & Co.

3. To remove the structures and surrounding walls on the encroached area;

4. Ordering appellee to pay the value of the land occupied by the two-storey building;

5. Ordering appellee to pay the sum of P20,000.00 for and as attorney's fees; As to Lumanlan's allegation in her counterclaim that she should be deemed a builder in good faith, a similar contention has been rejected in Tuason & Co. vs. Macalindong, L-15398, December 29, 1962, where we ruled that there being a presumptive knowledge of the Torrens titles issued to Tuason & Co. and its predecessors-in-interest since 1914, the buyer from the Deudors (or from their transferees) can not, in good conscience, say now that she believed her vendor had rights of ownership over the lot purchased. The reason given by the Court is that

6. Costs against appellee.

Had he investigated before buying and before building his house on the questioned lot, he would have been informed that the land is registered under the Torrens system in the name of J. M. Tuason & Co., Inc., If he failed to make the necessary inquiry, appellant is now bound conclusively by appellee's Torrens title (Sec. 51, Act 496; Emas vs. Zuzuarregui, 35 Phil. 144) (Tuason & Co., Inc. vs. Macalindong, ante).

Acting on the motions for reconsideration of both petitioner and private respondent, respondent Court ordered the deletion of paragraph 4 of the dispositive portion in an Amended Decision dated February 9, 1993, as follows: 4

WHEREFORE, premises considered, our decision of August 28, 1992 is hereby modified deleting paragraph 4 of the dispositive portion of our decision which reads:

Lumanlan had chosen to ignore the Torrens title of Tuason & Co., Inc. and relied instead upon the Deudors' claim of ownership, perhaps because such course appeared to her as more advantageous; hence, she has only herself to blame for the consequences now that the Deudors' claim has been abandoned by the Deudors themselves, and can not pretend good faith. The Court of First Instance, therefore, did not err in holding that she was not a rightful possessor and sentencing her to vacate.

4. Ordering appellee to pay the value of the land occupied by the two-storey building.

The motion for reconsideration of appellee is hereby DENIED for lack of merit.

Respondent could have asked that she recover or be credited with the amounts paid by her to the Deudors, but as no claim to such credit was ever advanced by her in the trial Court, no pronouncement can be made thereon in this appeal. Equity demands, however, that her right to claim such return, or to have the amount offset against the sums she was sentenced to pay, should be, as it is, reserved.

The foregoing Amended Decision is also challenged in the instant petition.

The Facts The facts are not disputed. Respondent Court merely reproduced the factual findings of the trial court, as follows: 5

WHEREFORE, the decision of the Court of Appeals is reversed and that of the Court of First Instance reinstated. Costs against respondent, Estrella Vda. de Lumanlan.

TECNOGAS PHILIPPINES MANUFACTURING CORPORATION, petitioner, vs. COURT OF APPEALS (FORMER SPECIAL SEVENTEENTH DIVISION) and EDUARDO UY, respondents.

PANGANIBAN, J.:

The parties in this case are owners of adjoining lots in Paraaque, Metro Manila. It was discovered in a survey, that a portion of a building of petitioner, which was presumably constructed by its predecessor-in-interest, encroached on a portion of the lot owned by private respondent. What are the rights and obligations of the parties? Is petitioner considered a builder in bad faith because, as held by respondent Court, he is "presumed to know the metes and bounds of his property as described in his certificate of title"? Does petitioner succeed into the good faith or bad faith of his predecessor-in-interest which presumably constructed the building? These are the questions raised in the petition for review of the Decision 1 dated August 28, 1992, in CA-G.R. CV No. 28293 of respondent Court 2 where the disposition reads: 3

WHEREFORE, premises considered, the Decision of the Regional Trial Court is hereby reversed and set aside and another one entered

That plaintiff (herein petitioner) which is a corporation duly organized and existing under and by virtue of Philippine laws is the registered owner of a parcel of land situated in Barrio San Dionisio, Paraaque, Metro Manila known as Lot 4331-A (should be 4531-A) of Lot 4531 of the Cadastral Survey of Paraaque, Metro Manila, covered by Transfer Certificate of Title No. 409316 of the Registry of Deeds of the Province of Rizal; that said land was purchased by plaintiff from Pariz Industries, Inc. in 1970, together with all the buildings and improvements including the wall existing thereon; that the defendant (herein private respondent) is the registered owner of a parcel of land known as Lot No. 4531-B of Lot 4531 of the Cadastral Survey of Paraaque, LRC (GLRO) Rec. No. 19645 covered by Transfer Certificate of Title No. 279838, of the Registry of Deeds for the Province of Rizal; that said land which adjoins plaintiff's land was purchased by defendant from a certain Enrile Antonio also in 1970; that in 1971, defendant purchased another lot also adjoining plaintiffs land from a certain Miguel Rodriguez and the same was registered in defendant's name under Transfer Certificate of Title No. 31390, of the Registry of Deeds for the Province of Rizal; that portions of the buildings and wall bought by plaintiff together with the land from Pariz Industries are occupying a portion of defendant's adjoining land; that upon learning of the encroachment or occupation by its buildings and wall of a portion of defendant's land, plaintiff offered to buy from defendant that particular portion of defendant's land occupied by portions of its buildings and wall with an area of 770 square meters, more or less, but defendant, however, refused the offer. In 1973, the parties entered into a private agreement before a certain Col. Rosales in Malacaang, wherein plaintiff agreed to demolish the wall at the back portion of its land thus giving to defendant possession of a portion of his land previously

Page 39 of 46

enclosed by plaintiff's wall; that defendant later filed a complaint before the office of Municipal Engineer of Paraaque, Metro Manila as well as before the Office of the Provincial Fiscal of Rizal against plaintiff in connection with the encroachment or occupation by plaintiff's buildings and walls of a portion of its land but said complaint did not prosper; that defendant dug or caused to be dug a canal along plaintiff's wall, a portion of which collapsed in June, 1980, and led to the filing by plaintiff of the supplemental complaint in the above-entitled case and a separate criminal complaint for malicious mischief against defendant and his wife which ultimately resulted into the conviction in court of defendant's wife for the crime of malicious mischief; that while trial of the case was in progress, plaintiff filed in Court a formal proposal for settlement of the case but said proposal, however, was ignored by defendant. After trial on the merits, the Regional Trial Court 6 of Pasay City, Branch 117, in Civil Case No. PQ-7631-P, rendered a decision dated December 4, 1989 in favor of petitioner who was the plaintiff therein. The dispositive portion reads: 7

Whether or not the respondent Court of Appeals erred in ordering the removal of the "structures and surrounding walls on the encroached area" and in withdrawing its earlier ruling in its August 28, 1992 decision for the petitioner "to pay for the value of the land occupied" by the building, only because the private respondent has "manifested its choice to demolish" it despite the absence of compulsory sale where the builder fails to pay for the land, and which "choice" private respondent deliberately deleted from its September 1, 1980 answer to the supplemental complaint in the Regional Trial Court.

In its Memorandum, petitioner poses the following issues:

A.

WHEREFORE, judgment is hereby rendered in favor of plaintiff and against defendant and ordering the latter to sell to plaintiff that portion of land owned by him and occupied by portions of plaintiff's buildings and wall at the price of P2,000.00 per square meter and to pay the former:

The time when to determine the good faith of the builder under Article 448 of the New Civil Code, is reckonedduring the period when it was actually being built; and in a case where no evidence was presented norintroduced as to the good faith or bad faith of the builder at that time, as in this case, he must be presumed to be a "builder in good faith," since "bad faith cannot be presumed." 9

B.

1. The sum of P44,000.00 to compensate for the losses in materials and properties incurred by plaintiff through thievery as a result of the destruction of its wall;

In a specific "boundary overlap situation" which involves a builder in good faith, as in this case, it is now well settled that the lot owner, who builds on the adjacent lot is not charged with "constructive notice" of the technical metes and bounds contained in their torrens titles to determine the exact and precise extent of his boundary perimeter. 10

2. The sum of P7,500.00 as and by way of attorney's fees; and C. 3. The costs of this suit. The respondent court's citation of the twin cases of Tuason & Co. v. Lumanlan and Tuason & Co. v.Macalindong is not the "judicial authority" for a boundary dispute situation between adjacent torrens titled lot owners, as the facts of the present case do not fall within nor square with the involved principle of a dissimilar case. 11

Appeal was duly interposed with respondent Court, which as previously stated, reversed and set aside the decision of the Regional Trial Court and rendered the assailed Decision and Amended Decision. Hence, this recourse under Rule 45 of the Rules of Court.

The Issues The petition raises the following issues: 8

D.

(A)

Quite contrary to respondent Uy's reasoning, petitioner Tecnogas continues to be a builder in good faith, even if it subsequently built/repaired the walls/other permanent structures thereon while the case a quo was pending and even while respondent sent the petitioner many letters/filed cases thereon. 12

Whether or not the respondent Court of Appeals erred in holding the petitioner a builder in bad faith because it is "presumed to know the metes and bounds of his property."

D.(E.)

(B)

The amicable settlement between the parties should be interpreted as a contract and enforced only in accordance with its explicit terms, and not over and beyond that agreed upon; because the courts do not have the power to create a contract nor expand its scope. 13

Whether or not the respondent Court of Appeals erred when it used the amicable settlement between the petitioner and the private respondent, where both parties agreed to the demolition of the rear portion of the fence, as estoppel amounting to recognition by petitioner of respondent's right over his property including the portions of the land where the other structures and the building stand, which were not included in the settlement.

E.(F.)

(C)

As a general rule, although the landowner has the option to choose between: (1) "buying the building built in good faith", or (2) "selling the portion of his land on which stands the building" under Article 448 of the Civil Code; the first option is not absolute, because an exception thereto, once it would be impractical for the landowner to choose

Page 40 of 46

to exercise the first alternative, i.e. buy that portion of the house standing on his land, for the whole building might be rendered useless. The workable solution is for him to select the second alternative, namely, to sell to the builder that part of his land on which was constructed a portion of the house. 14
15

improperly or wrongfully. 25 The good faith ceases from the moment defects in the title are made known to the possessor, by extraneous evidence or by suit for recovery of the property by the true owner. 26

Private respondent, on the other hand, argues that the petition is "suffering from the following flaws:

Recall that the encroachment in the present case was caused by a very slight deviation of the erected wall (as fence) which was supposed to run in a straight line from point 9 to point 1 of petitioner's lot. It was an error which, in the context of the attendant facts, was consistent with good faith. Consequently, the builder, if sued by the aggrieved landowner for recovery of possession, could have invoked the provisions of Art. 448 of the Civil Code, which reads:

1. It did not give the exact citations of cases decided by the Honorable Supreme Court that allegedly contradicts the ruling of the Hon. Court of Appeals based on the doctrine laid down in Tuason vs.Lumanlan case citing also Tuason vs. Macalindong case (Supra).

2. Assuming that the doctrine in the alleged Co Tao vs. Chico case is contradictory to the doctrine inTuason vs. Lumanlan and Tuason vs. Macalindong, the two cases being more current, the same should prevail.

The owner of the land on which anything has been built, sown or planted in good faith, shall have the right to appropriate as his own the works, sowing or planting, after payment of the indemnity provided for in articles 546 and 548, or to oblige the one who built or planted to pay the price of the land, and the one who sowed, the proper rent. However, the builder or planter cannot be obliged to buy the land if its value is considerably more than that of the building or trees. In such case, he shall pay reasonable rent, if the owner of the land does not choose to appropriate the building or trees after proper indemnity. The parties shall agree upon the terms of the lease and in case of disagreement, the court shall fix the terms thereof.

Further, private respondent contends that the following "unmistakably" point to the bad faith of petitioner: (1) private respondent's purchase of the two lots, "was ahead of the purchase by petitioner of the building and lot from Pariz Industries"; (2) the declaration of the General Manager of Tecnogas that the sale between petitioner and Pariz Industries "was not registered" because of some problems with China Banking Corporation; and (3) the Deed of Sale in favor of petitioner was registered in its name only in "the month of May 1973." 16

The obvious benefit to the builder under this article is that, instead of being outrightly ejected from the land, he can compel the landowner to make a choice between the two options: (1) to appropriate the building by paying the indemnity required by law, or (2) sell the land to the builder. The landowner cannot refuse to exercise either option and compel instead the owner of the building to remove it from the land. 27

The Court's Ru1ing The question, however, is whether the same benefit can be invoked by petitioner who, as earlier stated, is not the builder of the offending structures but possesses them as buyer.

The petition should be granted.

Good Faith or Bad Faith Respondent Court, citing the cases of J.M. Tuason & Co., Inc. vs. Vda. de Lumanlan 17 and J.M. Tuason & Co.,Inc. vs. Macalindong, 18 ruled that petitioner "cannot be considered in good faith" because as a land owner, it is "presumed to know the metes and bounds of his own property, specially if the same are reflected in a properly issued certificate of title. One who erroneously builds on the adjoining lot should be considered a builder in (b)ad (f)aith, there being presumptive knowledge of the Torrens title, the area, and the extent of the boundaries." 19

We answer such question in the affirmative.

We disagree with respondent Court. The two cases it relied upon do not support its main pronouncement that a registered owner of land has presumptive knowledge of the metes and bounds of its own land, and is therefore in bad faith if he mistakenly builds on an adjoining land. Aside from the fact that those cases had factual moorings radically different from those obtaining here, there is nothing in those cases which would suggest, however remotely, that bad faith is imputable to a registered owner of land when a part of his building encroaches upon a neighbor's land, simply because he is supposedly presumed to know the boundaries of his land as described in his certificate of title. No such doctrinal statement could have been made in those cases because such issue was not before the Supreme Court. Quite the contrary, we have rejected such a theory in Co Tao vs. Chico, 20 where we held that unless one is versed in the science of surveying, "no one can determine the precise extent or location of his property by merely examining his paper title."

In the first place, there is no sufficient showing that petitioner was aware of the encroachment at the time it acquired the property from Pariz Industries. We agree with the trial court that various factors in evidence adequately show petitioner's lack of awareness thereof. In any case, contrary proof has not overthrown the presumption of good faith under Article 527 of the Civil Code, as already stated, taken together with the disputable presumptions of the law on evidence. These presumptions state, under Section 3 (a) of Rule 131 of the Rules of Court, that the person is innocent of a crime or wrong; and under Section 3 (ff) of Rule 131, that the law has been obeyed. In fact, private respondent Eduardo Uy himself was unaware of such intrusion into his property until after 1971 when he hired a surveyor, following his purchase of another adjoining lot, to survey all his newly acquired lots. Upon being apprised of the encroachment, petitioner immediately offered to buy the area occupied by its building a species of conduct consistent with good faith.

In the second place, upon delivery of the property by Pariz Industries, as seller, to the petitioner, as buyer, the latter acquired ownership of the property. Consequently and as earlier discussed, petitioner is deemed to have stepped into the shoes of the seller in regard to all rights of ownership over the immovable sold, including the right to compel the private respondent to exercise either of the two options provided under Article 448 of the Civil Code.

Estoppel There is no question that when petitioner purchased the land from Pariz Industries, the buildings and other structures were already in existence. The record is not clear as to who actually built those structures, but it may well be assumed that petitioner's predecessor-in-interest, Pariz Industries, did so. Article 527 of the Civil Code presumes good faith, and since no proof exists to show that the encroachment over a narrow, needle-shaped portion of private respondent's land was done in bad faith by the builder of the encroaching structures, the latter should be presumed to have built them in good faith. 21 It is presumed that possession continues to be enjoyed in the same character in which it was acquired, until the contrary is proved. 22 Good faith consists in the belief of the builder that the land he is building on is his, and his ignorance of any defect or flaw in his title. 23 Hence, such good faith, by law, passed on to Pariz's successor, petitioner in this case. Further, "(w)here one derives title to property from another, the act, declaration, or omission of the latter, while holding the title, in relation to the property, is evidence against the former." 24 And possession acquired in good faith does not lose this character except in case and from the moment facts exist which show that the possessor is not unaware that he possesses the thing

Respondent Court ruled that the amicable settlement entered into between petitioner and private respondent estops the former from questioning the private respondent's "right" over the disputed property. It held that by undertaking to demolish the fence under said settlement, petitioner recognized private respondent's right over the property, and "cannot later on compel" private respondent "to sell to it the land since" private respondent "is under no obligation to sell." 28

We do not agree. Petitioner cannot be held in estoppel for entering into the amicable settlement, the pertinent portions of which read: 29

Page 41 of 46

That the parties hereto have agreed that the rear portion of the fence that separates the property of the complainant and respondent shall be demolished up to the back of the building housing the machineries which demolision (sic) shall be undertaken by the complainant at anytime.

accession, he is entitled to the ownership of the accessory thing. (3 Manresa 213; Bernardo vs. Bataclan, 37 Off. Gaz. 1382; Co Tao vs. Chan Chico, G.R. No. 49167, April 30, 1949; Article applied; see Cabral, et al. vs. Ibanez [S.C.] 52 Off. Gaz. 217; Marfori vs. Velasco, [C.A.] 52 Off. Gaz. 2050).

That the fence which serve(s) as a wall housing the electroplating machineries shall not be demolished in the mean time which portion shall be subject to negotiation by herein parties.

From the foregoing, it is clear that petitioner agreed only to the demolition of a portion of the wall separating the adjoining properties of the parties i.e. "up to the back of the building housing the machineries." But that portion of the fence which served as the wall housing the electroplating machineries was not to be demolished. Rather, it was to "be subject to negotiation by herein parties." The settlement may have recognized the ownership of private respondent but such admission cannot be equated with bad faith. Petitioner was only trying to avoid a litigation, one reason for entering into an amicable settlement. As was ruled in Osmea vs. Commission on Audit, 30

The private respondent's insistence on the removal of the encroaching structures as the proper remedy, which respondent Court sustained in its assailed Decisions, is thus legally flawed. This is not one of the remedies bestowed upon him by law. It would be available only if and when he chooses to compel the petitioner to buy the land at a reasonable price but the latter fails to pay such price. 33 This has not taken place. Hence, his options are limited to: (1) appropriating the encroaching portion of petitioner's building after payment of proper indemnity, or (2) obliging the latter to buy the lot occupied by the structure. He cannot exercise a remedy of his own liking. Neither is petitioner's prayer that private respondent be ordered to sell the land 34 the proper remedy. While that was dubbed as the "more workable solution" in Grana and Torralba vs. The Court of Appeals, et al., 35 it was not the relief granted in that case as the landowners were directed to exercise "within 30 days from this decision their option to either buy the portion of the petitioners' house on their land or sell to said petitioners the portion of their land on which it stands." 36 Moreover, in Grana and Torralba, the area involved was only 87 square meters while this case involves 520 square meters 37. In line with the case of Depra vs. Dumlao, 38 this case will have to be remanded to the trial court for further proceedings to fully implement the mandate of Art. 448. It is a rule of procedure for the Supreme Court to strive to settle the entire controversy in a single proceeding leaving no root or branch to bear the seeds of future litigation. 39

A compromise is a bilateral act or transaction that is expressly acknowledged as a juridical agreement by the Civil Code and is therein dealt with in some detail. "A compromise," declares Article 2208 of said Code, "is a contract whereby the parties, by making reciprocal concessions, avoid a litigation or put an end to one already commenced."

xxx xxx xxx

Petitioner, however, must also pay the rent for the property occupied by its building as prescribed by respondent Court from October 4, 1979, but only up to the date private respondent serves notice of its option upon petitioner and the trial court; that is, if such option is for private respondent to appropriate the encroaching structure. In such event, petitioner would have a right of retention which negates the obligation to pay rent. 40 The rent should however continue if the option chosen is compulsory sale, but only up to the actual transfer of ownership.

The Civil Code not only defines and authorizes compromises, it in fact encourages them in civil actions. Art. 2029 states that "The Court shall endeavor to persuade the litigants in a civil case to agree upon some fair compromise." . . .

The award of attorney's fees by respondent Court against petitioner is unwarranted since the action appears to have been filed in good faith. Besides, there should be no penalty on the right to litigate. 41

In the context of the established facts, we hold that petitioner did not lose its rights under Article 448 of the Civil Code on the basis merely of the fact that some years after acquiring the property in good faith, it learned about and aptly recognized the right of private respondent to a portion of the land occupied by its building. The supervening awareness of the encroachment by petitioner does not militate against its right to claim the status of a builder in good faith. In fact, a judicious reading of said Article 448 will readily show that the landowner's exercise of his option can only take place after the builder shall have come to know of the intrusion in short, when both parties shall have become aware of it. Only then will the occasion for exercising the option arise, for it is only then that both parties will have been aware that a problem exists in regard to their property rights.

WHEREFORE, premises considered, the petition is hereby GRANTED and the assailed Decision and the Amended Decision are REVERSED and SET ASIDE. In accordance with the case of Depra vs. Dumlao, 42 this case is REMANDED to the Regional Trial Court of Pasay City, Branch 117, for further proceedings consistent with Articles 448 and 546 43 of the Civil Code, as follows:

The trial court shall determine:

a) the present fair price of private respondent's 520 square-meter area of land; Options of Private Respondent b) the increase in value ("plus value") which the said area of 520 square meters may have acquired by reason of the existence of the portion of the building on the area;

What then is the applicable provision in this case which private respondent may invoke as his remedy: Article 448 or Article 450 31 of the Civil Code?

c) the fair market value of the encroaching portion of the building; and In view of the good faith of both petitioner and private respondent, their rights and obligations are to be governed by Art. 448. The essential fairness of this codal provision has been pointed out by Mme. Justice Ameurfina Melencio-Herrera, citing Manresa and applicable precedents, in the case of Depra vs. Dumlao, 32 to wit:

d) whether the value of said area of land is considerably more than the fair market value of the portion of the building thereon.

Where the builder, planter or sower has acted in good faith, a conflict of rights arises between the owners, and it becomes necessary to protect the owner of the improvements without causing injustice to the owner of the land. In view of the impracticality of creating a state of forced co-ownership, the law has provided a just solution by giving the owner of the land the option to acquire the improvements after payment of the proper indemnity, or to oblige the builder or planter to pay for the land and the sower to pay the proper rent. It is the owner of the land who is authorized to exercise the option, because his right is older, and because, by the principle of

2. After said amounts shall have been determined by competent evidence, the regional trial court shall render judgment as follows:

a) The private respondent shall be granted a period of fifteen (15) days within which to exercise his option under the law (Article 448, Civil Code), whether to appropriate the portion of the building as his own by paying to petitioner its fair market value, or to

Page 42 of 46

oblige petitioner to pay the price of said area. The amounts to be respectively paid by petitioner and private respondent, in accordance with the option thus exercised by written notice of the other party and to the court, shall be paid by the obligor within fifteen (15) days from such notice of the option by tendering the amount to the trial court in favor of the party entitled to receive it;

Is a lot buyer who constructs improvements on the wrong property erroneously delivered by the owner's agent, a builder in good faith? This is the main issue resolved in this petition for review on certiorari to reverse the Decision1 of the Court of Appeals2 in CA-G.R. No. 11040, promulgated on August 20, 1987.

b) If private respondent exercises the option to oblige petitioner to pay the price of the land but the latter rejects such purchase because, as found by the trial court, the value of the land is considerably more than that of the portion of the building, petitioner shall give written notice of such rejection to private respondent and to the trial court within fifteen (15) days from notice of private respondent's option to sell the land. In that event, the parties shall be given a period of fifteen (15) days from such notice of rejection within which to agree upon the terms of the lease, and give the trial court formal written notice of the agreement and its provisos. If no agreement is reached by the parties, the trial court, within fifteen (15) days from and after the termination of the said period fixed for negotiation, shall then fix the terms of the lease provided that the monthly rental to be fixed by the Court shall not be less than two thousand pesos (P2,000.00) per month, payable within the first five (5) days of each calendar month. The period for the forced lease shall not be more than two (2) years, counted from the finality of the judgment, considering the long period of time since 1970 that petitioner has occupied the subject area. The rental thus fixed shall be increased by ten percent (10%) for the second year of the forced lease. Petitioner shall not make any further constructions or improvements on the building. Upon expiration of the two-year period, or upon default by petitioner in the payment of rentals for two (2) consecutive months, private respondent shall be entitled to terminate the forced lease, to recover his land, and to have the portion of the building removed by petitioner or at latter's expense. The rentals herein provided shall be tendered by petitioner to the trial court for payment to private respondent, and such tender shall constitute evidence of whether or not compliance was made within the period fixed by the said court.

By resolution dated November 13, 1995, the First Division of this Court resolved to transfer this case (along with several others) to the Third Division. After due deliberation and consultation, the Court assigned the writing of this Decision to the undersigned ponente.

The Facts

The facts, as found by respondent Court, are as follows:

Edith Robillo purchased from petitioner a parcel of land designated as Lot 9, Phase II and located at Taculing Road, Pleasantville Subdivision, Bacolod City. In 1975, respondent Eldred Jardinico bought the rights to the lot from Robillo. At that time, Lot 9 was vacant.

Upon completing all payments, Jardinico secured from the Register of Deeds of Bacolod City on December 19, 1978 Transfer Certificate of Title No. 106367 in his name. It was then that he discovered that improvements had been introduced on Lot 9 by respondent Wilson Kee, who had taken possession thereof.

c) In any event, petitioner shall pay private respondent an amount computed at two thousand pesos (P2,000.00) per month as reasonable compensation for the occupancy of private respondent's land for the period counted from October 4, 1979, up to the date private respondent serves notice of its option to appropriate the encroaching structures, otherwise up to the actual transfer of ownership to petitioner or, in case a forced lease has to be imposed, up to the commencement date of the forced lease referred to in the preceding paragraph;

It appears that on March 26, 1974, Kee bought on installment Lot 8 of the same subdivision from C.T. Torres Enterprises, Inc. (CTTEI), the exclusive real estate agent of petitioner. Under the Contract to Sell on Installment, Kee could possess the lot even before the completion of all installment payments. On January 20, 1975, Kee paid CTTEI the relocation fee of P50.00 and another P50.00 on January 27, 1975, for the preparation of the lot plan. These amounts were paid prior to Kee's taking actual possession of Lot 8. After the preparation of the lot plan and a copy thereof given to Kee, CTTEI through its employee, Zenaida Octaviano, accompanied Kee's wife, Donabelle Kee, to inspect Lot 8. Unfortunately, the parcel of land pointed by Octaviano was Lot 9. Thereafter, Kee proceeded to construct his residence, a store, an auto repair shop and other improvements on the lot.

After discovering that Lot 9 was occupied by Kee, Jardinico confronted him. The parties tried to reach an amicable settlement, but failed.

d) The periods to be fixed by the trial court in its decision shall be non-extendible, and upon failure of the party obliged to tender to the trial court the amount due to the obligee, the party entitled to such payment shall be entitled to an order of execution for the enforcement of payment of the amount due and for compliance with such other acts as may be required by the prestation due the obligee.

On January 30, 1981, Jardinico's lawyer wrote Kee, demanding that the latter remove all improvements and vacate Lot 9. When Kee refused to vacate Lot 9, Jardinico filed with the Municipal Trial Court in Cities, Branch 3, Bacolod City (MTCC), a complaint for ejectment with damages against Kee.

Kee, in turn, filed a third-party complaint against petitioner and CTTEI.

No costs.

SO ORDERED.

The MTCC held that the erroneous delivery of Lot 9 to Kee was attributable to CTTEI. It further ruled that petitioner and CTTEI could not successfully invoke as a defense the failure of Kee to give notice of his intention to begin construction required under paragraph 22 of the Contract to Sell on Installment and his having built a sari-sari store without the prior approval of petitioner required under paragraph 26 of said contract, saying that the purpose of these requirements was merely to regulate the type of improvements to be constructed on the Lot.3

PLEASANTVILLE DEVELOPMENT CORPORATION, petitioner, vs. COURT OF APPEALS, WILSON KEE, C.T. TORRES ENTERPRISES, INC. and ELDRED JARDINICO,respondents.

However, the MTCC found that petitioner had already rescinded its contract with Kee over Lot 8 for the latter's failure to pay the installments due, and that Kee had not contested the rescission. The rescission was effected in 1979, before the complaint was instituted. The MTCC concluded that Kee no longer had any right over the lot subject of the contract between him and petitioner. Consequently, Kee must pay reasonable rentals for the use of Lot 9, and, furthermore, he cannot claim reimbursement for the improvements he introduced on said lot.

DECISION The MTCC thus disposed: PANGANIBAN, J.: IN VIEW OF ALL THE FOREGOING, judgment is hereby rendered as follows:

Page 43 of 46

1. Defendant Wilson Kee is ordered to vacate the premises of Lot 9, covered by TCT No. 106367 and to remove all structures and improvements he introduced thereon;

b. If Jardinico prefers that Kee buy the land, the third-party defendants shall answer for the amount representing the value of Lot 9 that Kee should pay to Jardinico.

2. Defendant Wilson Kee is ordered to pay to the plaintiff rentals at the rate of P15.00 a day computed from the time this suit was filed on March 12, 1981 until he actually vacates the premises. This amount shall bear interests (sic) at the rate of 12 per cent (sic) per annum.

3. Third-party defendants C.T. Torres Enterprises, Inc. and Pleasantville Development Corporation are ordered to pay in solidum the amount of P3,000.00 to Jardinico as attorney's fees, as well as litigation expenses.

3. Third-Party Defendant C.T. Torres Enterprises, Inc. and Pleasantville Subdivision are ordered to pay the plaintiff jointly and severally the sum of P3,000.00 as attorney's fees and P700.00 as cost and litigation expenses.4

4. The award of rentals to Jardinico is dispensed with.

On appeal, the Regional Trial Court, Branch 48, Bacolod City (RTC) ruled that petitioner and CTTEI were not at fault or were not negligent, there being no preponderant evidence to show that they directly participated in the delivery of Lot 9 to Kee5. It found Kee a builder in bad faith. It further ruled that even assuming arguendo that Kee was acting in good faith, he was, nonetheless, guilty of unlawfully usurping the possessory right of Jardinico over Lot 9 from the time he was served with notice to vacate said lot, and thus was liable for rental.

Furthermore, the case is REMANDED to the court of origin for the determination of the actual value of the improvements and the property (Lot 9), as well as for further proceedings in conformity with Article 448 of the New Civil Code.7

Petitioner then filed the instant petition against Kee, Jardinico and CTTEI.

The Issues The RTC thus disposed: The petition submitted the following grounds to justify a review of the respondent Court's Decision, as follows: WHEREFORE, the decision appealed from is affirmed with respect to the order against the defendant to vacate the premises of Lot No. 9 covered by Transfer Certificate of Title No. T-106367 of the land records of Bacolod City; the removal of all structures and improvements introduced thereon at his expense and the payment to plaintiff (sic) the sum of Fifteen (P15.00) Pesos a day as reasonable rental to be computed from January 30, 1981, the date of the demand, and not from the date of the filing of the complaint, until he had vacated (sic) the premises, with interest thereon at 12% per annum. This Court further renders judgment against the defendant to pay the plaintiff the sum of Three Thousand (P3,000.00) Pesos as attorney's fees, plus costs of litigation.

1. The Court of Appeals has decided the case in a way probably not in accord with law or the the (sic) applicable decisions of the Supreme Court on third-party complaints, by ordering third-party defendants to pay the demolition expenses and/or price of the land;

2. The Court of Appeals has so far departed from the accepted course of judicial proceedings, by granting to private respondent-Kee the rights of a builder in good faith in excess of what the law provides, thus enriching private respondent Kee at the expense of the petitioner;

The third-party complaint against Third-Party Defendants Pleasantville Development Corporation and C.T. Torres Enterprises, Inc. is dismissed. The order against Third-Party Defendants to pay attorney's fees to plaintiff and costs of litigation is reversed.6

3. In the light of the subsequent events or circumstances which changed the rights of the parties, it becomes imperative to set aside or at least modify the judgment of the Court of Appeals to harmonize with justice and the facts;

Following the denial of his motion for reconsideration on October 20, 1986, Kee appealed directly to the Supreme Court, which referred the matter to the Court of Appeals. 4. Private respondent-Kee in accordance with the findings of facts of the lower court is clearly a builder in bad faith, having violated several provisions of the contract to sell on installments; The appellate court ruled that Kee was a builder in good faith, as he was unaware of the "mix-up" when he began construction of the improvements on Lot 8. It further ruled that the erroneous delivery was due to the negligence of CTTEI, and that such wrong delivery was likewise imputable to its principal, petitioner herein. The appellate court also ruled that the award of rentals was without basis.

5. The decision of the Court of Appeals, holding the principal, Pleasantville Development Corporation (liable) for the acts made by the agent in excess of its authority is clearly in violation of the provision of the law;

Thus, the Court of Appeals disposed: 6. The award of attorney's fees is clearly without basis and is equivalent to putting a premium in (sic) court litigation. WHEREFORE, the petition is GRANTED, the appealed decision is REVERSED, and judgment is rendered as follows: From these grounds, the issues could be re-stated as follows: 1. Wilson Kee is declared a builder in good faith with respect to the improvements he introduced on Lot 9, and is entitled to the rights granted him under Articles 448, 546 and 548 of the New Civil Code.

(1) Was Kee a builder in good faith?

2. Third-party defendants C.T. Torres Enterprises, Inc. and Pleasantville Development Corporation are solidarily liable under the following circumstances:

(2) What is the liability, if any, of petitioner and its agent, C.T. Torres Enterprises, Inc.? and

(3) Is the award of attorney's fees proper? A. If Eldred Jardinico decides to appropriate the improvements and, thereafter, remove these structures, the third-party defendants shall answer for all demolition expenses and the value of the improvements thus destroyed or rendered useless;

The First Issue: Good Faith

Page 44 of 46

Petitioner contends that the Court of Appeals erred in reversing the RTC's ruling that Kee was a builder in bad faith.

Petitioner fails to persuade this Court to abandon the findings and conclusions of the Court of Appeals that Kee was a builder in good faith. We agree with the following observation of the Court of Appeals:

13. The Vendee hereby declares that prior to the execution of his contract he/she has personally examined or inspected the property made subject-matter hereof, as to its location, contours, as well as the natural condition of the lots and from the date hereof whatever consequential change therein made due to erosion, the said Vendee shall bear the expenses of the necessary fillings, when the same is so desired by him/her.11

The roots of the controversy can be traced directly to the errors committed by CTTEI, when it pointed the wrong property to Wilson Kee and his wife. It is highly improbable that a purchaser of a lot would knowingly and willingly build his residence on a lot owned by another, deliberately exposing himself and his family to the risk of being ejected from the land and losing all improvements thereon, not to mention the social humiliation that would follow.

The subject matter of this provision of the contract is the change of the location, contour and condition of the lot due to erosion. It merely provides that the vendee, having examined the property prior to the execution of the contract, agrees to shoulder the expenses resulting from such change.

Under the circumstances, Kee had acted in the manner of a prudent man in ascertaining the identity of his property. Lot 8 is covered by Transfer Certificate of Title No. T-69561, while Lot 9 is identified in Transfer Certificate of Title No. T-106367. Hence, under the Torrens system of land registration, Kee is presumed to have knowledge of the metes and bounds of the property with which he is dealing. . . .

We do not agree with the interpretation of petitioner that Kee contracted away his right to recover damages resulting from petitioner's negligence. Such waiver would be contrary to public policy and cannot be allowed. "Rights may be waived, unless the waiver is contrary to law, public order, public policy, morals, or good customs, or prejudicial to a third person with a right recognized by law." 12

The Second Issue: Petitioner's Liability xxx xxx xxx Kee filed a third-party complaint against petitioner and CTTEI, which was dismissed by the RTC after ruling that there was no evidence from which fault or negligence on the part of petitioner and CTTEI can be inferred. The Court of Appeals disagreed and found CTTEI negligent for the erroneous delivery of the lot by Octaviano, its employee.

But as Kee is a layman not versed in the technical description of his property, he had to find a way to ascertain that what was described in TCT No. 69561 matched Lot 8. Thus, he went to the subdivision developer's agent and applied and paid for the relocation of the lot, as well as for the production of a lot plan by CTTEI's geodetic engineer. Upon Kee's receipt of the map, his wife went to the subdivision site accompanied by CTTEI's employee, Octaviano, who authoritatively declared that the land she was pointing to was indeed Lot 8. Having full faith and confidence in the reputation of CTTEI, and because of the company's positive identification of the property, Kee saw no reason to suspect that there had been a misdelivery. The steps Kee had taken to protect his interests were reasonable. There was no need for him to have acted ex-abundantia cautela, such as being present during the geodetic engineer's relocation survey or hiring an independent geodetic engineer to countercheck for errors, for the final delivery of subdivision lots to their owners is part of the regular course of everyday business of CTTEI. Because of CTTEI's blunder, what Kee had hoped to forestall did in fact transpire. 8 Kee's efforts all went to naught.

Petitioner does not dispute the fact that CTTEI was its agent. But it contends that the erroneous delivery of Lot 9 to Kee was an act which was clearly outside the scope of its authority, and consequently, CTTEI I alone should be liable. It asserts that "while [CTTEI] was authorized to sell the lot belonging to the herein petitioner, it was never authorized to deliver the wrong lot to Kee" 13.

Petitioner's contention is without merit.

Good faith consists in the belief of the builder that the land he is building on is his and his ignorance of any defect or flaw in his title 9. And as good faith is presumed, petitioner has the burden of proving bad faith on the part of Kee 10.

The rule is that the principal is responsible for the acts of the agent, done within the scope of his authority, and should bear the damage caused to third persons 14. On the other hand, the agent who exceeds his authority is personally liable for the damage 15

At the time he built improvements on Lot 8, Kee believed that said lot was what he bought from petitioner. He was not aware that the lot delivered to him was not Lot 8. Thus, Kee's good faith. Petitioner failed to prove otherwise.

CTTEI was acting within its authority as the sole real estate representative of petitioner when it made the delivery to Kee. In acting within its scope of authority, it was, however, negligent. It is this negligence that is the basis of petitioner's liability, as principal of CTTEI, per Articles 1909 and 1910 of the Civil Code.

To demonstrate Kee's bad faith, petitioner points to Kee's violation of paragraphs 22 and 26 of the Contract of Sale on Installment.

Pending resolution of the case before the Court of Appeals, Jardinico and Kee on July 24, 1987 entered into a deed of sale, wherein the former sold Lot 9 to Kee. Jardinico and Kee did not inform the Court of Appeals of such deal.

We disagree. Such violations have no bearing whatsoever on whether Kee was a builder in good faith, that is, on his state of mind at the time he built the improvements on Lot 9. These alleged violations may give rise to petitioner's cause of action against Kee under the said contract (contractual breach), but may not be bases to negate the presumption that Kee was a builder in good faith.

The deed of sale contained the following provision:

1. That Civil Case No. 3815 entitled "Jardinico vs. Kee" which is now pending appeal with the Court of Appeals, regardless of the outcome of the decision shall be mutually disregarded and shall not be pursued by the parties herein and shall be considered dismissed and without effect whatso-ever; 16

Petitioner also points out that, as found by the trial court, the Contract of Sale on Installment covering Lot 8 between it and Kee was rescinded long before the present action was instituted. This has no relevance on the liability of petitioner, as such fact does not negate the negligence of its agent in pointing out the wrong lot. to Kee. Such circumstance is relevant only as it gives Jardinico a cause of action for unlawful detainer against Kee.

Kee asserts though that the "terms and conditions in said deed of sale are strictly for the parties thereto" and that "(t)here is no waiver made by either of the parties in said deed of whatever favorable judgment or award the honorable respondent Court of Appeals may make in their favor against herein petitioner Pleasantville Development Corporation and/or private respondent C.T. Torres Enterprises; Inc." 17

Petitioner next contends that Kee cannot "claim that another lot was erroneously pointed out to him" because the latter agreed to the following provision in the Contract of Sale on installment, to wit: Obviously, the deed of sale can have no effect on the liability of petitioner. As we have earlier stated, petitioner's liability is grounded on the negligence of its agent. On the other hand, what the deed of sale regulates are the

Page 45 of 46

reciprocal rights of Kee and Jardinico; it stressed that they had reached an agreement independent of the outcome of the case.

There is also no further need, as ruled by the appellate Court, to remand the case to the court of origin "for determination of the actual value of the improvements and the property (Lot 9), as well as for further proceedings in conformity with Article 448 of the New Civil Code."

Petitioner further assails the following holding of the Court of Appeals: WHEREFORE , the petition is partially GRANTED. The Decision of the Court of Appeals is hereby MODIFIED as follows: 2. Third-party defendants C.T. Torres Enterprises, Inc. and Pleasantville Development Corporation are solidarily liable under the following circumstances: (1) Wilson Kee is declared a builder in good faith; a. If Eldred Jardinico decides to appropriate the improvements and, thereafter, remove these structures, the third-party defendants shall answer for all demolition expenses and the value of the improvements thus destroyed or rendered useless;

(2) Petitioner Pleasantville Development Corporation and respondent C.T. Torres Enterprises, Inc. are declared solidarily liable for damages due to negligence; however, since the amount and/or extent of such damages was not proven during the trial, the same cannot now be quantified and awarded;

b. If Jardinico prefers that Kee buy the land, the third-party defendants shall answer for the amount representing the value of Lot 9 that Kee should pay to Jardinico. 18

(3) Petitioner Pleasantville Development Corporation and respondent C.T. Torres Enterprises, Inc. are ordered to pay in solidum the amount of P3,000.00 to Jardinico as attorney's fees, as well as litigation expenses; and

(4) The award of rentals to Jardinico is dispensed with. Petitioner contends that if the above holding would be carried out, Kee would be unjustly enriched at its expense. In other words, Kee would be able to own the lot, as buyer, without having to pay anything on it, because the aforequoted portion of respondent Court's Decision would require petitioner and CTTEI jointly and solidarily to "answer" or reimburse Kee therefor.

SO ORDERED.

We agree with petitioner.

Petitioner' s liability lies in the negligence of its agent CTTEI. For such negligence, the petitioner should be held liable for damages. Now, the extent and/or amount of damages to be awarded is a factual issue which should be determined after evidence is adduced. However, there is no showing that such evidence was actually presented in the trial court; hence no damages could flow be awarded.

The rights of Kee and Jardinico vis-a-vis each other, as builder in good faith and owner in good faith, respectively, are regulated by law (i.e., Arts. 448, 546 and 548 of the Civil Code). It was error for the Court of Appeals to make a "slight modification" in the application of such law, on the ground of "equity". At any rate, as it stands now, Kee and Jardinico have amicably settled through their deed of sale their rights and obligations with regards to Lot 9. Thus, we delete items 2 (a) and (b) of the dispositive portion of the Court of Appeals' Decision [as reproduced above] holding petitioner and CTTEI solidarily liable.

The Third Issue: Attorney's Fees

The MTCC awarded Jardinico attorney's fees and costs in the amount of P3,000.00 and P700.00, respectively, as prayed for in his complaint. The RTC deleted the award, consistent with its ruling that petitioner was without fault or negligence. The Court of Appeals, however, reinstated the award of attorney's fees after ruling that petitioner was liable for its agent's negligence.

The award of attorney's fees lies within the discretion of the court and depends upon the circumstances of each case 19. We shall not interfere with the discretion of the Court of Appeals. Jardinico was compelled to litigate for the protection of his interests and for the recovery of damages sustained as a result of the negligence of petitioner's agent 20.

In sum, we rule that Kee is a builder in good faith. The disposition of the Court of Appeals that Kee "is entitled to the rights granted him under Articles 448, 546 and 548 of the New Civil Code" is deleted, in view of the deed of sale entered into by Kee and Jardinico, which deed now governs the rights of Jardinico and Kee as to each other.

Page 46 of 46

You might also like